(2008) AAP (American Academy of Pediatrics) Prep PDF
(2008) AAP (American Academy of Pediatrics) Prep PDF
(2008) AAP (American Academy of Pediatrics) Prep PDF
Question: 1
A 10-year-old girl has had a rash for 4 days without other symptoms. She is taking no
medications. Physical examination reveals erythematous cheeks (Item Q1A) and a lacy,
reticulated erythema involving the extremities (Item Q1B).
A. erythema infectiosum
B. phototoxic reaction
D. scarlet fever
kamel hassan
c=AD,
o=msd,
ou=1,
Reason: I agree to the terms defined by the placement of my signature in this document
Question: 1
Courtesy of D. Krowchuk
Question: 1
Courtesy of D. Krowchuk
References:
American Academy of Pediatrics. Parvovirus B19 (erythema infectiosum, fifth disease). In:
Pickering LK, Baker CJ, Long SS, McMillian JA, eds. Red Book: 2006 Report of the Committee
on Infectious Diseases. 27th ed. Elk Grove Village, Ill: American Academy of Pediatrics;
2006:484-487
Krowchuk DP, Mancini AJ, eds. Erythema infectiosum/human parvovirus B19 infection (fifth
disease). In: Pediatric Dermatology. A Quick Reference Guide. Elk Grove Village, Ill: American
Academy of Pediatrics; 2007:87-90
Paller AS, Mancini AJ. Photosensitivity and photoreactions. In: Hurwitz Clinical Pediatric
Dermatology. 3rd ed. Philadelphia, Pa: Elsevier Saunders; 2006:503-523
Weston WL, Lane AT, Morelli JG. Viral infections. In: Color Textbook of Pediatric Dermatology.
3rd ed. St. Louis, Mo: Mosby; 2002:89-118
Critique: 1
The rash of erythema infectiosum begins as confluent erythema of the cheeks, the so-called
"slapped-cheek" appearance.
Courtesy of D. Krowchuk
Critique: 1
Courtesy of D. Krowchuk
Critique: 1
Papular purpuric "gloves and socks" syndrome is a unique manifestation of parvovirus B19
infection characterized by a petechial or purpuric eruption on the hands and feet.
Courtesy of dermatlas.org
Critique: 1
Courtesy of M. Rimsza
Critique: 1
Courtesy of dermatlas.org
Critique: 1
Patients who have systemic lupus erythematosus often have an erythematous, scaling malar
eruption.
Courtesy of dermatlas.org
Question: 2
An 8-month-old infant presents with the primary complaint of irritability. He has been exclusively
breastfed since birth. His mother was not interested in providing any supplemental foods
because her milk supply has been adequate. Physical examination reveals a fussy infant who
has frontal bossing and whose weight and height are both at the 25th percentile. The infant
becomes irritable with movement of the left arm. Arm radiography reveals a humeral fracture
and bowing of both radii. Chest radiography demonstrates enlargement of the costochondral
junctions.
A. congenital syphilis
B. osteogenesis imperfecta
E. vitamin E deficiency
References:
Holick MF. Resurrection of vitamin D and rickets. J Clin Invest. 2006;116:2062-2072. Available
at: http://www.jci.org/cgi/content/full/116/8/2062
Kleinman RE, ed. Breastfeeding. In: Pediatric Nutrition Handbook. 5th ed. Elk Grove Village, Ill:
American Academy of Pediatrics; 2004:55-85
Peng LF, Serwint JR. A comparison of breastfed children with nutritional rickets who present
during and after the first year of life. Clin Pediatr. 2003;42:711-717. Abstract available at:
http://www.ncbi.nlm.nih.gov/entrez/query.fcgi?db=pubmed&cmd=Retrieve&dopt=AbstractPlus&li
st_uids=14601920
Critique: 2
Courtesy of R. Schwartz
Critique: 2
Courtesy of D. Mulvihill
Critique: 2
Courtesy of D. Mulvihill
Question: 3
You are evaluating an 8-week-old infant whose birthweight was 1,000 g and who was delivered
at 30 weeks’ gestation. He experienced early respiratory distress and sepsis, but now these
problems have resolved, and he recently progressed from parenteral nutrition to full enteral
feedings.
Of the following, the feeding that will provide the BEST mineral content to ensure healthy bone
development for this infant is
B. human milk
C. premature formula
References:
De Schepper J, Cools F, Vandenplas Y, Louis O. Whole body bone mineral content is similar at
discharge from the hospital in premature infants receiving fortified breast milk or preterm
formula. J Pediatr Gastroenterol Nutr. 2005;41:230-234. Abstract available at:
http://www.ncbi.nlm.nih.gov/entrez/query.fcgi?db=pubmed&cmd=Retrieve&dopt=AbstractPlus&li
st_uids=16056105
Demarini S. Calcium and phosphorous nutrition in preterm infants. Acta Paediatr Suppl.
2005;94:87-92. Abstract available at:
http://www.ncbi.nlm.nih.gov/entrez/query.fcgi?db=pubmed&cmd=Retrieve&dopt=AbstractPlus&li
st_uids=16214772
Kleinman RE. Assessment of nutritional status. In: Pediatric Nutrition Handbook. 5th ed. Elk
Grove Village, Ill: American Academy of Pediatrics; 2004:407-424
Kleinman RE. Nutritional needs of the preterm infant. In: Pediatric Nutrition Handbook. 5th ed. Elk
Grove Village, Ill: American Academy of Pediatrics; 2004:23-54
Lapillonne A, Salle BL, Glorieux FH, Claris O. Bone mineralization and growth are enhanced in
preterm infants fed an isocaloric, nutrient-enriched preterm formula through term. Am J Clin
Nutr. 2004;80:1595-1603. Abstract available at:
http://www.ncbi.nlm.nih.gov/entrez/query.fcgi?db=pubmed&cmd=Retrieve&dopt=AbstractPlus&li
st_uids=15585774
Rubin LP. Disorders of calcium and phosphorus metabolism. In: Taeusch HW, Ballard RA,
Gleason CA, eds. Avery’s Diseases of the Newborn. 8th ed. Philadelphia, Pa: Elsevier
Saunders; 2005:1346-1365
Stamler RJ. Enteral nutrition for the high-risk neonate. In: Taeusch HW, Ballard RA, Gleason CA,
eds. Avery’s Diseases of the Newborn. 8th ed. Philadelphia, Pa: Elsevier Saunders; 2005:1043-
1060
Tender JAF. In brief: preterm infant nutrition. Pediatr Rev. 2004;25:328-329. Available at:
http://pedsinreview.aappublications.org/cgi/content/full/25/9/328
Question: 4
You are addressing a group of new mothers regarding infant feeding. One asks you when an
infant can be switched from formula to whole cow milk.
Of the following, you are MOST likely to respond that whole cow milk
B. can be given at 9 months of age if the infant is also taking a wide variety of supplemental
foods
C. may be given as a supplement at any age as long as the infant also receives human milk
D. should be avoided until 12 months of age because its iron content is absorbed poorly
E. should be avoided until 2 years of age because its caloric content is inadequate for optimal
growth
References:
Formula feeding of term infants. In: Kleinman RE, ed. Pediatric Nutrition Handbook. 5th ed. Elk
Grove Village, Ill: American Academy of Pediatrics; 2004:87-97
Heird WC. The feeding of infants and children. In: Behrman RE, Kliegman RM, Jenson HB, eds.
Nelson Textbook of Pediatrics. 17th ed. Philadelphia, Pa: Saunders; 2004:157-166
Question: 5
An 18-month-old girl who has been adopted from Russia presents to your office for an initial
health supervision visit. The mother notes that the girl limps, and she thinks one leg is longer
than the other. There is no history of medical attention for this problem in the adoption
documentation. Review of available medical history and the current physical examination reveal
no signs of illness. The child appears to be otherwise growing and developing normally. On
physical examination, you measure a leg length discrepancy of 2 cm, and the girl has a
“waddling” type limp and difficulty in abducting her right thigh at the hip.
C. Legg-Calvé-Perthes disease
References:
Scherl S. Common lower extremity problems in children. Pediatr Rev. 2004;25:52-62. Available
at: http://pedsinreview.aappublications.org/cgi/content/full/25/2/52
Critique: 5
Asymmetry of the thigh folds in developmental dysplasia of the hip. There is an additional skin fold
on the affected (right) side.
Courtesy of W. Accosti
Critique: 5
In the Ortolani test, the hip is flexed and abducted and the femoral head is lifted anteriorly into the
acetabulum. The maneuver relocates a dislocated hip, resulting in a "clunk" that may be heard
and felt.
Reprinted with permission from Scherl SA. Common lower extremity problems in children. Pediatr
Rev. 2004;25:52-62
Critique: 5
The Barlow test dislocates the hip. The hip is flexed and adducted. Applying a posterior force
(pushing down) produces a "clunk" as the femoral head dislocates posteriorly.
Reprinted with permission from Scherl SA. Common lower extremity problems in children. Pediatr
Rev. 2004;25:52-62
Critique: 5
Ultrasonography of the normal hip (top) reveals that more than 50% of the femoral head is
covered by the acetabulum. The acetabulum is deep, and the alpha angle is 60 degrees or
greater.
Ultrasonography of the hip in developmental dysplasia (bottom) shows that the acetabulum is
shallow, resulting in an alpha angle of 50 degrees or less (48 degrees in this patient).
Courtesy of D. Mulvihill
Critique: 5
Developmental dysplasia of the hip. The right acetabulum is shallow and steep in orientation. The
right hip is dislocated superolaterally. The right ossification center is smaller than the left.
Courtesy of D. Krowchuk
Critique: 5
Legg-Calvé-Perthes disease: Anteroposterior view of the hips demonstrates sclerosis around the
left femoral epiphysis with flattening of the femoral head. The left medial joint space is widened,
and there is demineralization of the left proximal femur compared with the right.
Courtesy of D. Krowchuk
Critique: 5
Slipped capital femoral epiphysis is characterized by an upward anterior movement of the femoral
neck on the capital epiphysis (which becomes displaced posteriorly and inferiorly). In the normal
hip (right), a line drawn along the superior margin of the femoral neck transects a portion of the
ossified epiphysis. This does not occur on the affected side (left).
Courtesy of D. Krowchuk
Question: 6
A 2-week-old infant presents to the emergency department with a 1-day history of decreased
feeding, pallor, diaphoresis, and increasing somnolence. He was born at term, and the delivery
was uncomplicated. On physical examination, his heart rate is 190 beats/min, his respiratory
rate is 80 breaths/min, his blood pressure is 50/30 mm Hg, and his extremities are cool and pale
with poor pulses. You place the infant on a cardiorespiratory monitor and begin your
assessment and management.
Of the following, the MOST appropriate pair of tests to consider in this child is
References:
Friedman AH, Fahey JT. The transition from fetal to neonatal circulation: normal responses and
implications for infants with heart disease. Semin Perinatol. 1993;17:106-121
Lister G. Poor systemic perfusion and circulatory shock. In: Rudolph CD, Rudolph AM, eds.
Rudolph’s Pediatrics. 21st ed. New York, NY: McGraw-Hill Medical Publishing Division; 2003:285-
292
Rudolph AM. The fetal circulation and its adjustments after birth. In: Moller JH, Hoffman JIE, eds.
Pediatric Cardiovascular Medicine. Philadelphia, Pa: Churchill Livingstone; 2000:60-64
Talner NS, McGovern JJ, Carboni MP. Congestive heart failure. In: Moller JH, Hoffman JIE, eds.
Pediatric Cardiovascular Medicine. Philadelphia, Pa: Churchill Livingstone; 2000:817-829
Question: 7
A 7-year-old boy presents for evaluation of attention-deficit/hyperactivity disorder, following the
suggestion of his first-grade teacher. The child’s academic and behavioral function were
described as normal in preschool and kindergarten, but this year he constantly talks out of turn,
does not stay in his chair, and has frequent emotional “melt-downs.” His mother says his
memory is poor and grades are declining. On physical examination, you note very brisk reflexes
in the arms and legs, two beats of clonus at the ankles, and slow and clumsy fine-motor
movements.
B. electroencephalography
C. electromyography
References:
Johnston MV. Neurodegenerative disorders of childhood. In: Behrman RE, Kliegman RM,
Jenson HB, eds. Nelson Textbook of Pediatrics. 17th ed. Philadelphia, Pa: Saunders; 2004:2029-
2034
Kolodny EH, Fattal-Valevski A. Degenerative disorders. In: Maria BL, ed. Current Management
in Child Neurology. 3rd ed. Hamilton, Ontario, Canada: BC Decker; 2005:265-276
Lyons G, Kolodny EH, Pastores GM. Neurology of Hereditary Metabolic Disease of Children.
3rd ed. New York, NY: McGraw-Hill Medical Publishing Division; 2006
Critique: 7
Axial magnetic resonance imaging of the brain showing ventricular enlargement and diffuse
reduction in white matter.
Courtesy of D. Mulvihill
Question: 8
You are urgently called to the newborn nursery to evaluate a 3-day-old term male infant who is
lethargic. The baby was taking formula well for the first 2 days but vomited after his last 2
feedings and has become increasingly difficult to arouse. A review of the record reveals that he
was born at 36 weeks’ gestation to a 30-year-old gravida 2, now para 2 woman. The mother is
Rh-negative and received Rh immune globulin during the pregnancy. Pregnancy, labor, and
delivery were uneventful; Apgar scores were 8 and 9 at 1 and 5 minutes. Family history is
noncontributory. On physical examination, the baby appears well developed, is very jaundiced,
and has hypotonia and tachypnea. He responds with a grimace to sternal rubbing but does not
arouse to voice or to touch. Findings on abdominal examination are normal. You order a series
of laboratory tests, start intravenous fluids, and arrange for transfer to the neonatal intensive
care unit. A laboratory technician subsequently notifies you of some critical laboratory values,
including a platelet count of 35x103/mcL (35x109/L), serum carbon dioxide of 4 mEq/L (4
mmol/L), anion gap of 28 mEq/L (28 mmol/L), serum ammonia of 250 mcmol/L, and total bilirubin
of 20 mg/dL (342 mcmol/L). There are large ketones in the urine.
A. bilirubin encephalopathy
B. citrullinemia
C. hypoxic-ischemic encephalopathy
D. propionic acidemia
References:
Greene CL, Goodman SI. Catastrophic metabolic encephalopathies in the newborn period.
Evaluation and management. Clin Perinatol. 1997;24:773-786. Abstract available at:
http://www.ncbi.nlm.nih.gov/entrez/query.fcgi?db=pubmed&cmd=Retrieve&dopt=AbstractPlus&li
st_uids=9395862
Nyhan WL, Barshop BA, Ozand PT. Carbamyl phosphate synthetase deficiency. In: Atlas of
Metabolic Diseases. 2nd ed. New York, NY: Oxford University Press; 2005:210-215
Nyhan WL, Barshop BA, Ozand PT. Propionic acidemia. In: Atlas of Metabolic Diseases. 2nd ed.
New York, NY: Oxford University Press; 2005:8-17
Volpe JJ. Hypoxic-ischemic encephalopathy: clinical aspects. In: Neurology of the Newborn. 3rd
ed. Philadelphia, Pa: WB Saunders Co; 1995:314-318
Wong RJ, Stevenson, DK, Ahlfors, CE, Vreman, HE. Neonatal Jaundice. NeoReviews. 2007;
8:e58
Question: 9
A 15-year-old girl comes to your office with the complaint of a vaginal discharge. She states that
she is sexually active with a new partner and has noted a cream-colored, thin discharge. She
also has had some burning with urination and vaginal itching. On pelvic examination, you note
reddened labia majora and minora; a frothy, foul-smelling discharge in the vagina (Item Q9A);
and a cervix that has small erosions and petechiae. Her pregnancy test results are negative,
and microscopic evaluation of vaginal secretions shows motile organisms (Item Q9B).
Question: 9
References:
Centers for Disease Control and Prevention. Sexually transmitted diseases treatment guidelines,
2006. MMWR Recomm Rep. 2006;55(RR-11):1-94. Available at:
http://www.cdc.gov/mmwr/preview/mmwrhtml/rr5511a1.htm
Haward M, Shafer M-A. Vaginitis and cervicitis. In: Neinstein L, ed. Adolescent Health Care: A
Practical Guide. 4th ed. Philadelphia, Pa: Lippincott Williams & Wilkins; 2002:1011-1028
Critique: 9
Vaginal discharge caused by infection with Trichomonas vaginalis often appears frothy.
Question: 10
During the health supervision visit of a 5-year-old girl, you notice pubic hair (Sexual Maturity
Rating 3). Her height is at the 75th percentile and weight is at the 95th percentile. She has no
acne or clitoromegaly. Her mother tells you the girl developed an adult body odor around 8
months ago, and the mother noticed the pubic hair about 6 months ago. She adds that the pubic
hair is a little more noticeable now than when she first saw it.
C. measurement of 17-hydroxyprogesterone
D. measurement of testosterone
References:
Rosenfield RL, Qin K. Premature adrenarche. UpToDate Online 14.3. Available for subscription
at:
http://www.utdol.com/utd/content/topic.do?topicKey=pediendo/12377&type=P&selectedTitle=5~8
Question: 11
The parents of a 15-year-old boy previously diagnosed as having constitutional delay of growth
and puberty are concerned that their son is being bullied by his 10th-grade classmates. His
grades have declined from As to Bs, and he says he dislikes school. He has been in good
health, and he eats and exercises in moderation. On physical examination, his height and weight
are at the 10th percentile, and his genitalia are at Sexual Maturity Rating 1. When you speak to
the boy in private, he becomes tearful, complaining that he is afraid to change for gym and that
his friends no longer wish to be with him. He and his parents ask for your advice in this situation.
A. offer reassurance and arrange for follow-up evaluation in your office in 6 months
E. refer the boy to an endocrinologist for re-evaluation and possible hormonal therapy
References:
Garibaldi L. Physiology of puberty. In: Behrman RE, Kliegman RM, Jenson HB, eds. Nelson
Textbook of Pediatrics. 17th ed. Philadelphia, Pa: Saunders; 2004:1862
McKeever MO, Cheng TL, Root A. In brief: delayed puberty. Pediatr Rev. 2000;21:250-252.
Avalable at: http://pedsinreview.aappublications.org/cgi/content/full/21/7/250
Rosen DS, Foster C. Delayed puberty. Pediatr Rev. 2001;22:309-315. Available at:
http://pedsinreview.aappublications.org/cgi/content/full/22/9/309
Question: 12
A 16-year-old girl presents to the clinic with a 6-day history of low-grade fever and cough. On
physical examination, she has a temperature of 100.6°F (38.1°C) and widespread crackles
throughout her lung fields. You believe she has a “walking pneumonia” caused by Mycoplasma
pneumoniae.
Of the following, the MOST accurate method used to establish the diagnosis is
D. sputum culture
References:
American Academy of Pediatrics. Mycoplasma pneumoniae infections. In: Pickering LK, Baker
CJ, Long SS, McMillian JA, eds. Red Book: 2006 Report of the Committee on Infectious
Diseases. 27th ed. Elk Grove Village, Ill: American Academy of Pediatrics; 2006:468-470
Question: 13
You are evaluating a 12-year-old girl who has a 1-month history of daily fevers (up to 104ºF
[40°C]), cervical adenopathy, severe malaise, headache, and lower back pain. She lives at
home with her parents and two sisters, all of whom have been well. She has a 5-year-old cat
and two birds for pets. Six months ago, she spent 2 weeks visiting relatives who live on a ranch
in Mexico where she learned to milk the cows, feed the pigs and chickens, and ride horses. She
also sampled the local cuisine. Physical examination reveals a febrile, tired-appearing girl who is
having rigors. She has diffuse 1 x 1-cm nontender cervical adenopathy, with splenomegaly and
tenderness to palpation of her lower back. Her white blood cell count is 4.9x103/mcL (4.9x109/L)
with 31% polymorphonuclear leukocytes, 15% band forms, 48% lymphocytes, and 6%
monocytes; erythrocyte sedimentation rate is 70 mm/hr; and C-reactive protein concentration is
6.8 mg/dL.
A. brucellosis
B. cat-scratch disease
D. leptospirosis
E. toxocariasis
References:
Goldstein EJC. Household pets and human infections. Infect Dis Clin North Am. 1991;5:117-130.
Abstract available at:
http://www.ncbi.nlm.nih.gov/entrez/query.fcgi?db=pubmed&cmd=Retrieve&dopt=AbstractPlus&li
st_uids=2051011
Weinberg AN. Ecology and epidemiology of zoonotic pathogens. Infect Dis Clin North Am.
1991;5:1-6. Abstract available at:
http://www.ncbi.nlm.nih.gov/entrez/query.fcgi?db=pubmed&cmd=Retrieve&dopt=AbstractPlus&li
st_uids=2051009
Weinberg AN. Zoonoses. In: Mandell GL, Bennett JE, Dolin R, eds. Mandell, Douglas, and
Bennett’s Principles and Practice of Infectious Diseases. 6th ed. Philadelphia, Pa: Elsevier
Churchill Livingstone; 2005:3630-3636
Critique: 13
Question: 14
A male infant who is experiencing failure to thrive and hypernatremic dehydration is admitted to
the hospital. After administration of intravenous fluids, euvolemia is restored and the serum
sodium is normalized to 140 mEq/L (140 mmol/L). A water deprivation test results in a 5% loss of
body weight over 4 hours and an increase in serum sodium concentration to 145 mEq/L (145
mmol/L) and serum osmolality to 310 mOsm/kg (310 mmol/kg). Simultaneous urine osmolality is
50 mOsm/kg (50 mmol/kg). Subcutaneous administration of desmopressin does not reduce
urine output or increase urine osmolality. You decide to place the infant on a formula that
possesses a low renal solute load.
Of the following, the statement that BEST describes the properties of infant formulas as they
relate to renal solute load is that
A. human milk has a greater renal solute load than do cow milk-based formulas
B. the carbohydrate composition of the formula increases the renal solute load
C. the fat composition of infant formulas more greatly alters the renal solute load when the
medium-chain triglycerides component is increased
D. the primary minerals involved in renal solute load are sodium, potassium, chloride, and
phosphorus
E. the protein composition of the formula has no effect on renal solute load
References:
Sasaki S, Ishibashi K, Marumo F. Aquaporin-2 and -3: representatives of two subgroups of the
aquaporin family colocalized in the kidney collecting duct. Annu Rev Physiol. 1998;60:199-220.
Abstract available at:
http://www.ncbi.nlm.nih.gov/entrez/query.fcgi?db=pubmed&cmd=Retrieve&dopt=AbstractPlus&li
st_uids=9558461
Question: 15
A 10-year-old African-American boy presents to your office complaining of a 12-month history of
stomach pain, nausea, bloating, and diarrhea that occurs 45 to 60 minutes after eating dairy
foods. He states that his symptoms occur only when he eats “too much.” He denies emesis,
hematochezia, or pruritus associated with these episodes. On physical examination, the boy
appears healthy and has normal vital signs. His abdomen is soft and has normal bowel sounds,
and results of a stool guaiac test are negative.
Of the following, the MOST likely cause for this boy’s symptoms is
B. lactose intolerance
References:
Assa’ad AH. Gastrointestinal food allergy and intolerance. Pediatr Ann. 2006;35:718-726.
Abstract available at:
http://www.ncbi.nlm.nih.gov/entrez/query.fcgi?db=pubmed&cmd=Retrieve&dopt=AbstractPlus&li
st_uids=17048714
Fiocchi A, Restani P, Leo G, et al. Clinical tolerance to lactose in children with cow’s milk allergy.
Pediatrics. 2003;112:359-362. Available at:
http://pediatrics.aappublications.org/cgi/content/full/112/2/359
Sampson HA, Leung DYM. Adverse reactions to foods. In: Behrman RE, Kliegman RM, Jenson
HB, eds. Nelson Textbook of Pediatrics. 17th ed. Philadelphia, Pa: Saunders; 2004:789-792
Question: 16
A 2-year-old boy is brought to the emergency department after his father found the boy in the
garage gagging, coughing, and drooling profusely. His parents report that he had opened a
number of containers, but they are unable to recall the names of the products. On physical
examination, the boy is awake and crying, his heart rate is 160 beats/min, his respiratory rate is
24 breaths/min, he has clear lung sounds, and his blood pressure is 100/60 mm Hg. He has
reactive, mid-sized pupils and white eschars on his tongue and soft palate. Shortly after his
examination, he begins to vomit.
A. antifreeze
B. drain cleaner
C. gasoline
E. organophosphate insecticide
References:
Bird S. Organophosphate and carbamate toxicity. UpToDate. Online 14.3. Available at:
http://www.utdol.com/utd/content/topic.do?topicKey=ad_tox/9425&type=A&selectedTitle=1~3
Cordero B, Savage RR, Cheng T. In brief: corrosive ingestions. Pediatr Rev. 2006;27:154-155.
Available at: http://pedsinreview.aappublications.org/cgi/content/full/27/4/154
Ferry GD. Caustic esophageal injury in children. UpToDate. Online 14.3. Available at:
http://www.utdol.com/utd/content/topic.do?topicKey=pedigast/11441&type=P&selectedTitle=3~6
Sivilotti MLA, Winchester JF. Methanol and ethylene glycol intoxication. UpToDate. Online 14.3.
Available at:
http://www.utdol.com/utd/content/topic.do?topicKey=ad_tox/8204&type=P&selectedTitle=5~7
Critique: 16
Calcium oxalate crystalluria: Microscopy of uncentrifuged urine under polarized light reveals
numerous clumps of calcium oxalate crystals.
Courtesy of K. Waibel
Question: 17
You are evaluating a 4-year-old girl who has a 2-day history of perineal pruritus and dysuria.
There is no history of trauma or sexual abuse. Physical examination reveals Sexual Maturity
Rating 1 genitalia, with erythema of the labia majora, labia minora, and vaginal introitus. The
hymenal tissue appears normal, and there is no vaginal discharge.
References:
Emans SJ. Vulvovaginal problems in the prepubertal child. In: Emans SJ, Laufer MR, Goldstein
DP, eds. Pediatric and Adolescent Gynecology. 5th ed. Philadelphia, Pa: Lippincott Williams &
Wilkins; 2005:83-119
Sanfilippo JS. Vulvovaginitis. In: Behrman RE, Kliegman RM, Jenson HB, eds. Nelson Textbook
of Pediatrics. 17th ed. Philadelphia, Pa: Saunders; 2004:1828-1832
Question: 18
You are examining a 12-week-old infant who was born at 28 weeks’ gestation, developed
necrotizing enterocolitis 7 days after birth, and underwent a bowel resection of 20 cm of ileum.
The baby has been dependent on parenteral nutrition since shortly after birth and has
cholestasis due to the parenteral nutrition. Attempts to feed her with a cow milk-based formula
have resulted in diarrhea. Stool analysis demonstrates 0.25% (trace) reducing substances and
3+ fecal fat.
Of the following, the intervention that is MOST likely to decrease the infant’s diarrhea is
References:
Berseth CL. Feeding methods for the preterm infant. Semin Neonatol. 2001;6:417-424. Abstract
available at:
http://www.ncbi.nlm.nih.gov/entrez/query.fcgi?db=pubmed&cmd=Retrieve&dopt=AbstractPlus&li
st_uids=11988031
Kleinman RE, ed. Infant nutrition and the development of gastrointestinal function. In: Pediatric
Nutrition Handbook. 5th ed. Elk Grove Village, Ill: American Academy of Pediatrics; 2004:3-14
Lin PW, Stoll BJ. Necrotising enterocolitis. Lancet. 2006;368:1271-1283. Abstract available at:
http://www.ncbi.nlm.nih.gov/entrez/query.fcgi?db=pubmed&cmd=Retrieve&dopt=AbstractPlus&li
st_uids=17027734
Question: 19
A term infant is delivered by emergency cesarean section following the acute onset of maternal
vaginal bleeding and profound fetal bradycardia. The Apgar scores are 1, 2, and 3 at 1, 5, and
10 minutes, respectively. Resuscitation includes intubation and assisted ventilation, chest
compressions, and intravenous epinephrine. The infant is admitted to the neonatal intensive care
unit and has seizures 6 hours after birth.
Of the following, a TRUE statement about other organ-system injury that may occur in the infant
is that
References:
Care of the neonate. In: Guidelines for Perinatal Care. 5th ed. Elk Grove Village, Ill; Washington
DC: American Academy of Pediatrics; The American College of Obstetricians and
Gynecologists; 2002:187-236
Hankins GD, Koen S, Gei AF, Lopez SM, Van Hook JW, Anderson GD. Neonatal organ system
injury in acute birth asphyxia sufficient to result in neonatal encephalopathy. Obstet Gynecol.
2002;99:688-691. Abstract available at:
http://www.ncbi.nlm.nih.gov/entrez/query.fcgi?db=pubmed&cmd=Retrieve&dopt=AbstractPlus&li
st_uids=11978273
Niermeyer S, Clarke SB. Delivery room care. In: Merenstein GB, Gardner SL, eds. Handbook of
Neonatal Intensive Care. 6th ed. St.Louis, Mo: Mosby Elsevier; 2006:54-78
Talati AJ, Yang W, Yolton K, Korones SB, Bada HS. Combination of early perinatal factors to
identify near-term and term neonates for neuroprotection. J Perinatol. 2005;25:245-250. Abstract
available at:
http://www.ncbi.nlm.nih.gov/entrez/query.fcgi?db=pubmed&cmd=Retrieve&dopt=AbstractPlus&li
st_uids=15703778
Question: 20
A medical student rotating in your clinic tells you about a 5-month-old infant he has evaluated. He
reports that the infant is fed goat milk exclusively and asks you if this is adequate nutrition at this
age.
A. folate
B. iron
C. niacin
D. vitamin A
E. vitamin D
References:
Formula feeding of term infants. In: Kleinman RE, ed. Pediatric Nutrition Handbook. 5th ed. Elk
Grove Village, Ill: American Academy of Pediatrics; 2004:87-97
Glader B. Megaloblastic anemias. In: Behrman RE, Kliegman RM, Jenson HB, eds. Nelson
Textbook of Pediatrics. 17th ed. Philadelphia, Pa: Saunders; 2004:1611-1613
Question: 21
You are seeing a 4-year-old boy for a health supervision visit prior to enrollment in preschool.
His mother expresses some concerns about his speech articulation, although he uses many
words. Results of his physical examination, including pneumatic otoscopy, are normal, as are
results of tympanometry. Routine screening audiometry in your office, despite a cooperative
child and a quiet examination room, yields equivocal results.
Of the following, the BEST next step in the evaluation of this child’s hearing is
C. referral to an audiologist
D. referral to an otolaryngologist
References:
Daly KA, Hunter LL, Giebink GS. Chronic otitis media with effusion. Pediatr Rev. 1999;20:85-94.
Available at: http://pedsinreview.aappublications.org/cgi/content/full/20/3/85
Gregg RB, Wiorek LS, Arvedson JC. Pediatric audiology: a review. Pediatr Rev. 2004;25:224-
234. Available at: http://pedsinreview.aappublications.org/cgi/content/full/25/7/224
Question: 22
Yesterday, you received a call from the newborn nursery that they were referring to you a term
infant who was being discharged at 4 days of age. The female newborn’s birthweight was 3.3 kg
and the delivery was by repeat cesarean section. Findings on physical examination at
discharge, including heart rate, respiratory rate, and blood pressure, were normal. Her lungs
were clear, and no murmurs were noted. She was breastfeeding without difficulty. Today, her
mother calls to tell you that she is difficult to awaken, pale, and breathing much more rapidly than
she was in the hospital nursery. She has had one wet diaper in the last 12 hours. When you
meet them in the emergency department, you note that the infant has cool extremities, weak
pulses, and lethargy.
A. aortic coarctation
C. tetralogy of Fallot
References:
Dreyer WJ, Fisher DJ. Clinical recognition and management of chronic congestive cardiac
failure. In: Garson A Jr, Bricker JT, Fisher DJ, Neish SR, eds. The Science and Practice of
Pediatric Cardiology. 2nd ed. Baltimore, Md: Williams & Wilkins, 1998:2309-2325
Lister G. Poor systemic perfusion and circulatory shock. In: Rudolph CD, Rudolph AM, eds.
Rudolph’s Pediatrics. 21st ed. New York, NY: McGraw-Hill Medical Publishing Division; 2003:285-
292
Talner NS, McGovern JJ, Carboni MP. Congestive heart failure. In: Moller JH, Hoffman JIE, eds.
Pediatric Cardiovascular Medicine. Philadelphia, Pa: Churchill Livingstone; 2000:817-829
Critique: 22
Courtesy of A. Cook
Critique: 22
An atrioventricular canal defect results in shunting of blood from the systemic to pulmonary
system. If the shunt volume is sufficient, congestive heart failure results.
Critique: 22
In tetralogy of Fallot, cyanosis is caused by right-to-left shunting at the ventricular septal defect
as pulmonary stenosis worsens.
Courtesy of A. Johnson
Critique: 22
In transposition of the great arteries, the left ventricle connects to the pulmonary artery and the
right ventricle to the aorta.
Courtesy of A. Johnson
Question: 23
A 14-year-old girl who has a 1-year history of migraine headaches presents to the emergency
department with a severe headache that she calls “the worst headache of my life.” The
headache occurred suddenly after she lifted a heavy box. Her mother says that the girl has
been holding her head stiffly. On physical examination, she appears in severe pain and has
meningismus. Other findings on the physical examination are normal.
D. lumbar puncture
References:
Huang J, Gailloud PH, Tamargo RJ. Vascular malformations. In: Singer HS, Kossoff EH,
Hartman AL, Crawford TO, eds. Treatment of Pediatric Neurological Disorders. Boca Raton, Fla:
Taylor & Francis Group; 2005:409-414
Johnston MV. Acute stroke syndromes. In: Behrman RE, Kliegman RM, Jenson HB, eds.
Nelson Textbook of Pediatrics. 17th ed. Philadelphia, Pa: Saunders; 2004:2035-2037
Pradilla G, Lesniak MS, Tamargo RJ. Pediatric intracranial aneurysms. In: Maria BL, ed. Current
Management in Child Neurology. 3rd ed. Hamilton, Ontario, Canada: BC Decker; 2005:606-612
Question: 24
You care for a newborn who has Down syndrome due to an unbalanced 14;21 translocation. At
the request of the cytogenetics laboratory director, you arranged for the parents’ blood to be
collected and karyotyped. You now are notified that the baby’s mother has an unusual karyotype
with a balanced 14;21 translocation. You plan to refer her for genetic counseling.
Of the following, the recurrence risk for having a baby who has Down syndrome in a future
pregnancy for this woman is CLOSEST to
B. 15%
C. 30%
D. 50%
E. 100%
References:
Gardner RJM, Sutherland GR. Robertsonian translocations. In: Chromosome Abnormalities and
Genetic Counseling. 3rd ed. New York, NY: Oxford University Press; 2004:122-137
Nussbaum RL, McInnes RR, Willard HF. Clinical cytogenetics: disorders of the autosomes and
the sex chromosomes. In: Thompson and Thompson Genetics in Medicine. 6th ed. Philadelphia,
Pa: Saunders; 2004:157-180
Question: 25
A 15-year-old girl comes to your office because she never has had a menstrual period. She has
no chronic illnesses and is active playing softball once a week. Her mother and sister both had
menarche at age 13 years. On physical examination, she is at the 15th percentile for height and
weight and has no hirsutism or acne, no breast development, and Sexual Maturity Rating 3 pubic
hair development.
References:
Emans SJ. Amenorrhea in the adolescent. In: Emans SJ, Laufer MR, Goldstein DP, eds.
Pediatric and Adolescent Gynecology. 5th ed. Philadelphia, Pa: Lippincott, Williams & Wilkins;
2005:214-269
Emans SJ. Delayed puberty. In: Emans SJ, Laufer MR, Goldstein DP, eds. Pediatric and
Adolescent Gynecology. 5th ed. Philadelphia, Pa: Lippincott, Williams & Wilkins; 2005:181-213
Frias JL, Davenport ML, Committee on Genetics and Section on Endocrinology. Clinical report:
health supervision for children with Turner syndrome. Pediatrics. 2003;111:692-702. Available
at: http://pediatrics.aappublications.org/cgi/content/full/111/3/692
Gordon CM, Neinstein LS. Amenorrhea. In: Neinstein LS, ed. Adolescent Health Care: A
Practical Guide. 4th ed. Philadelphia, Pa: Lippincott Williams & Wilkins; 2002:973-993
Critique: 25
A girl who has Turner syndrome exhibits wide-spaced nipples, broad chest, and lack of
secondary sexual development.
Courtesy of M. Rimsza
Question: 26
You are seeing a 10-year-old girl for her yearly health supervision visit. On physical
examination, you palpate a smooth and symmetric thyroid that seems twice normal size (Item
Q26). There are no palpable nodules. Serum free thyroxine and thyroid-stimulating hormone
(TSH) values are both normal. Serum thyroperoxidase antibody concentrations are elevated.
References:
Heptulla RA. Thyroid gland. In: McMillan JA, Feigin RD, DeAngelis C, Jones MD Jr, eds. Oski’s
Pediatrics: Principles & Practice. 4th ed. Philadelphia, Pa: Lippincott Williams & Wilkins; 2006:
2123-2132
Svensson J, Ericsson U-B, Nilsson P, et al. Levothyroxine treatment reduces thyroid size in
children and adolescents with chronic autoimmune thyroiditis. J Clin Endocrinol Metab.
2006;91:1729-1734. Abstract available at:
http://www.ncbi.nlm.nih.gov/entrez/query.fcgi?db=pubmed&cmd=Retrieve&dopt=AbstractPlus&li
st_uids=16507633
Question: 27
A 7-year-old girl is being treated with a gonadotropin-releasing hormone agonist by an
endocrinologist for precocious puberty. On physical examination, she is at Sexual Maturity
Rating 3. Her parents are pleased with the medical treatment, but are concerned that their
daughter is having problems fitting in with the 7-year-old girls with whom she used to play, who
are teasing her. She is now gravitating toward playing with older children in her neighborhood.
The parents ask for guidance regarding their daughter’s behavior.
A. explain that having older friends is beneficial to her self-esteem and should be encouraged
B. explain that their daughter should be treated more maturely because her body is maturing at
a faster rate
C. recommend psychological counseling to help the girl deal with her feelings regarding the
changes in her body
D. suggest that her parents call the girl’s playmates and tell them to stop teasing her
E. suggest that the parents get a pet for their daughter so she will spend less time with the
neighborhood children
References:
Garibaldi L. Disorders of pubertal development. In: Behrman RE, Kliegman RM, Jenson HB, eds.
Nelson Textbook of Pediatrics. 17th ed. Philadelphia, Pa: Saunders; 2004:1863-1869
Question: 28
A 6-year-old boy presents to the clinic with a 2-day history of fever and noisy breathing. His
mother is concerned because she feels that her son is “going to die.” His past medical history is
unremarkable, but he has not yet received his fifth diphtheria-tetanus-acellular pertussis (DTaP)
or his second measles-mumps-rubella (MMR) vaccination. On physical examination, the boy
appears scared and toxic and has labored respirations and a very harsh cough. He is not
drooling and can lie flat while you examine him. His temperature is 103.5°F (39.7°C), respiratory
rate is 35 breaths/min, heart rate is 168 beats/min, and blood pressure is 107/68 mm Hg.
Although he has tachypnea, his lungs are clear to auscultation, he has no heart murmur, and
findings on his abdominal examination are benign.
A. bacterial tracheitis
B. bronchitis
C. epiglottitis
E. laryngotracheobronchitis
References:
Pope J, McBride J. Consultation with the specialist: respiratory failure in children. Pediatr Rev.
2004;25:160-167. Available at: http://pedsinreview.aappublications.org/cgi/content/full/25/5/160
Roosevelt GE. Acute inflammatory upper airway obstruction. In: Behrman RE, Kliegman RM,
Jenson HB, eds. Nelson Textbook of Pediatrics. 17th ed. Philadelphia, Pa: Saunders; 2004:1405-
1409
Critique: 28
Bronchoscopic image in a patient who has bacterial tracheitis reveals thick secretions and
pseudomembrane formation.
Question: 29
You are speaking to the pediatric ward nurses regarding prevention of the nosocomial spread of
rotavirus disease after three patients developed the disease while hospitalized. You stress the
importance of good handwashing practices.
D. need to put on gowns, gloves, and mask before entering patient rooms
E. need to put on gowns, gloves, mask, and shoe covers before entering patient rooms
References:
American Academy of Pediatrics. Children in residential institutions. In: Pickering LK, Baker CJ,
Long SS, McMillan JA, eds. Red Book: 2006 Report of the Committee on Infectious Diseases.
27th ed. Elk Grove Village, Ill: American Academy of Pediatrics; 2006:90-92
American Academy of Pediatrics. Isolation precautions. In: Pickering LK, Baker CJ, Long SS,
McMillan JA, eds. Red Book: 2006 Report of the Committee on Infectious Diseases. 27th ed. Elk
Grove Village, Ill: American Academy of Pediatrics; 2006:154-160
Edmond M. Isolation. Infect Control Hosp Epidemiol. 1997;18:58-64. Abstract available at:
http://www.ncbi.nlm.nih.gov/entrez/query.fcgi?db=pubmed&cmd=Retrieve&adopt=AbstractPlus&l
ist_uids=9013248
Garner JS. The Hospital Infection Control Practices Advisory Committee. Guidelines for isolation
precautions in hospitals. Infect Control Hosp Epidemiol. 1996;17:53-80
Critique: 29
Question: 30
You are asked to evaluate a 3,500-g term infant who was found to have unilateral renal agenesis
on prenatal ultrasonography. Laboratory tests reveal a sodium concentration of 140 mEq/L (140
mmol/L), potassium of 4.1 mEq/L (4.1 mmol/L), chloride of 110 mEq/L (110 mmol/L), and
bicarbonate of 19 mEq/L (19 mmol/L).
References:
Arant BS Jr. Postnatal development of renal function during the first year of life. Pediatr Nephrol.
1987;1:308-313. Abstract available at:
http://www.ncbi.nlm.nih.gov/entrez/query.fcgi?db=pubmed&cmd=Retrieve&dopt=AbstractPlus&li
st_uids=3153294
Rose BD, Post TW. Regulation of acid-base balance. In: Clinical Physiology of Acid-base and
Electrolyte Disorders. 5th ed. New York, NY: McGraw-Hill Medical Publishing Division; 2001:325-
371
Schwaderer AL, Schwartz G J. Back to basics: acidosis and alkalosis. Pediatr Rev.
2004;25:350-357. Available at: http://pedsinreview.aappublications.org/cgi/content/full/25/10/350
Question: 31
A 9-year-old boy who has a history of asthma and allergic rhinitis presents with coughing,
wheezing, and chest tightness of 4 days’ duration. He only has 1 week of school left before
summer break, but his parents have kept him home because they are administering a beta-2
agonist inhaler every 4 hours to control his symptoms. They mention that “everyone was sick”
in his class, and he developed clear rhinorrhea and a temperature of 99.0°F (37.3°C) at the start
of his current illness.
A. coronavirus
B. influenza virus
C. parainfluenza virus
E. rhinovirus
References:
Dakhama A, Lee YM, Gelfand EW. Virus-induced airway dysfunction: pathogenesis and
biomechanisms. Pediatr Infect Dis J. 2005;24(suppl):S159-69. Abstract available at:
http://www.ncbi.nlm.nih.gov/sites/entrez?db=pubmed&cmd=Retrieve&dopt=AbstractPlus&list_uid
s=16378041
Heymann PW, Platts-Mills T, Johnston SL. Role of viral infections, atopy and antiviral immunity in
the etiology of wheezing exacerbations among children and young adults. Pediatr Infect Dis J.
2005;24(suppl):S217-S222. Abstract available at:
http://www.ncbi.nlm.nih.gov/entrez/query.fcgi?db=pubmed&cmd=Retrieve&dopt=AbstractPlus&li
st_uids=16378049
Liu AH, Spahn JD, Leung DYM. Childhood asthma. In: Behrman RE, Kliegman RM, Jenson HB,
eds. Nelson Textbook of Pediatrics. 17th ed. Philadelphia, Pa: Saunders; 2004:760-774
Question: 32
A 2-year-old boy is brought to the emergency department because of moderate respiratory
distress. History reveals that he was born at 25 weeks’ gestation and had bronchopulmonary
dysplasia. He has had rhinorrhea and cough for the past 2 days, and this morning he developed
retractions and wheezing that were unresponsive to albuterol. His usual medications include
albuterol and oxygen at 1 L/min by nasal cannula to maintain his oxygen saturation at 93%. On
physical examination, his heart rate is 160 beats/min, respiratory rate is 60 breaths/min, and
oxygen saturation is 82%. You place him on 100% oxygen using a nonrebreather mask and
obtain blood gases, which reveal a pH of 7.35, Pco2 of 70 mm Hg, Po2 of 226 mm Hg, and
HCO3 of 35 mEq/L (35 mmol/L). As you are interpreting the blood gases, the nurse notifies you
that the boy has developed apnea.
Of the following, the MOST likely explanation for his sudden deterioration is
References:
Pope J, McBride J. Consultation with the specialist: respiratory failure in children. Pediatr Rev.
2004;25:160-167. Available at: http://pedsinreview.aappublications.org/cgi/content/full/25/5/160
Schwaderer AL, Schwartz GJ. Back to basics: acidosis and alkalosis. Pediatr Rev. 2004;25:350
- 357. Available at: http://pedsinreview.aappublications.org/cgi/content/full/25/10/350
Question: 33
A 4-year-old boy presents with a 3-month history of a recurring pruritic eruption on the
abdomen. The parents report that each outbreak lasts 1 to 2 weeks. No one else in the home is
similarly affected. Physical examination reveals clustered erythematous papules measuring 4 to
6 mm in diameter (Item Q33). A few papules have a central punctum.
A. folliculitis
B. Gianotti-Crosti disease
C. molluscum contagiosum
D. papular urticaria
E. scabies
Question: 33
Courtesy of dermatlas.org
References:
Krowchuk DP, Mancini AJ, eds. Insect bites and papular urticaria. In: Pediatric Dermatology. A
Quick Reference Guide. Elk Grove Village, Ill: American Academy of Pediatrics; 2007:225-231
Paller AS, Mancini AJ. Bites and infestations. In: Hurwitz Clinical Pediatric Dermatology. 3rd ed.
Philadelphia, Pa: Elsevier Saunders; 2006:479-501
Weston WL, Lane AT, Morelli JG. Infestations. In: Color Textbook of Pediatric Dermatology. 3rd
ed. St. Louis, Mo: Mosby; 2002:89:77-88
Critique: 33
Courtesy of dermatlas.org
Critique: 33
Courtesy of D. Krowchuk
Critique: 33
Erythematous papules that involve the face and extremities, with relative sparing of the trunk, are
observed in Gianotti-Crosti disease.
Critique: 33
The lesions of molluscum contagiosum are translucent papules. Larger lesions have a central
umbilication (arrow).
Courtesy of D. Krowchuk
Critique: 33
Courtesy of D. Krowchuk
Question: 34
A 10-week-old infant has undergone abdominal surgery for gastroschisis. After 6 weeks of
parenteral nutrition (PN), cholestasis has developed.
Of the following, the intervention that is MOST likely to reduce the severity of cholestatic liver
disease due to PN is
References:
Goulet O. Chronic diarrhea and intestinal transplantation. In: Walker WA, Watkins JB, Duggan
C, eds. Nutrition in Pediatrics: Basic Science and Clinical Applications. 3rd ed. Hamilton, Ontario,
Canada: BC Decker; 2003:752-770
Gura KM, Duggan CP, Collier SB, et al. Reversal of parenteral nutrition-associated liver disease
in two infants with short bowel syndrome using parenteral fish oil: implications for future
management. Pediatrics. 2006;118:e197-e201. Available at:
http://pediatrics.aappublications.org/cgi/content/full/118/1/e197
Utter SL, Jaksic T, Duggan C. Short-bowel syndrome. In: Hansen AR, Puder M, eds. Manual of
Neonatal Surgical Intensive Care. Hamilton, Ontario, Canada: BC Decker; 2003:284-302
Question: 35
You are evaluating a 3-day-old preterm infant who was born at 26 weeks’ gestation and weighed
800 g. Her blood pressure has dropped acutely, and she has developed seizures. Physical
examination demonstrates equal mechanical breath sounds, no heart murmur, hypotonia, a
bulging anterior fontanelle, and lethargy. Laboratory evaluation reveals anemia, metabolic
acidemia, and hyperglycemia.
A. acute pneumothorax
B. intracranial hemorrhage
C. late-onset sepsis
E. perinatal asphyxia
References:
Linder N, Haskin O, Levit O, et al. Risk factors for intraventricular hemorrhage in very low birth
weight premature infants: a retrospective case-control study. Pediatrics. 2003;111:e590-e595.
Available at: http://pediatrics.aappublications.org/cgi/content/full/111/5/e590
Ment LR. Intraventricular hemorrhage of the preterm infant. In: McMillan JA, Feigin
RD, DeAngelis C, Jones MD Jr, eds. Oski's Pediatrics: Principles & Practice. 4th
ed. Philadelphia, Pa: Lippincott Williams & Wilkins: 2006:271-276
Mercer JS, Vohr BR, McGrath MM, Padbury JF, Wallach M, Oh W. Delayed cord clamping in
very preterm infants reduces the incidence of intraventricular hemorrhage and late-onset
sepsis: a randomized, controlled trial. Pediatrics. 2006;117:1235-1242. Available at:
http://pediatrics.aappublications.org/cgi/content/abstract/117/4/1235
Paige PL, Moe PC. Neurologic disorders. In: Merenstein GB, Gardner SL, eds. Handbook of
Neonatal Intensive Care. 6th ed. St.Louis, Mo: Mosby Elsevier; 2006:773-811
Critique: 35
Courtesy of D. Mulvihill
Critique: 35
Grade III intraventricular hemorrhage: Sagittal ultrasonography of the head shows a dilated lateral
ventricle filled with echogenic material representing blood.
Courtesy of D. Mulvihill
Critique: 35
Courtesy of D. Mulvihill
Question: 36
During the prenatal visit with new parents, a mother expresses concern about regulating the
temperature of the bath water for the new baby. You tell her that standards regarding hot water
heaters have been determined.
Of the following, the temperature that has been determined to be appropriate for hot water
heaters is CLOSEST to
A. 110ºF
B. 120ºF
C. 130ºF
D. 140ºF
E. 150ºF
Water TemperatureTime
150ºF2 seconds
140ºF6 seconds
130ºF30 seconds
120ºF5 minutes
Therefore, the Consumer Product Safety Commission has recommended that all water
heaters be set to 120ºF. Parents should call their local electric or gas companies for instructions
on adjusting the temperature. Parents who live in apartments should talk with their building
managers about having the temperature lowered because the hot water heaters in most
apartment buildings are set at higher temperatures to provide hot water to all tenants. In addition
to setting hot water heaters to a lower temperature, parents should hand-test water before
bathing children and infants, and young children never should be left unsupervised in the
bathroom.
References:
Hansbrough JF, Hansbrough W. Pediatric burns. Pediatr Rev. 1999;20:117-124. Available at:
http://pedsinreview.aappublications.org/cgi/content/full/20/4/117
Tap Water Scalds. Document 8065098. Consumer Product Safety Commission. Available at:
http://www.cpsc.gov/cpscpub/pubs/5098.html
Question: 37
A 7-year-old child who has autism presents with purulent, bloody discharge from the ear of 1
day’s duration. His mother is unaware of any antecedent events. The child is unable to provide
any history, but his mother explains that he is attending day camp. Examination of the affected
ear reveals blood-tinged, purulent discharge that prohibits visualization of the tympanic
membrane and apparent tenderness to touch of the pinna.
References:
Hughes E, Lee JH. Otitis externa. Pediatr Rev. 2001;22:191-197. Available at:
http://pedsinreview.aappublications.org/cgi/content/full/22/6/191?
Macfadyen CA, Acuin JM, Gamble C. Systemic antibiotics versus topical treatments for
chronically discharging ears with underlying eardrum perforations. Cochrane Database Syst
Rev. 2006;1:CD005608. Available at:
http://www.mrw.interscience.wiley.com/cochrane/clsysrev/articles/CD005608/frame.html
Macfadyen CA, Acuin JM, Gamble C. Topical antibiotics without steroids for chronically
discharging ears with underlying eardrum perforations. Cochrane Database Syst Rev.
2005;4:CD004618. Available at:
http://www.mrw.interscience.wiley.com/cochrane/clsysrev/articles/CD004618/frame.html
Rosenfeld RM, Brown L, Cannon CR, et al. Clinical practice guideline: acute otitis externa.
Otolaryngol Head Neck Surg. 2006;134(4 suppl):S4-S23. Abstract available at:
http://www.ncbi.nlm.nih.gov/entrez/query.fcgi?db=pubmed&cmd=Retrieve&dopt=AbstractPlus&li
st_uids=16638473
Vaile L, Williamson T, Waddell A, Taylor G. Interventions for ear discharge associated with
grommets (ventilation tubes). Cochrane Database Syst Rev. 2006;2:CD001933. Available at:
http://www.mrw.interscience.wiley.com/cochrane/clsysrev/articles/CD001933/frame.html
Critique: 37
Reprinted with permission from Hughes E, Lee JH. Otitis externa. Pediatr Rev. 2001;22:191-197
Question: 38
A 1-week-old infant presents to the emergency department with a 1-day history of poor feeding,
pallor, diaphoresis, and increasing somnolence. She was born at term, and the delivery was
uncomplicated. On physical examination, her heart rate is 180 beats/min, respiratory rate is 90
breaths/min, and blood pressure is 50/30 mm Hg. Her breath sounds are shallow, and cardiac
evaluation reveals no murmurs but a gallop rhythm. Her liver is palpable at 3 cm below the costal
margin. Her extremities are cool, pale, and mottled, and she has poor distal pulses. After you
administer normal saline at 20 mL/kg, her heart rate is 194 beats/min.
References:
Lister G. Poor systemic perfusion and circulatory shock. In: Rudolph CD, Rudolph AM, eds.
Rudolph’s Pediatrics. 21st ed. New York, NY: McGraw-Hill Medical Publishing Division; 2003:285-
292
Talner NS, McGovern JJ, Carboni MP. Congestive heart failure. In: Moller JH, Hoffman JIE, eds.
Question: 39
A 10-year-old boy presents with leg weakness that has progressed over 24 hours, bladder and
bowel incontinence, and back pain. There is no history of trauma. On physical examination, leg
reflexes are diminished, and there is numbness in the legs and lower trunk. Rectal examination
demonstrates decreased tone. Sensory examination shows absent pinprick sensation below T6.
C. lumbar puncture
References:
Haslam RHA. Spinal cord disorders. In: Behrman RE, Kliegman RM, Jenson HB, eds. Nelson
Textbook of Pediatrics. Philadelphia, Pa: Saunders; 2004:2049-2052
Kerr D, Krishnan C, Pidcock FS. Acute transverse myelitis. In: Singer HS, Kossoff EH, Hartman
AL, Crawford TO, eds. Treatment of Pediatric Neurologic Disorders. Boca Raton, Fla: Taylor &
Francis Group; 2005:445-454
Menkes JH, Ellenbogen RC. Traumatic brain and spinal cord injuries in children. In: Maria BL,
ed. Current Management in Child Neurology. 3rd ed. Hamilton, Ontario, Canada: BC Decker;
2005:515-527
Critique: 39
Courtesy of P. Fisher
Question: 40
A mother who is new to your area brings her 6-year-old daughter to your office for evaluation
due to poor school performance and behavioral concerns. The child is in the first grade and is
struggling with math and reading. The mother says that the girl is “hyper” and does not play well
with other children. Despite almost being hit by a car recently, she repeatedly runs into the road
in front of their house. On physical examination, you note that the child’s height, weight, and
head circumference are less than the 10th percentile. She has short palpebral fissures, a
smooth philtrum, and a thin upper lip (Item Q40).
A. Down syndrome
C. hypochondroplasia
D. Russell-Silver syndrome
E. 47,XXX
Question: 40
Short palpebral fissures and a thin upper lip, as described for the child in the vignette.
Courtesy of M. Rimsza
References:
Astley SJ, Clarren SK. Diagnosing the full spectrum of fetal alcohol-exposed individuals:
introducing the 4-digit diagnostic code. Alcohol Alcohol. 2000;35:400-410. Abstract available at:
http://www.ncbi.nlm.nih.gov/entrez/query.fcgi?db=pubmed&cmd=Retrieve&dopt=AbstractPlus&li
st_uids=10906009
Astley SJ. Comparison of the 4-digit diagnostic code and the Hoyme diagnostic guidelines for
fetal alcohol spectrum disorders. Pediatrics. 2006;118:1532-1545. Available at:
http://pediatrics.aappublications.org/cgi/content/full/118/4/1532
Down syndrome. In: Jones KL. Smith’s Recognizable Patterns of Human Malformation. 6th ed.
Philadelphia, Pa: Elsevier Saunders; 2006:7-13
Hoyme HE, May PA, Kalberg WO, et al. A practical clinical approach to diagnosis of fetal alcohol
Hypochondroplasia. In: Jones KL. Smith’s Recognizable Patterns of Human Malformation. 6th
ed. Philadelphia, Pa: Elsevier Saunders; 2006:398-399
National Center on Birth Defects and Developmental Disabilities, Centers for Disease Control
and Prevention. Fetal Alcohol Syndrome: Guidelines for Referral and Diagnosis. Atlanta, Ga:
Centers for Disease Control and Prevention; 2004. Available at:
http://www.cdc.gov/ncbddd/fas/documents/FAS_guidelines_accessible.pdf
Russell-Silver syndrome. In: Jones KL. Smith’s Recognizable Patterns of Human Malformation.
6th ed. Philadelphia, Pa: Elsevier Saunders; 2006:92-93
Critique: 40
Facial features of fetal alcohol syndrome include epicanthal folds, short palpebral fissures, a
relatively smooth philtrum, and a narrow upper lip.
Courtesy of M. Rimsza
Critique: 40
Children who have Down syndrome have epicanthal folds, upslanting palpebral fissures, and a
flat nasal bridge.
Courtesy of M. Rimsza
Critique: 40
Infants who have hypochondroplasia have findings similar to those who have achondroplasia
(shown here): The head is large, the chest is small when compared with the abdomen, and there
is proximal shortening of the upper extremities.
Courtesy of M. Rimsza
Question: 41
A 16-year-old girl comes to your office with a 2-day history of moderate pelvic pain. She is
sexually active and using no contraception. She has had a scant yellowish-white discharge from
her vagina, but no vaginal itching. On physical examination, she is afebrile, has a heart rate of 95
beats/min and a blood pressure of 110/75 mm Hg. Bowel sounds are of normal pitch and
frequency. She has no rebound or guarding on abdominal evaluation, but has moderate pelvic
tenderness to palpation. She has no costovertebral angle tenderness. Findings on pelvic
examination include scant yellow cervical discharge, a friable inflamed cervix, cervical motion
tenderness, and mild uterine tenderness but no adnexal masses. Results of her pregnancy test
are negative, and findings on urinalysis are normal.
B. endometrial biopsy
C. laparoscopy
References:
Centers for Disease Control and Prevention. Sexually transmitted diseases treatment guidelines,
2006. MMWR Recomm Rep. 2006;55(RR-11):1-94. Available at:
http://www.cdc.gov/mmwr/preview/mmwrhtml/rr5511a1.htm
Question: 42
A 1-year-old boy presents with generalized seizures. His general physical examination findings
are normal except for a prominently positive Chvostek response. Results of laboratory studies
include total serum calcium of 4.5 mg/dL (1.1 mmol/L) and phosphorus of 8.2 mg/dL (2.73
mmol/L). Blood urea nitrogen and creatinine values are normal for age.
B. hypoparathyroidism
C. hyperphosphatasia
References:
Cundy T, Hegde M, Naot D, et al. A mutation in the gene TNFRSF11B encoding osteoprotegerin
causes an idiopathic hyperphosphatasia phenotype. Hum Mol Genet. 2002;11:2119-2127.
Available at: http://hmg.oxfordjournals.org/cgi/content/full/11/18/2119
Jeha G, Kirkland JL. Etiology of hypocalcemia in infants and children. UpToDate Online 14.3.
Available for subscription at:
http://www.utdol.com/utd/content/topic.do?topicKey=pediendo/6894&type=P&selectedTitle=3~6
Nield LS, Mahajan P, Joshi A, Kamat D. Rickets: not a disease of the past. Am Fam Physician.
2006;74:619-626. Available at: http://www.aafp.org/afp/20060815/619.html
Singh J, Moghal N, Pearce SH, Cheetham T. The investigation of hypocalcemia and rickets.
Arch Dis Child. 2003;88:403-407. Available at: http://adc.bmj.com/cgi/content/full/88/5/403
Question: 43
The parents of a 12-year-old boy are concerned about his changing behavior. He was
previously a straight A student, but now he is getting Cs in most of his classes. He no longer
wants to be with his friends after school, and he recently quit the basketball team. On weekends,
he sleeps much of the day, and when awake, he stays alone in his room. Findings on his
physical examination are normal. He appears apathetic and speaks softly in a monotone.
References:
American Academy of Child and Adolescent Psychiatry. Practice parameters for the
assessment and treatment of children with depressive disorders. J Am Acad Child Adolesc
Psychiatry. 1998;37:63S-83S. Abstract available at:
http://www.ncbi.nlm.nih.gov/entrez/query.fcgi?db=pubmed&cmd=Retrieve&adopt=AbsractPlus&li
st_uids=9785729
Hatcher-Kay C, King CA. Depression and suicide. Pediatr Rev. 2003;24:363-371. Available at:
http://pedsinreview.aappublications.org/cgi/content/full/24/11/363
Mood disorders. In: Diagnostic and Statistical Manual of Mental Disorders. 4th ed. Text Revision.
Washington, DC: American Psychiatric Association; 2000:345-428
Pediatric Education & Outreach Program. Vignettes: depression. New York, NY: New York
University Child Study Center. Available at:
http://www.mssmtv.org/conferences/template/live_agendas/agenda_nyu_ptsd_2003.php
Question: 44
You are treating a 2-year-old girl who has suspected meningococcal bacteremia and meningitis.
Over the past 2 hours, she has required multiple fluid boluses and inotropic support to help
maintain her blood pressure. She has been intubated due to respiratory failure. Her temperature
is 96°F (35.6°C), and she is covered in a petechial and purpuric rash (Item Q44). Her most
recent laboratory results reveal a white blood cell count of 1.2x103/mcL (1.2x109/L) with 80%
lymphocytes, 10% neutrophils, and 10% band forms and a platelet count of 32x103/mcL
(32x109/L).
C. measurement of fibrinogen
Question: 44
Courtesy of G. Schutze
References:
Journeycake JM, Buchanan GR. Coagulation disorders. Pediatr Rev. 2003;24:83-91. Available
at: http://pedsinreview.aappublications.org/cgi/content/full/24/3/83
Critique: 44
Courtesy of M. Rimsza
Question: 45
You are hiring a pediatrician as a hospitalist at a community hospital.
Of the following, the disease for which immune status MUST be documented at the time of
employment is
A. diphtheria
B. hepatitis A
C. measles
D. meningococcal disease
E. tetanus
References:
American Academy of Pediatrics. Health care personnel. In: Pickering LK, Baker CJ, Long SS,
McMillan JA, eds. Red Book: 2006 Report of the Committee on Infectious Diseases. 27th ed. Elk
Grove Village, Ill: American Academy of Pediatrics; 2006:94-96
Question: 46
A 6-year-old child presents for a health supervision visit. On physical examination, his weight is
18 kg, height is 102 cm (<3rd percentile),pulse rate is 90 beats/min, respiratory rate is 18
breaths/min, and blood pressure is 134/88 mm Hg. Of note, he has pale conjunctivae and mild
edema. Among the results of laboratory evaluation are:
Of the following, the MOST likely explanation for this patient’s anemia is
B. erythropoietin deficiency
D. hemolysis
E. iron deficiency
References:
Chan JCM, Williams DM, Roth KS. Kidney failure in infants and children. Pediatr Rev. 2002;23:47-
60. Available at: http://pedsinreview.aappublications.org/cgi/content/full/23/2/47
Question: 47
You are working with a medical student in an outpatient pediatric clinic. His first case is a 4-
month-old male infant who has a 4-day history of rhinorrhea, coughing, and tachypnea. On
physical examination, the infant is fussy and has a temperature of 99.2°F (37.4°C), pulse rate of
110 beats/min, respiratory rate of 60 breaths/min, and pulse oximetry of 92% on room air. On
auscultation, expiratory wheezes are audible in all lung fields. You discuss the differential
diagnosis and ask the student what he knows about respiratory syncytial virus (RSV)
bronchiolitis.
Of the following, the MOST accurate statement regarding RSV bronchiolitis is that
A. household pets can be colonized with RSV and may act as reservoirs for transmission to
family members
B. most chest radiographs of infants hospitalized for RSV bronchiolitis appear normal
D. recurrent wheezing may occur for several years after mild or severe RSV bronchiolitis
References:
McCarthy CA, Hall CB. Respiratory syncytial virus: concerns and control. Pediatr Rev.
2003;24:301-309. Available at: http://pedsinreview.aappublications.org/cgi/content/full/24/9/301
McIntosh K. Respiratory syncytial virus. In: Behrman RE, Kliegman RM, Jenson HB, eds.
Nelson Textbook of Pediatrics. 17th ed. Philadelphia, Pa: Saunders; 2004:1076-1078
Question: 48
A 2-year-old girl presents to your office with a 3-week history of a “barky” cough. According to
her mother, the girl has had no fever or upper respiratory tract infection symptoms, but she has
complained intermittently of a sore throat for the past 2 weeks. On physical examination, the
child appears well and playful. Her throat is nonerythematous, and her lungs are clear to
auscultation. Because of the persistence of her symptoms, you obtain chest radiograph (Item
Q48).
C. spontaneous passage of foreign bodies located at the thoracic inlet is likely to occur
D. the majority of patients who have retained foreign bodies are asymptomatic
E. the most likely location for a foreign body to lodge is at the gastroesophageal junction
Question: 48
Courtesy of M. Wright
References:
Skae CC, Adam HM. In brief: esophageal foreign bodies. Pediatr Rev. 2005;26:34-35. Available
at: http://pedsinreview.aappublications.org/cgi/content/full/26/1/34
Waltzman ML. Management of esophageal coins. Curr Opin Pediatr. 2006;18:571-574. Abstract
available at:
http://www.ncbi.nlm.nih.gov/entrez/query.fcgi?db=pubmed&cmd=Retrieve&dopt=AbstractPlus&li
st_uids=16969175
Waltzman ML, Baskin M, Wypij D, Mooney D, Jones D, Fleisher G. A randomized clinical trial of
the management of esophageal coins in children. Pediatrics. 2005;116:614-619. Available at:
http://pediatrics.aappublications.org/cgi/content/full/116/3/614
Critique: 48
A toddler ingested a watch battery that eroded through the esophagus and aorta, resulting in
death.
Courtesy of D. Mulvihill
Critique: 48
Anteroposterior (top) and lateral (bottom) radiographs of the chest demonstrating a coin in the
esophagus.
Courtesy of D. Mulvihill
Question: 49
You are seeing an 8-month-old boy for the first time. His parents are concerned about a birth
mark on his chest that has enlarged slowly over the past 5 to 6 months (Item Q49).
A. interferon-alfa
B. intralesional corticosteroid
C. observation
E. systemic corticosteroid
Question: 49
Courtesy of D. Krowchuk
References:
Krowchuk DP, Mancini AJ, eds. Infantile hemangioma. In: Pediatric Dermatology. A Quick
Reference Guide. Elk Grove Village, Ill: American Academy of Pediatrics; 2007:285-293
Paller AS, Mancini AJ. Vascular disorders of infancy and childhood. In: Hurwitz Clinical Pediatric
Dermatology. 3rd ed. Philadelphia, Pa: Elsevier Saunders; 2006:307-344
Weston WL, Lane AT, Morelli JG. Vascular lesions. In: Color Textbook of Pediatric Dermatology.
3rd ed. St. Louis, Mo: Mosby; 2002:89:187-201
Critique: 49
Courtesy of D. Krowchuk
Critique: 49
A deep hemangioma is a compressible nodule or tumor that is blue and has surface
telangiectasias.
Critique: 49
Courtesy of D. Krowchuk
Critique: 49
The clinical hallmark of hemangioma involution is the appearance of a white or gray surface color.
Courtesy of D. Krowchuk
Critique: 49
Courtesy of D. Krowchuk
Critique: 49
Courtesy of D. Krowchuk
Critique: 49
A proliferating hemangioma near the eye may require oral corticosteroid therapy.
Courtesy of D. Krowchuk
Critique: 49
Courtesy of D. Krowchuk
Question: 50
A mother brings in her 5-year-old son in for a health supervision visit. Family history reveals that
the boy’s father has had a soft-tissue sarcoma and a colectomy for “colon polyps.” Results of
the boy’s physical examination are within normal limits. The mother asks if her son is at
increased risk for polyps and cancer. You review the father’s medical records, which indicate
that his colectomy was performed at age 20 because of the discovery of 50 adenomas in the
colon.
Of the following, the BEST recommendation for the son at this time is
References:
Mougenot JF, Olschwang S, Peuchmaur M. Intestinal polyps and polyposis. In: Walker WA,
Goulet O, Kleinman RE, Sherman P, Shneider BL, Sanderson IR, eds. Pediatric Gastrointestinal
Disease: Pathophysiology, Diagnosis, Management. 4th ed. Hamilton, Ontario, Canada: BC
Decker; 2004:966-988
Critique: 50
Image during colonoscopy of a patient who has familial adenomatous polyposis. Each of the
nodules (arrow) represents an adenoma.
Courtesy of A. Bousvaros
Critique: 50
Patients who have Peutz-Jegher syndrome have lentigines on the lips, oral mucosae, and
elsewhere.
Reprinted with permission from Krowchuk DP, Mancini AJ, eds. Pediatric Dermatology. A Quick
Reference Guide. Elk Grove Village, Ill: American Academy of Pediatrics; 2007
Critique: 50
Question: 51
You are conducting prenatal counseling with a 30-year-old mother of two healthy children. She is
34 weeks pregnant and asks you about potential newborn problems for her soon-to-be delivered
infant, reminding you that her last child was treated for early-onset group B streptococcal (GBS)
infection.
References:
American Academy of Pediatrics. Group B streptococcal infections. In: Pickering LK, Baker CJ,
Long SS, McMillan JA, eds. Red Book: 2006 Report of the Committee on Infectious Diseases.
27th ed. Elk Grove Village, Ill: American Academy of Pediatrics; 2006:620-627
Baker CJ. Group B streptococcal disease. In: McMillan JA, Feigin RD, DeAngelis C,
Jones MD, eds. Oski's Pediatrics: Principles & Practice. 4th ed. Philadelphia, Pa:
Lippincott Williams & Wilkins; 2006:501-507
Centers for Disease Control and Prevention. Group B Strep Prevention: Laboratory Personnel,
Resistance in GBS. Available at:
http://www.cdc.gov/groupBstrep/lab/lab_pers_guidelines.Resistance.htm
Puopolo KM, Madoff LC, Eichenwald EC. Early-onset group B streptococcal disease in the era of
maternal screening. Pediatrics. 2005;115:1240-1246. Available at:
http://pediatrics.aappublications.org/cgi/content/full/115/5/1240
Venkatesh M, Merenstein GB, Adams KM, Weisman LE. Infection in the neonate. In: Merenstein
GB, Gardner SL, eds. Handbook of Neonatal Intensive Care. 6th ed. St.Louis, Mo: Mosby
Elsevier; 2006:569-593
Question: 52
You are seeing 16-year-old twin brothers for health supervision visits. They tell you that they
plan to spend most of the summer boating and fishing at their camp on the lake.
Of the following, the advice that is MOST likely to decrease their risk of a boating-related fatality
is to
D. post the phone number to the United States Coast Guard on the boat
References:
National Center for Injury Prevention and Control. Boating Safety: National Safe Boating Week:
May 20-26, 2006. Atlanta, Ga: Centers for Disease Control and Prevention; 2006. Available at:
http://www.cdc.gov/ncipc/duip/safeboatingweek.htm
Question: 53
A mother brings in her 18-month-old child in February with a “cold that won’t go away.”
According to your notes in his medical record, you saw him 10 days ago because of the acute
onset of fever to 103°F (39.5°C) with cough, coryza, listlessness, and decreased appetite.
Rapid influenza testing at that office visit was positive for influenza A. The mother reports that
the toddler seemed to improve for about 3 days, but physical examination today reveals purulent
rhinorrhea, temperature of 102°F (39°C), and a faint rash (Item Q53) on his trunk. The patient’s
7-year-old brother recently complained of a sore throat and had fever; he improved without
treatment after 3 days.
Of the following, the organism MOST likely to be responsible for this toddler’s illness is
A. Bordetella pertussis
Question: 53
Eruption composed of fine erythematous papules, as described for the child in the vignette.
Courtesy of D. Krowchuk
References:
American Academy of Pediatrics. Group A streptococcal infections. In: Pickering LK, Baker CJ,
Long SS, McMIllan JA, eds. Red Book: 2006 Report of the Committee on Infectious Diseases.
27th ed. Elk Grove Village, Ill: American Academy of Pediatrics; 2006:610-620
Jaggi P, Shulman ST. Group A streptococcal infections. Pediatr Rev. 2006;27:99-105. Available
at: http://pedsinreview.aappublications.org/cgi/content/full/27/3/99
Critique: 53
Question: 54
You care for a 6-month-old boy who was born with pulmonary atresia and ventricular septal
defect. He received a modified Blalock-Taussig (systemic-to-pulmonary artery) shunt 5 days
after birth. His oxygen saturations have ranged between 70% and 84% at office visits over the
past 2 months. During a health supervision visit, you record a hematocrit of 57% (0.57).
Of the following, this child’s polycythemia puts him at INCREASED risk for
A. acute leukemia
B. bacteremia
C. cerebrovascular accident
E. necrotizing enterocolitis
References:
Braunlin EA. Complications in chronic cyanotic heart disease. In: Moller JH, Hoffman JIE, eds.
Pediatric Cardiovascular Medicine. Philadelphia, Pa: Churchill Livingstone; 2000:939-941
Question: 55
During the health supervision visit of a 10-year-old boy, you note some wasting and weakness
of his lower leg muscles, with diminished patellar and ankle reflexes. You examine his parents’
legs and feet and notice that his mother has a bilateral foot drop and deformed feet (Item Q55).
B. celiac disease
E. hydrocephalus
Question: 55
Deformities of the feet (hammer toes), as exhibited by the mother of the boy described in the
vignette.
Courtesy of D. Gilbert
References:
Ravakumar R, Narayanan V. Hereditary neuropathy. In: Maria BL, ed. Current Management in
Child Neurology. 3rd ed. Hamilton, Ontario, Canada: BC Decker; 2005:391-398
Ryan MM, Ouvrier R. Hereditary peripheral neuropathies of childhood. Curr Opin Neurol.
2005;18:105-110. Abstract available at:
http://www.ncbi.nlm.nih.gov/entrez/query.fcgi?db=pubmed&cmd=Retrieve&dopt=AbstractPlus&li
st_uids=15791138
Sarnat HB. Hereditary motor-sensory neuropathies. In: Behrman RE, Kliegman RM, Jenson HB,
eds. Nelson Textbook of Pediatrics. 17th ed. Philadelphia, Pa: Saunders; 2004:2076-2077
Question: 56
You are called to the nursery to evaluate a newborn who has respiratory distress. On
observation, the baby appears pink while crying but becomes dusky when calm. You note a
hypoplastic nose and depressed nasal bridge on physical examination. A review of her chest
radiographs reveals stippling of the vertebrae.
A. alcohol
B. hydantoin
C. retinoic acid
D. valproic acid
E. warfarin
References:
Fetal alcohol syndrome. In: Jones KL. Smith’s Recognizable Patterns of Human Malformation.
6th ed. Philadelphia, Pa: Elsevier Saunders; 2006:646-651
Fetal hydantoin syndrome. In: Jones KL. Smith’s Recognizable Patterns of Human Malformation.
6th ed. Philadelphia, Pa: Elsevier Saunders; 2006:652-653
Fetal valproate syndrome. In: Jones KL. Smith’s Recognizable Patterns of Human Malformation.
6th ed. Philadelphia, Pa: Elsevier Saunders; 2006:654-655
Fetal warfarin syndrome. In: Jones KL. Smith’s Recognizable Patterns of Human Malformation.
6th ed. Philadelphia, Pa: Elsevier Saunders; 2006:656-657
Retinoic acid embryopathy. In: Jones KL. Smith’s Recognizable Patterns of Human
Malformation. 6th ed. Philadelphia, Pa: Elsevier Saunders; 2006:660-661
Critique: 56
Short palpebral fissures, midface hypoplasia, and a relatively narrow upper lip are characteristic
of fetal alcohol syndrome.
Courtesy of M. Rimsza
Critique: 56
Arched eyebrows, wide-spaced eyes, and a short upturned nose are characteristic of infants
who have been exposed to phenytoin during pregnancy.
Courtesy of M. Rimsza
Question: 57
A 15-year-old boy comes to your office for a sports physical examination. Compared with when
you saw him at his 10-year-old health supervision visit, he has progressed from Sexual Maturity
Rating 1 to 4.
Of the following, the laboratory value that is MOST likely to have changed is
A. calcium
B. hemoglobin
C. platelet count
D. serum albumin
References:
Fisher M. Laboratory testing. In: Friedman SB, Fisher MM, Schonberg SK, Alderman EM, eds.
Comprehensive Adolescent Health Care 2nd ed. St Louis, Mo: Mosby; 1998:80-86
Robertson J, Shilkofski N. Blood chemistries and body fluids. In: The Harriet Lane Handbook.
17th ed. Philadelphia, Pa: Elsevier Mosby; 2005:659-674
Question: 58
A mother brings in her 1-year-old boy for the first time because she is concerned about his
“bowed legs” (Item Q58A). The mother is 4 ft 10 in tall and says she needed to have surgery to
straighten out her bowed legs when she was an adolescent, as did one of her brothers.
Radiographs of the boy’s long bones are obtained (Item Q58B).
Question: 58
Courtesy of M. Rimsza
Question: 58
Courtesy of R. Schwartz
References:
Rauch F, Scheinman SJ, Agus ZS, Drezner MK. Hereditary hypophosphatemic rickets and
tumor-induced osteomalacia. UpToDate Online 14.3. Available for subscription at:
http://www.utdol.com/utd/content/topic.do?topicKey=pediendo/11337&type=P&selectedTitle=2~2
Singh J, Moghal N, Pearce SH, Cheetham T. The investigation of hypocalcaemia and rickets.
Arch Dis Child. 2003;88:403-407. Available at: http://adc.bmj.com/cgi/content/full/88/5/403
Singhal A, Campbell DE. Hypocalcemia. eMedicine Pediatrics Endocrinology. 2006. Available at:
http://www.emedicine.com/ped/topic1111.htm
Critique: 58
Anteroposterior radiograph of the lower extremities in rickets shows poor mineralization and
widening and fraying of the metaphyses that is most marked at the knees.
Courtesy of D. Mulvihill
Question: 59
A 12-year-old girl is receiving learning support because of difficulty with reading and language
arts. She struggles to do well in the classroom but realizes she can never compete successfully
with her older sister academically. She is well-coordinated and enjoys playing basketball in her
yard. Her parents are concerned by her negative comments about herself and ask your advice
on how they can help her improve her self-image.
References:
Brooks R. How can parents foster self-esteem in their children? SchwabLearning.org. Available
at: http://www.schwablearning.org/articles.asp?r=400
Dworkin PH. School failure. In: Parker S, Zuckerman B, Augustyn M, eds. Developmental and
Behavioral Pediatrics: A Handbook for Primary Care. 2nd ed. Philadelphia, Pa: Lippincott
Williams & Williams; 2005:280-284
Lambros KM, Leslie LK. Management of the child with a learning disorder. Pediatr Ann.
2005;34:275-287. Abstract available at:
http://www.ncbi.nlm.nih.gov/entrez/query.fcgi?db=pubmed&cmd=Retrieve&adopt=AbsractPlus&li
st_uids=15871432
Question: 60
A 12-year-old boy who has a history of Hodgkin disease presents to the emergency department
with fever, headache, and malaise of 4 hours’ duration. He has not received any cancer
treatment in more than 1 year. His immunizations are up to date, including the 23-valent
pneumococcal, Haemophilus influenzae type b, and polysaccharide meningococcal vaccines.
He has had a splenectomy. He currently takes oral penicillin twice daily. Physical examination
reveals a toxic-appearing boy who has a temperature of 104°F (40°C), heart rate of 110
beats/min, respiratory rate of 30 breaths/min, and a blood pressure of 118/76 mm Hg. No other
findings on the physical examination suggest the source of his fever.
Of the following, the MOST likely cause of his fever is infection with
B. Pseudomonas aeruginosa
C. Staphylococcus aureus
D. Streptococcus pneumoniae
E. Streptococcus pyogenes
References:
Price VE, Dutta S, Blanchette VS, et al. The prevention and treatment of bacterial infections in
children with asplenia or hyposplenia: practice considerations at the Hospital for Sick Children,
Toronto. Pediatr Blood Cancer. 2006;46:597-603. Abstract available at:
http://www.ncbi.nlm.nih.gov/entrez/query.fcgi?db=pubmed&cmd=Retrieve&dopt=AbstractPlus&li
st_uids=16333816
Schutze GE, Mason EO Jr, Barson WJ, et al. Invasive pneumococcal infections in children with
asplenia. Pediatr Infect Dis J. 2002;21:278-282. Abstract available at:
http://www.ncbi.nlm.nih.gov/entrez/query.fcgi?db=pubmed&cmd=Retrieve&dopt=AbstractPlus&li
st_uids=12075756
Question: 61
You are evaluating a 6-year-old boy who has a 2-day history of fever (102ºF [38.9°C]),
headache, vomiting, moderate crampy periumbilical pain, and watery diarrheal stools (six to
eight a day) that have become blood-streaked in the last 12 hours. He just returned yesterday
from a 3-week visit to his grandparents’ farm in India. Physical examination reveals a tired boy
who has a temperature of 102.5ºF (39.2°C), tacky mucous membranes, and dry lips. He has
diffuse abdominal tenderness to palpation that is most pronounced in the periumbilical area and
associated with some voluntary guarding but no rebound. Laboratory tests show a peripheral
white blood cell count of 9.0 x103/mcL (9.0x109/L) with 60% polymorphonuclear leukocytes, 5%
band forms, 30% lymphocytes, and 5% monocytes. A test for the presence of fecal leukocytes
is positive. You suspect Campylobacter sp as the cause of this patient’s condition.
Of the following, the laboratory condition that is REQUIRED to isolate this organism is
References:
American Academy of Pediatrics. Campylobacter infections. In: Pickering LK, Baker CJ, Long
SS, McMillan JA, eds. Red Book: 2006 Report of the Committee on Infectious Diseases. 27th ed.
Elk Grove Village, Ill: American Academy of Pediatrics; 2006:240-242
Gill VJ, Fedorko DP, Witebsky FG. The clinician and the microbiology laboratory. In: Mandell GL,
Bennett JE, Dolin R, eds. Mandell, Douglas and Bennett’s Principles and Practice of Infectious
Diseases. 6th ed. Philadelphia, Pa: Elsevier Churchill Livingstone; 2005:203-241
Penner JL. The genus Campylobacter: a decade of progress. Clin Microbiol Rev. 1988;1:157-
172. Abstract available at:
http://www.ncbi.nlm.nih.gov/entrez/query.fcgi?db=pubmed&cmd=Retrieve&dopt=AbstractPlus&li
st_uids=3069194
Question: 62
A 4-month-old girl presents with fever. Results of urinalysis include 50 to 100 white blood cells
per high-power field and 3+ bacteria. Urine culture is positive for Escherichia coli.
Ultrasonography reveals hydroureteral nephrosis of the left upper pole, and voiding
cystourethrography shows a filling defect within the bladder (Item Q62).
Of the following, in addition to a urinary tract infection, the infant is MOST likely to have
A. bladder diverticulum
C. ureteral stone
E. ureterocele
Question: 62
Fluoroscopic image from a voiding cystourethrogram showing a defect within the contrast-filled
bladder, as described for the patient in the vignette. There is reflux into both lower pole collecting
systems of duplex kidneys.
Courtesy of D. Mulvihill
References:
Critique: 62
Fluoroscopic image from a voiding cystourethrogram showing a large mass within the contrast-
filled bladder consistent with an ureterocele. There is reflux into both lower pole collecting
systems of duplex kidneys.
Courtesy of D. Mulvihill
Critique: 62
Intravenous pyelography of ureteropelvic junction obstruction shows a distended left renal pelvis
with an abrupt change in caliber to a normal-sized ureter (arrow).
Courtesy of D. Mulvihill
Critique: 62
Noncontrast axial computed tomography (CT) scan of the pelvis shows a density at the
ureterovesical junction consistent with a ureteral stone. CT is the preferred imaging method for
the patient suspected of having a genitourinary stone.
Courtesy of D. Mulvihill
Critique: 62
A fluoroscopic spot film from a voiding cystourethrogram shows a significant change in the caliber
of the urethra at the level of the posterior urethral folds that is consistent with valves.
Courtesy of D. Mulvihill
Critique: 62
Courtesy of D. Mulvihill
Question: 63
A 16-year-old girl who has moderate persistent asthma presents to the emergency department
with coughing, wheezing, and increasing dyspnea. She states that she was feeling fine until she
was exposed to cologne that one of her classmates was wearing. An ambulance was called
after her symptoms did not improve following administration of two puffs of her beta-2 agonist
inhaler. On physical examination, the teenager has a respiratory rate of 30 breaths/min, heart
rate of 90 beats/min, and pulse oximetry of 98% on room air. She has difficulty completing a
sentence and points to her neck, saying it is “hard to get air in.” Her lungs are clear to
auscultation, and rhinolaryngoscopy demonstrates adduction of one of the vocal cords during
inspiration with a posterior glottic “chink.” Pulmonary function testing shows a blunted inspiratory
loop (Item Q63).
Of the following, the MOST likely cause for this patient’s symptoms is
A. asthma exacerbation
B. subglottic stenosis
Question: 63
Blunted inspiratory flow curve in pink (arrow), as exhibited by the patient in the vignette.
Courtesy of K. Waibel
References:
Lasley MV. New treatments for asthma. Pediatr Rev. 2003;24:222-232. Available at:
http://pedsinreview.aappublications.org/cgi/content/full/24/7/222
Liu AH, Spahn JD, Leung DYM. Childhood asthma. In: Behrman RE, Kliegman RM, Jenson HB,
eds. Nelson Textbook of Pediatrics. 17th ed. Philadelphia, Pa: Saunders; 2004:760-773
Critique: 63
Expiratory (top) and inspiratory (bottom) flow loops demonstrate a normal flow-volume curve
(blue) and a curve consistent with vocal cord dysfunction (pink). Symptomatic patients who have
vocal cord dysfunction exhibit the characteristic blunted inspiratory loop (arrow) caused by
paradoxic adduction of the vocal cords during inspiration.
Courtesy of K. Waibel
Critique: 63
Acquired subglottic stenosis. Note the circumferential narrowing of the subglottic area (arrow).
Courtesy of D. Kirse
Critique: 63
Vocal cord nodules appear as growths located anteriorly on the vocal cords.
Courtesy of D. Kirse
Question: 64
A 1-year-old year boy who has chronic kidney disease from posterior urethral valves presents
to your office because his breathing has been noisy for the past 2 hours. His usual medications
include calcium carbonate and vitamin D. On physical examination, you note inspiratory stridor.
He has not had upper respiratory tract symptoms or fever, and there is no history of choking.
The mother reports that the boy was seen by the nephrologist 1 week ago, and because of
worsening renal function, he was begun on sodium bicarbonate.
Of the following, the electrolyte abnormality that BEST explains his current symptoms is
A. hyperkalemia
B. hypermagnesemia
C. hypocalcemia
D. hyponatremia
E. hypophosphatemia
References:
Greenbaum LA. Electrolyte and acid-base disorders. In: Behrman RE, Kleigman R, Jenson HB,
eds. Nelson Textbook of Pediatrics. 17th ed. Philadelphia, Pa: Saunders; 2004:191-241
Srivastava T, Warady DA. Bone metabolism and renal osteodystrophy in children with renal
disease. UpToDate. Online 14.3. Available at:
http://www.utdol.com/utd/content/topic.do?topicKey=pedineph/14068&type=P&selectedTitle=46~
49
Question: 65
You are asked to evaluate a healthy 9-year-old boy who has a rash of several weeks’ duration.
Physical examination reveals several erythematous plaques (Item Q65) on the trunk
surmounted by a thick, adherent scale. You observe pinpoint areas of hemorrhage on the
surfaces of some lesions.
A. nummular eczema
B. pityriasis rosea
C. psoriasis
D. seborrheic dermatitis
E. tinea corporis
Question: 65
Courtesy of D. Krowchuk
References:
Krowchuk DP, Mancini AJ, eds. Psoriasis. In: Pediatric Dermatology. A Quick Reference Guide.
Elk Grove Village, Ill: American Academy of Pediatrics; 2007:253-256
Paller AS, Mancini AJ. Papulosquamous and related disorders. In: Hurwitz Clinical Pediatric
Dermatology. 3rd ed. Philadelphia, Pa: Elsevier Saunders; 2006:85-106
Weston WL, Lane AT, Morelli JG. Papulosquamous disorders. In: Color Textbook of Pediatric
Dermatology. 3rd ed. St. Louis, Mo: Mosby; 2002:89:119-143
Critique: 65
When scale is removed from the surface of psoriatic lesions, pinpoint areas of bleeding (arrow)
often are observed (ie, Auspitz sign).
Courtesy of D. Krowchuk
Critique: 65
Courtesy of D. Krowchuk
Critique: 65
Courtesy of D. Krowchuk
Critique: 65
Courtesy of D. Krowchuk
Critique: 65
Pitting (shallow depressions) of the nails may occur in patients who have psoriasis.
Courtesy of D. Krowchuk
Critique: 65
Lesions of psoriasis may occur in areas of trauma (ie, the Koebner phenomenon).
Courtesy of dermatlas.org
Critique: 65
Courtesy of dermatlas.org
Critique: 65
Courtesy of D. Krowchuk
Critique: 65
In pityriasis rosea, lesions are aligned with long axes parallel to lines of skin stress. On the back,
this alignment may mimic the appearance of the branches of a fir tree.
Courtesy of D. Krowchuk
Critique: 65
The lesions of seborrheic dermatitis are erythematous macules or patches that have a greasy
scale.
Courtesy of D. Krowchuk
Critique: 65
Tinea corporis produces annular lesions that have an elevated scaling border and central
clearing.
Courtesy of D. Krowchuk
Question: 66
During rounds with a group of medical students in the intensive care unit, you review the case of
an 8-year-old boy who presented with gastrointestinal bleeding, received a blood transfusion,
and underwent endoscopy. An ulcer was identified in the duodenal bulb (Item Q66). The boy
now is being treated with intravenous pantoprazole.
Of the following, the BEST explanation of the mechanism of action of pantoprazole is that it
Question: 66
Large ulcer in the duodenal bulb, as described for the boy in the vignette. The ulcer crater is
covered by whitish exudate.
Courtesy of A. Bousvaros
References:
Laheij RJ, Sturkenboom MC, Hassing RJ, Dieleman J, Stricker BH, Jansen JM. Risk of
community-acquired pneumonia and use of gastric acid-suppressive drugs. JAMA
2004;292:1955-1960. Abstract available at:
http://www.ncbi.nlm.nih.gov/entrez/query.fcgi?db=pubmed&cmd=Retrieve&dopt=AbstractPlus&li
st_uids=15507580
Patel AS, Pohl JF, Easley DJ. What’s new: proton pump inhibitors and pediatrics. Pediatr Rev.
2003;24:12-15. Available at: http://pedsinreview.aappublications.org/cgi/content/full/24/1/12
Critique: 66
Drawing of a gastric parietal cell. Omeprazole and lansoprazole inhibit the hydrogen-potassium
ATPase pump; ranitidine and famotidine block histamines effect by binding the histamine-2
receptor.
Courtesy of A. Bousvaros
Question: 67
You are measuring serum electrolytes at 12 hours of age in a 4,500-g infant delivered by
cesarean section at 36 weeks’ gestation. The Apgar scores were 6 and 8 at 1 and 5 minutes,
respectively. The infant is in no acute distress, breathing room air, and generally well-appearing,
although he exhibits mild hypotonia. The laboratory results are:
Of the following, the MOST likely cause of neonatal hypocalcemia for this infant is
B. hypoglycemia
E. vitamin D deficiency
References:
Hsu SC, Levine MA. Perinatal calcium metabolism: physiology and pathophysiology. Semin
Neonatol. 2004;9:23-36. Abstract available at:
http://www.ncbi.nlm.nih.gov/entrez/query.fcgi?db=pubmed&cmd=Retrieve&dopt=AbstractPlus&li
st_uids=15013473
Rubin LP. Disorders of calcium and phosphorus metabolism. In: Taeusch HW, Ballard RA,
Gleason CA, eds. Avery’s Diseases of the Newborn. 8th ed. Philadelphia, Pa: Elsevier
Saunders; 2005:1346-1365
Walton DM, Thomas DC, Aly HZ, Short BL. Morbid hypocalcemia associated with phosphate
enema in a six-week-old infant. Pediatrics. 2000;106:e37. Available at:
http://pediatrics.aappublications.org/cgi/content/full/106/3/e37
Critique: 67
Question: 68
A 2-year-old girl presents to the clinic with anal itching. Her perianal area is slightly reddened and
shows mild excoriation. A tape test from the perianal area reveals the structures shown in Item
68 (Item Q68).
Question: 68
References:
American Academy of Pediatrics. Pinworm infection (Enterobius vermicularis). In: Pickering LK,
Baker CJ, Long SS, McMIllan JA, eds. Red Book: 2006 Report of the Committee on Infectious
Diseases. 27th ed. Elk Grove Village, Ill: American Academy of Pediatrics; 2006:520-522
Robinson J. Infectious diseases in schools and child care facilities. Pediatr Rev. 2001;22:39-46.
Available at: http://pedsinreview.aappublications.org/cgi/content/full/22/2/39
Critique: 68
Pinworm infestation: Frames A and B are eggs, and Frame C is the appearance of eggs on
cellulose tape test.
Courtesy of Red Book Online and the Centers for Disease Control and Prevention
Question: 69
The father of a 9-year-old boy calls you because his son has been exposed to several children
at school who have “strep throat.” Although the boy appears well and has no symptoms, the
father notes that “all the other parents” are getting antibiotics for their children. He asks your
advice.
C. offer to see the boy immediately to examine him for signs of pharyngitis
E. tell the father to bring the boy in immediately for an injection of benzathine penicillin
References:
American Academy of Pediatrics. Group B streptococcal infections. In: Pickering LK, Baker CJ,
Long SS, McMillan JA, eds. Red Book: 2006 Report of the Committee on Infectious Diseases.
27th ed. Elk Grove Village, Ill: American Academy of Pediatrics; 2006:620-627
Question: 70
During a routine health supervision visit, the mother of one of your patients informs you that she
is 28 weeks pregnant and that fetal echocardiography has revealed tetralogy of Fallot in an
otherwise normal fetus. She asks you about the prognosis for children who have this condition.
B. complete repair is associated with an excellent result, with less than 5% mortality
C. surgery most likely will be undertaken before the child is discharged after birth
References:
Aboulhosn J, Child JS. Management after childhood repair of tetralogy of Fallot. Curr Treat
Options Cardiovasc Med. 2006;8:474-483.
http://www.ncbi.nlm.nih.gov/entrez/query.fcgi?db=pubmed&cmd=Retrieve&dopt=AbstractPlus&li
st_uids=17078912
Shillingford AJ, Wernovsky G. Academic performance and behavioral difficulties after neonatal
and infant heart surgery. Pediatr Clin North Am. 2004;51:1625-1639.
http://www.ncbi.nlm.nih.gov/entrez/query.fcgi?db=pubmed&cmd=Retrieve&dopt=AbstractPlus&li
st_uids=15561177
Tweddell J, Spray TL. Newborn heart surgery: reasonable expectations and outcomes. Pediatr
Clin North Am. 2004;51:1611-1624.
http://www.ncbi.nlm.nih.gov/entrez/query.fcgi?db=pubmed&cmd=Retrieve&dopt=AbstractPlus&list
_uids=15561176
Question: 71
A 5-year-old girl presents with leg weakness of 12 hours’ duration. She is afebrile and describes
no pain in her back or elsewhere. Findings on physical examination include distal leg weakness
and diminished reflexes in the legs. Finger-to-nose testing reveals slight dysmetria. Rectal tone
is normal. You find an engorged tick on her occipital scalp.
A. botulism
B. Guillain-Barré syndrome
C. hydrocephalus
D. tick paralysis
E. transverse myelitis
References:
Daugherty RJ, Posner JC, Henretig FM, McHugh LA, Tan CG. Tick paralysis: atypical
presentation, unusual location. Pediatr Emerg Care. 2005;21:677-680. Abstract available at:
http://www.ncbi.nlm.nih.gov/entrez/query.fcgi?db=pubmed&cmd=Retrieve&dopt=AbstractPlus&li
st_uids=16215474
Li Z, Turner RP. Pediatric tick paralysis: discussion of two cases and literature review. Pediatr
Neurol. 2004;31:304-307. Abstract available at:
http://www.ncbi.nlm.nih.gov/entrez/query.fcgi?db=pubmed&cmd=Retrieve&dopt=AbstractPlus&li
st_uids=15464647
Sarnat HB. Disorders of neuromuscular transmission and of motor neurons. In: Behrman RE,
Kliegman RM, Jenson HB, eds. Nelson Textbook of Pediatrics. 17th ed. Philadelphia, Pa:
Saunders; 2004:2072-2076
Vedanarayanan V, Sorey WH, Subramony SH. Tick paralysis. Semin Neurol. 2004;24:181-184.
Abstract available at:
http://www.ncbi.nlm.nih.gov/entrez/query.fcgi?db=pubmed&cmd=Retrieve&dopt=AbstractPlus&li
st_uids=15257515
Question: 72
Parents who are new to your area bring their 4-year-old son to your office for evaluation of poor
growth and developmental delays. They say that he was growing and developing normally until
about 2 years of age. He now has a few single words. You review his records and note that his
height was at the 50th percentile at age 2, and it is at the 10th percentile today. The
occipitofrontal circumference is at the 97th percentile. On physical examination, the boy has
dark and heavy eyebrows, full cheeks (Item Q72A), and full lips. His hands are broad, and his
fingers are stubby and cannot be fully straightened (Item Q72B). There are flesh-colored
nodules over the scapular regions and hepatosplenomegaly. A review of the family history
reveals that the mother had two maternal uncles who died in mid- to late childhood of unknown
causes.
Of the following, the condition that is MOST consistent with this child’s presentation is
B. Hunter syndrome
C. Hurler syndrome
D. I-cell disease
E. Sanfilippo syndrome
Question: 72
Heavy eyebrows and full cheeks, as exhibited by the boy in the vignette.
Courtesy of M. Rimsza
Question: 72
Inability to extend the fingers fully, as exhibited by the boy in the vignette.
Courtesy of M. Rimsza
References:
Nyhan WL, Barshop BA, Ozand PT. Glycogenosis type I — von Gierke disease. In: Atlas of
Metabolic Diseases. 2nd ed. New York, NY: Oxford University Press; 2005:385-393
Nyhan WL, Barshop BA, Ozand PT. Mucopolysaccharidoses. In: Atlas of Metabolic Diseases.
2nd ed. New York, NY: Oxford University Press; 2005:499-550
Critique: 72
Patients who have Hurler syndrome may develop corneal clouding. This patient also has
strabismus.
Courtesy of M. Rimsza
Question: 73
A 16-year-old girl comes to your office complaining of irregular menstrual periods. She had
menarche at 11 years of age and experiences menstrual periods every 2 weeks to 3 months.
She has noticed increased acne, facial hair growth, and a 20-lb weight gain over the past year.
A. Cushing syndrome
B. hypothyroidism
C. Noonan syndrome
D. ovarian tumor
References:
Emans SJ. Androgen abnormalities in the adolescent girl. In: Emans SJ, Laufer MR, Goldstein
DP, eds. Pediatric and Adolescent Gynecology. 5th ed. Philadelphia, Pa: Lippincott, Williams &
Wilkins; 2005:287-333
Critique: 73
Patients who have Cushing syndrome often exhibit large purple striae.
Courtesy of M. Rimsza
Critique: 73
Courtesy of M. Rimsza
Question: 74
An obstetrician calls to tell you she is caring for a woman who is 36 weeks pregnant and has
required treatment with propylthiouracil during pregnancy for Graves disease. The mother is
worried about the risk of neonatal thyrotoxicosis in the infant and wishes advice.
Of the following, a TRUE statement about infants born to mothers who have Graves disease is
that
A. approximately 50% of infants have elevated concentrations of thyroid hormones, but only
20% require treatment
E. infants whose mother’s disease is controlled adequately during pregnancy have a decreased
risk of thyrotoxicosis
References:
LaFranchi S. Evaluation and management of neonatal Graves’ disease. UpToDate Online 14.3.
Available at:
http://www.utdol.com/utd/content/topic.do?topicKey=pediendo/5108&type=P&selectedTitle=5~6
Ogilvy-Stuart AL. Neonatal thyroid disorders. Arch Dis Child Fetal Neonatal Ed. 2002;87:F165-
F171. Available at: http://fn.bmj.com/cgi/content/full/87/3/F165
Question: 75
A 5-year-old girl recently was diagnosed with an autistic disorder and mental retardation. Her
parents are upset by her lack of progress in her special education program and seek your
guidance in treating her autism. At a parent support group, they were told about the use of
complementary and alternative medical approaches to therapy. They ask whether they should
pursue these interventions.
A. explain to the parents that alternative treatments have been demonstrated to be ineffective
B. explain to the parents that they must consider the benefits, risks, and evidence regarding
efficacy for each treatment
C. refer the parents for psychological counseling to deal with their guilt feelings
D. suggest the parents discuss the alternative treatments with the special education teachers
E. tell the parents that they should not expect much progress because their daughter has both
mental retardation and autism
References:
Cohen MH, Kemper KJ, Stevens L, Hashimoto D, Gilmour J. Pediatric use of complementary
therapies: ethical and policy choices. Pediatrics. 2005;116:e568-e575. Available at:
http://pediatrics.aappublications.org/cgi/content/full/116/4/e568
Committee on Children With Disabilities. Technical report: the pediatrician's role in the diagnosis
and management of autistic spectrum disorder in children. Pediatrics. 2001;107:e85. Available
at: http://pediatrics.aappublications.org/cgi/content/full/107/5/e85
Levy SE, Mandell DS, Merhar S, Ittenbach RF, Pinto-Martin JA. Use of complementary and
alternative medicine among children recently diagnosed with autistic spectrum disorder. J Dev
Behav Pediatr. 2003;24:418-423. Abstract available at:
http://www.ncbi.nlm.nih.gov/entrez/query.fcgi?db=pubmed&cmd=Retrieve&adopt=AbsractPlus&li
st_uids=14671475
Question: 76
You receive a phone call at your office from a resident, who is making rounds in the hospital.
She tells you that computed tomography scan of the 14-year-old girl you admitted last night with
a history of fever and increasing headaches revealed a large brain abscess and asks your
advice on choice of antimicrobial therapy.
Of the following, the BEST combination of antimicrobial agents to start for this patient is
B. clindamycin + ceftriaxone
D. piperacillin-tazobactam + gentamicin
References:
Haslam RHA. Brain abscess. In: Behrman RE, Kliegman RM, Jenson HB, eds. Nelson Textbook
of Pediatrics. 17th ed. Philadelphia, Pa: Saunders; 2004:2047
Critique: 76
Brain abscess: Computed tomography scan with contrast demonstrates a rim-enhancing lesion
(arrow) extending to an area of bony destruction in an opacified frontal sinus. Vasogenic edema
surrounding the abscess causes mass effect and shift of the midline to the right.
Courtesy of D. Krowchuk
Question: 77
A 6-month-old male infant was treated for congenital syphilis at 2 weeks of age with 14 days of
intravenous penicillin. At the time of his diagnosis, physical examination demonstrated only
hepatosplenomegaly. Laboratory evaluation showed a rapid plasma reagin of 1:16, a negative
cerebrospinal fluid VDRL result, and a positive fluorescent treponemal antibody absorption
result.
Of the following, the MOST appropriate test to determine if the patient has been treated
successfully is
A. blood culture
References:
American Academy of Pediatrics. Syphilis. In: Pickering LK, Baker CJ, Long SS, McMillan JA,
eds. Red Book: 2006 Report of the Committee on Infectious Diseases. 27th ed. Elk Grove
Village, Ill: American Academy of Pediatrics; 2006:631-644
Brown ST, Zaidi A, Larsen SA, Reynolds GH. Serological response to syphilis treatment: a new
analysis of old data. JAMA. 1985;253:1296-1299. Abstract available at:
http://www.ncbi.nlm.nih.gov/entrez/query.fcgi?db=pubmed&cmd=Retrieve&dopt=AbstractPlus&li
st_uids=3155812
Sanchez PJ, Gutman LT. Syphilis. In: Feign RD, Cherry JD, Demmler GJ, Kaplan SL, eds.
Textbook of Pediatric Infectious Diseases. 5th ed. Philadelphia, Pa: Saunders; 2004:1724-1742
Tramont E. Treponema pallidum (syphilis). In: Mandell GL, Bennett JE, Dolin R, eds. Mandell,
Douglas and Bennett’s Principles and Practice of Infectious Diseases. 6th ed. Philadelphia, Pa:
Elsevier Churchill Livingstone; 2005:2768-2784
Critique: 77
Courtesy of Centers for Disease Control and Prevention, Public Health Image Library
Question: 78
During your evaluation of a 16-year-old boy at his health supervision visit, he reports that he had
an episode of cola-colored urine associated with an upper respiratory tract infection 6 weeks
ago. The only significant finding on his medical history is mild hearing loss. Today his blood
pressure is 112/66 mm Hg. Urinalysis shows 3+ blood, 2+ protein, and 20 to 50 red blood cells
per high-power field (RBC/HPF). Other laboratory findings include:
Serum complement components (C3 and C4) are normal, and antinuclear antibody titers are
negative. You discuss these results with his mother, who then tells you that she recalls being
told she had blood in her urine. Urinalysis for the child’s mother demonstrates 2+ blood with 10 to
20 RBC/HPF.
A. boys who have this condition are more likely to develop chronic renal failure than are girls
D. the biologic defect in the kidney is believed to involve the glomerular epithelial cell
References:
Fouser L. Consultation with the specialist: familial nephritis/Alport syndrome. Pediatr Rev.
1998;19:265-267. Available at: http://pedsinreview.aappublications.org/cgi/content/full/19/8/265
Jais JP, Knebelmann B, Giatras I, et al. X-linked Alport syndrome: natural history in 195 families
and genotype-phenotype correlations in males. J Am Soc Nephrol. 2000;11:649-657. Abstract
available at:
http://jasn.asnjournals.org/cgi/gca?allch=&SEARCHID=1&FULLTEXT=alport+syndrome&VOLU
ME=11&ISSUE=4&FIRSTINDEX=0&hits=10&RESULTFORMAT=&gca=jnephrol%3B11%2F4%2
F649&allchb=
Kashtan CE. Familial hematurias: what we know and what we don't. Pediatr Nephrol.
2005;20:1027-1035. Abstract available at:
Abstract available at:
http://jasn.asnjournals.org/cgi/gca?allch=&SEARCHID=1&FULLTEXT=alport+syndrome&VOLU
ME=11&ISSUE=4&FIRSTINDEX=0&hits=10&RESULTFORMAT=&gca=jnephrol%3B11%2F4%2
F649&allchb=
Rana K, Wang YY, Buzza M, et al. The genetics of thin basement membrane nephropathy.
Semin Nephrol. 2005;25:163-170. Abstract available at:
http://www.ncbi.nlm.nih.gov/entrez/query.fcgi?db=pubmed&cmd=Retrieve&dopt=AbstractPlus&li
st_uids=15880327
Question: 79
A 17-year-old boy is applying for entry into military service and requires a complete history and
physical examination. During the interview, he states that he is healthy, although he admits to
being treated for three cases of pneumonia over the past 10 years. A chest radiograph
performed during the last infection showed a left lower lobe pneumonia, and the patient states
that the infection is “always on that side.” The only finding of note on the physical examination
today is slightly diminished breath sounds over the left lower lobe.
Of the following, the MOST likely cause for this boy’s recurrent pneumonias is
A. bronchogenic cyst
D. extrapulmonary sequestration
E. intrapulmonary sequestration
References:
Corbett HJ, Humphrey GM. Pulmonary sequestration. Paediatr Respir Rev. 2004;5:59-68.
Abstract available at:
http://www.ncbi.nlm.nih.gov/entrez/query.fcgi?db=pubmed&cmd=Retrieve&dopt=AbstractPlus&li
st_uids=15222956
Green TP, Finder JD. Congenital disorders of the lung. In: Behrman RE, Kliegman RM, Jenson
HB, eds. Nelson Textbook of Pediatrics. 17th ed. Philadelphia, Pa: Saunders; 2004:1423-1425
Critique: 79
Anteroposterior radiograph of the chest shows multiple large cysts in the right lower lobe. Several
of the cysts contain air fluid levels (arrow), likely due to superimposed infection, a common
presentation for intralobar sequestration.
Courtesy of D. Mulvihill
Critique: 79
Lateral radiograph of the chest shows a single large bronchogenic cyst in the left lung without
internal septations.
Courtesy of D. Mulvihill
Critique: 79
Courtesy of D. Mulvihill
Critique: 79
Anteroposterior radiograph of the chest in congenital lobar emphysema shows overinflation of the
left upper lobe with shift of the mediastinum to the right. There is left lower lobe atelectasis.
Courtesy of D. Mulvihill
Question: 80
A 14-year-old boy presents to the office with a 4-hour history of severe scrotal pain after running
in a track meet. He is pale, nauseous, and in obvious pain. He denies trauma, dysuria, history of
fever, or penile discharge. He is sexually active. On physical examination, his penis appears
normal and without meatal discharge, but the scrotum is swollen bilaterally, and there is
significant tenderness to palpation of both testicles. The testicles appear high in the scrotum,
and the cremasteric reflexes are absent bilaterally.
Of the following, the next MOST appropriate step in the evaluation of this patient is
E. urinalysis
References:
Adelman WP, Joffe A. Consultation with the specialist: testicular masses/cancer. Pediatr Rev.
2005;26:341-344. Available at: http://pedsinreview.aappublications.org/cgi/content/full/26/9/341
Brenner JS, Aderonke O. Causes of scrotal pain in children and adolescents. UpToDate. Online
14.3. Available at:
http://www.utdol.com/utd/content/topic.do?topicKey=adol_med/6756&type=P&selectedTitle=5~9
Leslie JA, Cain MP. Pediatric urologic emergencies and urgencies. Pediatr Clin North Am.
2006;53:513-527. Abstract available at:
http://www.ncbi.nlm.nih.gov/entrez/query.fcgi?db=pubmed&cmd=Retrieve&dopt=AbstractPlus&li
st_uids=16716794
Critique: 80
Critique: 80
Normally (left), the tunica vaginalis attaches to the posterolateral aspect of the testis. In the bell-
clapper deformity (center), the tunica vaginalis completely encircles the distal spermatic cord,
epididymis, and testis, allowing the structures to swing and rotate within the tunica vaginalis (like a
clapper in a bell). This abnormality predisposes to testicular torsion (right).
Courtesy of A. Johnson
Question: 81
An 8-year-old boy presents with a 2-week history of an enlarging, tender lump on the scalp. The
only notable findings on physical examination are alopecia overlying a boggy mass on the scalp
and posterior cervical lymphadenopathy (Item Q81).
A. cefazolin intravenously
B. griseofulvin orally
D. ketoconazole topically
E. mupirocin topically
Question: 81
Courtesy of M. Rimsza
References:
Krowchuk DP, Mancini AJ, eds. Tinea capitis. In: Pediatric Dermatology. A Quick Reference
Guide. Elk Grove Village, Ill: American Academy of Pediatrics; 2007:193-197
Paller AS, Mancini AJ. Skin disorders due to fungi. In: Hurwitz Clinical Pediatric Dermatology. 3rd
ed. Philadelphia, Pa: Elsevier Saunders; 2006:449-478
Roberts BJ, Friedlander SF. Tinea capitis: a treatment update. Pediatr Ann. 2005;34:191-200.
Abstract available at:
http://www.ncbi.nlm.nih.gov/entrez/query.fcgi?db=pubmed&cmd=Retrieve&dopt=AbstractPlus&li
st_uids=15792111
Weston WL, Lane AT, Morelli JG. Fungal and yeast infections of the skin. In: Color Textbook of
Pediatric Dermatology. 3rd ed. St. Louis, Mo: Mosby; 2002:89:63-76
Question: 82
A 14-year-old boy who recently emigrated from Vietnam is brought in by his mother because of
intermittent abdominal pain and vomiting of 1 year’s duration. He is a runner and has taken four
doses of ibuprofen this month for chronic knee pain. In the past, he has been treated with
ranitidine, which partially relieved the abdominal symptoms. You refer him to a
gastroenterologist, who performs endoscopy and discovers a nodular antrum (Item Q82) and
small duodenal ulcer.
Of the following, the MOST likely diagnosis for his symptoms and endoscopic findings is
B. Crohn disease
C. eosinophilic gastritis
Question: 82
Courtesy of A. Bousvaros
References:
Gold BD, Colletti RB, Abbott M, et al; North American Society for Pediatric Gastroenterology and
Nutrition. Helicobacter pylori infection in children: recommendations for diagnosis and treatment.
J Pediatr Gastroenterol Nutr. 2000;31:490-497. Available at:
http://www.jpgn.org/pt/re/jpgn/fulltext.00005176-200011000-00007.htm
Rowland M, Bourke B, Drumm B. Helicobacter pylori and peptic ulcer disease. In: Walker WA,
Goulet O, Kleinman RE, Sherman PM, Shneider BL, Sanderson IR, eds. Pediatric
Gastrointestinal Disease: Pathophysiology, Diagnosis, Management. 4th ed. Hamilton, Ontario,
Canada: BC Decker; 2004:491-512
Critique: 82
Endoscopic view of the stomach demonstrating nodular inflammation in the gastric antrum, a
common feature of Helicobacter pylori gastritis.
Courtesy of A. Bousvaros
Critique: 82
Silver stain of a gastric biopsy demonstrating curved-shaped H pylori bacteria in a gastric crypt
(arrows).
Courtesy of J. Glickman
Question: 83
A 5-day-old term infant presents to the emergency department with a history of bile-stained
emesis. She is well nourished and hydrated and had an unremarkable course in the newborn
nursery. She was discharged at 48 hours after birth and was breastfeeding, but her mother
states the baby always has vomited. Physical examination reveals an afebrile infant who has
normal vital signs, but no audible bowel sounds on abdominal evaluation. A flat-plate abdominal
radiograph reveals a paucity of bowel gas (Item Q83).
A. anorectal atresia
B. cystic fibrosis
D. septic ileus
E. tracheoesophageal fistula
Question: 83
Anteroposterior radiograph of the abdomen shows dilated loops of bowel in the upper abdomen
but no gas distally.
Courtesy of D. Mulvihill
References:
Dutta S, Albanese CT. Minimal access surgery in the neonate. NeoReviews. 2006;7:e400-e409.
Available at: http://neoreviews.aappublications.org/cgi/content/full/7/8/e400
Pursley D, Hansen AR, Puder M. Gastrointestinal disorders: part 4: obstruction. In: Hansen AR,
Puder M, eds. Manual of Neonatal Surgical Intensive Care. Hamilton, Ontario, Canada: BC
Decker, Inc; 2003:237-259
Roaten JB, Bensard DD, Price FN. Neonatal surgery. In: Merenstein GB, Gardner SL, eds.
Handbook of Neonatal Intensive Care. 6th ed. St.Louis, Mo: Mosby Elsevier; 2006:838-862
Critique: 83
Anteroposterior radiograph of the abdomen shows dilated loops of bowel in the upper abdomen
but no gas distally, findings that are consistent with malrotation and midgut volvulus.
Courtesy of D. Mulvihill
Critique: 83
Courtesy of D. Mulvihill
Critique: 83
Abdominal radiograph demonstrating the "double-bubble." In this case, the finding results from
duodenal atresia. The appearance is due to a distended, gas-filled stomach (right arrow) and
proximal duodenum (left arrow).
Courtesy of D. Mulvihill
Question: 84
A worried mother brings her 18-month-old son to the emergency department because of a rash
that developed today. She reports that he has had a runny nose and temperature to 104ºF
(40°C) for the last 3 days, but the fever resolved yesterday. His immunizations are up to date.
On physical examination, he appears well, is afebrile, and has diffuse 2- to 3-mm erythematous
macules and papules over his trunk (Item Q84).
A. adenovirus
B. Coxsackievirus
C. Epstein-Barr virus
D. human herpesvirus 6
E. parvovirus B19
Question: 84
Courtesy of D. Krowchuk
References:
Leach CT. Roseola (human herpesviruses 6 and 7). In: Behrman RE, Kliegman RM, Jenson
HB, eds. Nelson Textbook of Pediatrics. 17th ed. Philadelphia, Pa: Saunders; 2004:1069-1071
Wolfrey JD, Billica WH, Gulbranson SH, et al. Pediatric exanthems. Clin Fam Pract. 2003;5:557-
588
Critique: 84
Courtesy of D. Krowchuk
Critique: 84
Courtesy of D. Krowchuk
Critique: 84
Patients who have infectious mononucleosis may develop a morbilliform eruption if they receive
amoxicillin or ampicillin.
Courtesy of D. Krowchuk
Critique: 84
Courtesy of D. Krowchuk
Question: 85
A 10-year-old boy who has moved to your practice recently has a history of frequent sore
throats for which he has received antibiotics. His mother is frustrated with the recurrences and
wants his tonsils removed. The child reports a runny nose, mild cough, and abdominal pain.
Findings on physical examination include a temperature of 100.3°F (38°C), conjunctival
erythema, enlarged tonsils with some erythema and exudate, and vesicular lesions on the soft
palate. There is no cervical adenopathy or rash. Results of a rapid streptococcal antigen test
are negative.
A. adenovirus infection
B. Coxsackievirus infection
C. mononucleosis
D. sinusitis
E. streptococcal pharyngitis
References:
American Academy of Pediatrics. Principles of appropriate use for upper respiratory tract
infections. In: Pickering LK, Baker CJ, Long SS, McMillan JA, eds. Red Book: 2006 Report of the
Committee on Infectious Diseases. 27th ed. Elk Grove Village, Ill: American Academy of
Pediatrics; 2006:737-740
Del Mar CB, Glasziou PP, Spinks AB. Antibiotics for sore throat. Cochrane Database Syst Rev.
2006;4:CD000023. Available at:
http://www.mrw.interscience.wiley.com/cochrane/clsysrev/articles/CD000023/frame.html
Hall MC, Kieke B, Belongia EA. Spectrum bias of a rapid antigen detection test for group A beta-
hemolytic streptococcal pharyngitis in a pediatric population. Pediatrics. 2004;114:182-186.
Available at: http://pediatrics.aappublications.org/cgi/content/full/114/1/182
Jaggi P, Shulman ST. Group A streptococcal infections. Pediatr Rev. 2006;27:99-105. Available
at: http://pedsinreview.aappublications.org/cgi/content/full/27/3/99
Critique: 85
Courtesy of M. Rimsza
Critique: 85
Critique: 85
In herpangina caused by Coxsackievirus infection, patients develop vesicles and ulcers in the
posterior pharynx.
Question: 86
A 5-day-old child is brought to the emergency department because he has been difficult to
arouse over the last 6 hours. His parents report that he has not been interested in feeding today
and that he has been breathing rapidly and with a grunting noise. On physical examination, the
infant’s heart rate is 185 beats/min, respiratory rate is 80 breaths/min, and blood pressure is
55/40 mm Hg. A pulse is palpable in the right brachial region, but not in the feet. All of his
extremities are cool and mottled, with a capillary refill of more than 2 seconds.
initiation of the PGE1. Furosemide, a diuretic, has no beneficial role for the infant whose
systemic perfusion is limited by a closing ductus arteriosus. Lumbar puncture and computed
tomography scan of the head are not indicated as initial management.
References:
Friedman AH, Fahey JT. The transition from fetal to neonatal circulation: normal responses and
implications for infants with heart disease. Semin Perinatol. 1993;17:106–121. Abstract available
at:
http://www.ncbi.nlm.nih.gov/entrez/query.fcgi?db=pubmed&cmd=Retrieve&dopt=AbstractPlus&li
st_uids=8327901
Lister G. Poor systemic perfusion and circulatory shock. In: Rudolph CD, Rudolph AM, eds.
Rudolph’s Pediatrics. 21st ed. New York, NY: McGraw-Hill Medical Publishing Division; 2003:285-
292
Rudolph AM. The fetal circulation and its adjustments after birth. In: Moller JH, Hoffman JIE, eds.
Pediatric Cardiovascular Medicine. Philadelphia, Pa: Churchill Livingstone; 2000:60-64
Question: 87
A 3-month-old infant presents to the emergency department with fussiness and decreased
alertness. During triage, he experiences a seizure. Physical examination reveals somnolence
and a bulging fontanelle. Emergent head computed tomography scan documents acute and
chronic subdural hematomas (Item Q87).
Of the following, the procedure that is MOST likely to reveal the cause of these findings is
A. cerebral angiography
B. electroencephalography
C. lumbar puncture
E. retinal examination
Question: 87
Courtesy of D. Krowchuk
References:
Johnson CF. Abuse and neglect of children. In: Behrman RE, Kliegman RM, Jenson HB, eds.
Nelson Textbook of Pediatrics. 17th ed. Philadelphia, Pa: Saunders; 2004:121-131
Kaplan R. Shaken baby syndrome (shaken-impact syndrome). In: Singer HS, Kossoff EH,
Hartman AL, Crawford TO, eds. Treatment of Pediatric Neurologic Disorders. Boca Raton, Fla:
Taylor & Francis Group; 2005:329-334
Kemp AM. Investigating subdural haemorrhage in infants. Arch Dis Child. 2002;86:98-102.
Abstract available at:
http://www.ncbi.nlm.nih.gov/entrez/query.fcgi?db=pubmed&cmd=Retrieve&dopt=AbstractPlus&li
st_uids=11827902
Levine LM. Pediatric ocular trauma and shaken infant syndrome. Pediatr Clin North Am.
2003;50:137-148. Abstract available at:
http://www.ncbi.nlm.nih.gov/entrez/query.fcgi?db=pubmed&cmd=Retrieve&dopt=AbstractPlus&li
st_uids=12713109
Critique: 87
Question: 88
You are treating a 16-year-old girl who has a history of generalized tonic-clonic seizures, the
most recent of which occurred 6 months ago. She is followed by a neurologist who is treating
her with phenylhydantoin with good results. During a routine physical examination, she confides
that she is sexually active, and she requests a form of birth control. After some discussion, she
asks to be started on oral contraceptive pills (OCPs).
Of the following, the MOST accurate statement about oral contraceptive use in this scenario is
that
C. the girl can be weaned off her antiepileptic medication at this time
E. the OCP estrogen dose should be higher than that usually prescribed
References:
Crawford P. Best practice guidelines for the management of women with epilepsy. Epilepsia.
2005;46(suppl):117-124. Abstract available at:
http://www.ncbi.nlm.nih.gov/entrez/query.fcgi?db=pubmed&cmd=Retrieve&dopt=AbstractPlus&li
st_uids=16302885
Hatcher RA, Trussell J, Stewart FH, et al, eds. Contraceptive Technology. 18th ed. New York,
NY: Ardent Media, Inc; 2004
O’Brien MD, Guillebaud J. Critical review: contraception for women with epilepsy. Epilepsia.
2006;47:1419-1422. Abstract available at:
http://www.ncbi.nlm.nih.gov/entrez/query.fcgi?db=pubmed&cmd=Retrieve&dopt=AbstractPlus&li
st_uids=16981856
Rimsza ME. Counseling the adolescent about contraception. Pediatr Rev. 2003;24:162-170.
Available at: http://pedsinreview.aappublications.org/cgi/content/full/24/5/162
Question: 89
A 17-year-old boy comes to your office from a homeless shelter with complaints of a penile
discharge. He denies pain with urination, skin rashes, or joint pain. He claims to use condoms
most of the time. On physical examination, he is afebrile and has no skin rashes or testicular or
epididymal tenderness or swelling, but he has copious purulent urethral discharge. Results of
testing for sexually transmitted infections will not be available for 2 days.
B. azithromycin 1 g orally in a single dose plus cefixime 400 mg orally in a single dose
D. doxycycline 100 mg orally BID for 7 days plus clindamycin 300 mg twice a day for 7 days
References:
Centers for Disease Control and Prevention. Sexually transmitted diseases treatment guidelines,
2006. MMWR Recomm Rep. 2006;55(RR-11):1-94. Available at:
http://www.cdc.gov/mmwr/preview/mmwrhtml/rr5511a1.htm
Centers for Disease Control and Prevention. Updated recommended treatment regimens for
gonococcal infections and associated conditions - United States, April 2007. Available at:
http://www.cdc.gov/std/treatment/2006/updated-regimens.htm
Question: 90
You are examining a newborn and discover that her genitalia appear abnormal. There is an
enlarged clitoris (1.5 cm in length) (Item Q90) and a small amount of posterior labial fusion.
Amniocentesis was performed during pregnancy, and the karyotype was 46,XX.
A. danazol
B. ethinyl estradiol
C. fluorocarbons
D. organophosphate pesticides
E. progesterone in oil
Question: 90
Courtesy of M. Rimsza
References:
Bird S. Organophosphate and carbamate toxicity. UpToDate Online 14.3. Available for
subscription at:
http://www.utdol.com/utd/content/topic.do?topicKey=ad_tox/9425&type=A&selectedTitle=1~3
Brumskill PJ. The effects of fetal exposure to danazol. Br J Obstet Gynaecol. 1992;99:212-215.
Abstract available at:
http://www.ncbi.nlm.nih.gov/entrez/query.fcgi?db=pubmed&cmd=Retrieve&dopt=AbstractPlus&li
st_uids=1606119
Fridey JL. Oxygen carriers as alternatives to red cell transfusion. UpToDate Online 14.3.
Available for subscription at:
http://www.utdol.com/utd/content/topic.do?topicKey=transfus/11560&type=A&selectedTitle=1~8
Question: 91
You are seeing a 9-year-old boy who is new to your practice for a health supervision visit. He is
a sociable child who talks to you about his new school. You notice that he has repeated facial
movements, eye blinking, and throat clearing. When you mention this to his parents, they admit
to noting these repetitive mannerisms for more than 1 year, which they have attributed to a
nervous habit. They ask you what these mannerisms may represent.
E. vocal and motor tics are most likely due to Tourette syndrome
References:
Boris NW, Dalton R. Habit disorders. In: Behrman RE, Kliegman RM, Jenson HB, eds. Nelson
Textbook of Pediatrics. 17th ed. Philadelphia, Pa: Saunders; 2004:80-81
Disorders usually first diagnosed in infancy, childhood or adolescence. In: Diagnostic and
Statistical Manual of Mental Disorders. 4th ed. Text Revision. Washington, DC: American
Psychiatric Association; 1994:39-134
Johnston MV. Movement disorders. In: Behrman RE, Kliegman RM, Jenson HB, eds. Nelson
Textbook of Pediatrics. 17th ed. Philadelphia, Pa: Saunders; 2004:2019-2022
Pleasure D, De Vivo DC. The nervous system. In: Rudolph CD, Rudolph AM, eds. Rudolph’s
Pediatrics. 21st ed. New York, NY: McGraw-Hill Medical Publishing Division; 2003:2165-2350
Schlaggar BL, Mink JW. Movement disorders in children. Pediatr Rev. 2003;24:39-51. Available
at: http://pedsinreview.aappublications.org/cgi/content/full/24/2/39
Critique: 91
Question: 92
A 13-year-old boy presents to the emergency department with a 3-day history of severe sore
throat and fever. He is having trouble swallowing due to pain. His past medical history is
unremarkable. On physical examination, he has a temperature of 102.6°F (39.3°C) and a large
left tonsil, with swelling of the left soft palate and deviation of the uvula to the right (Item Q92).
You suspect a peritonsillar abscess.
A. ampicillin
B. ceftriaxone
C. clindamycin
D. penicillin V
E. vancomycin
Question: 92
References:
Critique: 92
Peritonsillar abscess: A large left tonsil and swelling of the peritonsillar tissue that displaces the
uvula to the right.
Question: 93
You are evaluating a 5-month-old male adopted from a Somalian orphanage. According to his
medical records, his biologic mother was treated for malaria during her last month of pregnancy
and was suspected of having syphilis, based on a rapid plasma reagin (RPR) test result of 1:4,
for which she received a course of treatment with an unknown oral antibiotic. Prior to arriving in
the United States, the baby was found to have an RPR result of 1:2. Findings on physical
examination of the infant today are completely normal.
B. perform polymerase chain reaction for treponemal DNA and repeat RPR
C. repeat RPR
References:
American Academy of Pediatrics. Syphilis. In: Pickering LK, Baker CJ, Long SS, McMillan JA,
eds. Red Book: 2006 Report of the Committee on Infectious Diseases. 27th ed. Elk Grove
Village, Ill: American Academy of Pediatrics; 2006:631-644
Larson SA, Pope V, Johnson RE, Kennedy EJ Jr, eds. A Manual of Tests for Syphilis. 9th ed.
Washington, DC: American Public Health Association; 1998
Sanchez P, Gutman LT. Syphilis. In: Feign RD, Cherry JD, Demmler GJ, Kaplan SL, eds.
Textbook of Pediatric Infectious Diseases. 5th ed. Philadelphia, Pa: Saunders; 2004:1724-1742
Tramont EC. Treponema pallidum (syphilis). In: Mandell GL, Bennett JE, Dolin R, eds. Mandell,
Douglas and Bennett’s Principles and Practice of Infectious Diseases. 6th ed. Philadelphia, Pa:
Elsevier Churchill Livingstone; 2005:2768-2784
Question: 94
A 6-year-old boy presents with cola-colored urine. His mother reports that he had a sore throat
10 days ago. On physical examination, his blood pressure is 136/88 mm Hg, and he has mild
swelling of the face and lower extremities.
References:
Brouhard BH, Travis LB. Acute postinfectious glomerulonephritis. In: Edelmann CM Jr, ed.
Pediatric Kidney Disease. 2nd ed. Boston, Mass: Little, Brown and Company; 1992:1199-1221
Sulyok E. Acute proliferative glomerulonephritis. In: Avner ED, Harmon WE, Niaudet P, eds.
Pediatric Nephrology. 5th ed. Philadelphia, Pa: Lippincott William & Wilkins; 2004:601-613
Question: 95
A 5-month-old female infant presents with a 4-day history of increasing lethargy, drooling,
constipation, and a weak cry. The parents report that the infant had been very healthy until this
past week; she had no preceding sick contacts, viral symptoms, or fever. The infant is bottle fed
and over the past month has been introduced to various home-pureed fresh fruits and
vegetables, although the parents deny giving her any honey or homemade canned products.
They currently have two dogs and a cat and live in the northwestern United States in a new
subdivision that is still under construction. On physical examination, the infant is afebrile but has
symmetrically decreased tone, absent deep tendon reflexes, poor muscle strength, and a weak
cry.
A. botulism
B. diphtheria
C. lead poisoning
D. meningitis
E. tetanus
References:
Schechter R, Arnon SS. Botulism (Clostridium botulinum). In: Behrman RE, Kliegman RM,
Jenson HB, eds. Nelson Textbook of Pediatrics. 17th ed. Philadelphia, Pa: Saunders; 2004:947-
950
Question: 96
The mother of one of your patients calls you because she has found an engorged tick on her
daughter’s scalp. She wants to know how to remove it safely.
References:
Howard J, Loiselle J. A clinician’s guide to safe and effective tick removal. Contemp Pediatr. May
2006. Available at:
http://www.contemporarypediatrics.com/contpeds/article/articleDetail.jsp?id=325403&searchStri
ng=tick
Krakowski AC, Golden DBK. Hymenoptera stings: a practical guide to prevention and
management. Contemp Pediatr. August 2006. Available at:
http://www.contemporarypediatrics.com/contpeds/article/articleDetail.jsp?id=364742&pageID=1
Needham GR. Evaluation of five popular methods for tick removal. Pediatrics. 1985;75:997-
1002. Available at: http://pediatrics.aappublications.org/cgi/reprint/75/6/997
Critique: 96
Proper tick removal: Using blunt forceps, grasp the tick as close to the skin as possible (top).
Use traction perpendicular to the skin surface to remove the tick (bottom).
Question: 97
A 14-year-old boy requests treatment for his acne. He is using no medications and has no
known drug allergies. Physical examination of the face reveals a few small inflammatory papules
and numerous blackheads and whiteheads; there is no scarring (Item Q97). No acne lesions
are present on the chest and back.
D. clindamycin topically
E. tretinoin topically
Question: 97
Courtesy of D. Krowchuk
References:
James WD. Clinical practice. Acne. N Engl J Med. 2005;352:1463-1472. Abstract available at:
http://www.ncbi.nlm.nih.gov/entrez/query.fcgi?db=pubmed&cmd=Retrieve&dopt=AbstractPlus&li
st_uids=15814882
Krowchuk DP. Managing adolescent acne. A guide for pediatricians. Pediatr Rev. 2005;26:250-
261. Available at: http://pedsinreview.aappublications.org/cgi/content/full/26/7/250
Krowchuk DP, Mancini AJ, eds. Acne vulgaris. In: Pediatric Dermatology. A Quick Reference
Guide. Elk Grove Village, Ill: American Academy of Pediatrics; 2007:41-51
Weston WL, Lane AT, Morelli JG. Acne. In: Color Textbook of Pediatric Dermatology. 3rd ed. St.
Louis, Mo: Mosby; 2002:89:15-25
Zaenglein AL, Thiboudot DM. Expert committee recommendations for acne management.
Pediatrics. 2006;118:1188-1199. Available at:
http://pediatrics.aappublications.org/cgi/content/full/118/3/1188
Critique: 97
Courtesy of D. Krowchuk
Critique: 97
Blackheads and whiteheads (skin-colored or white papules) are evidence of follicular obstruction
in acne.
Courtesy of D. Krowchuk
Critique: 97
Mixed facial acne that includes a few inflammatory lesions and multiple blackheads and
whiteheads.
Courtesy of D. Krowchuk
Question: 98
A 16-year-old girl who has juvenile idiopathic arthritis (juvenile rheumatoid arthritis) presents with
a history of chronic anemia. Her current medications include ibuprofen, methotrexate, and folic
acid. Physical examination demonstrates tenderness to palpation of the upper abdomen and a
left knee effusion. The stool is positive for occult blood. Results of laboratory studies include a
hematocrit of 28% (0.28), mean cell volume of 72 fL, reticulocyte count of 1%, white blood cell
count of 4.2x103/mcL (4.2x109/L), serum iron concentration of 14 mcg/dL (2.5 mcmol/L), and
total iron binding capacity of 400 mcg/dL (71.6 mcmol/L). Transaminases, bilirubin, amylase, and
lipase values are normal. You initiate iron therapy.
A. abdominal ultrasonography
C. mesenteric angiography
References:
Wilcox CM, Ladabaum U. A patient with high risk of gastrointestinal bleeding requiring
nonsteroidal anti-inflammatory drugs. Clin Gastroenterol Hepatol. 2006;4:1090-1093
Question: 99
A 10-day-old infant who was born at 28 weeks’ gestation has been weaned off of the ventilator
and is receiving supplemental oxygen at an Fio2 of 0.30. He experiences episodic apnea 10 to
12 times daily and has corresponding bradycardia and oxygen desaturation, as measured by
pulse oximetry (Item Q99). Some of the episodes require tactile stimulation and an increase in
supplemental oxygen for resolution. Physical examination reveals no baseline tachypnea,
tachycardia, or increased work of breathing. No heart murmur is appreciated, and he is
normotensive. He appears acyanotic and well perfused, with a recent hematocrit of 35% (0.35).
Results of head ultrasonography performed 3 days ago were normal.
C. intravenous ampicillin
Question: 99
Apnea, as described for the infant in the vignette: Respiration ceases for approximately 30
seconds and is associated with bradycardia and a decline in oxygen saturation.
Courtesy of B. Carter
References:
Baird TM, Martin RJ, Abu-Shaweesh JM. Clinical associations, treatment, and outcome of apnea
of prematurity. NeoReviews. 2002;3:e66-e70. Available at:
http://neoreviews.aappublications.org/cgi/content/full/3/4/e66
Herzlinger RA. Apnea. In: McMillan JA, Feigin RD, DeAngelis C, Jones MD, eds.
Oski's Pediatrics: Principles & Practice. 4th ed. Philadelphia, Pa: Lippincott Williams
& Wilkins; 2006:318-320
Martin RJ, Abu-Shaweesh JM, Baird TM. Pathophysiologic mechanisms underlying apnea of
prematurity. NeoReviews. 2002;3:e59-e65. Available at:
http://neoreviews.aappublications.org/cgi/content/full/3/4/e59
Schmidt B, Roberts RS, Davis P, et al; Caffeine for Apnea of Prematurity Trial Group. Caffeine
therapy for apnea of prematurity. N Engl J Med. 2006;354:2112-2121. Abstract available at:
http://www.ncbi.nlm.nih.gov/entrez/query.fcgi?db=pubmed&cmd=Retrieve&dopt=AbstractPlus&li
st_uids=16707748
Critique: 99
Reprinted with permission from Baird TM, Martin RJ, Abu-Shaweesh JM. Clinical associations,
treatment, and outcome of apnea of prematurity. NeoReviews. 2002;3:e66-e70
Critique: 99
Courtesy of B. Carter
Question: 100
You are evaluating a 10-year-old boy who just moved to the United States from Africa. He
reports fever and abdominal pain of 2 weeks’ duration. Physical examination reveals an ill-
appearing boy who has a temperature of 103ºF (39.5°C), a respiratory rate of 40 breaths/min,
hepatomegaly, and tenderness of the right upper quadrant. Abdominal ultrasonography shows a
liver mass consistent with an abscess (Item Q100).
A. Ascaris lumbricoides
B. Entamoeba histolytica
C. Strongyloides stercoralis
D. Taenia solium
E. Wuchereria bancrofti
Question: 100
Transverse ultrasonographic image of the liver shows a large anechoic lesion in the right lobe
consistent with an abscess.
Courtesy of D. Mulvihill
References:
American Academy of Pediatrics. Amebiasis. In: Pickering LK, Baker CJ, Long SS, McMillan JA,
eds. Red Book: 2006 Report of the Committee on Infectious Diseases. 27th ed. Elk Grove
Village, Ill: American Academy of Pediatrics; 2006:204-206
American Academy of Pediatrics. Ascaris lumbricoides infections. In: Pickering LK, Baker CJ,
Long SS, McMillan JA, eds. Red Book: 2006 Report of the Committee on Infectious Diseases.
26th ed. Elk Grove Village, Ill: American Academy of Pediatrics; 2006:218-219
Critique: 100
Transverse ultrasonographic image of the liver shows a large anechoic lesion in the right lobe
consistent with an amebic abscess.
Courtesy of D. Mulvihill
Critique: 100
Courtesy of D. Gilbert
Question: 101
A mother brings in her 6-month-old child for a health supervision visit. She explains that the
family just moved from a rural area where they had obtained their water from a private well. In
the past, she needed to give supplemental fluoride to her children beginning at age 6 months and
wonders if this is necessary for this child.
C. fluoride 0.25 mg/d supplementation should begin at age 6 months if there is less than 0.03
ppm in community water
E. supplementation may stop when the child’s deciduous teeth begin to fall out
It is well established that the use of fluoride supplements in the diet and dental hygiene of children
offers protection against the development of dental caries in both deciduous and permanent teeth.
However, the challenge and controversy of providing fluoride in water without contributing to
fluorosis and the difficulty with access to dental care for many children contribute to suboptimal
fluoride use. Underuse of fluoride may contribute to the formation of dental caries in some
children who may or may not have risk factors for nursing bottle caries and a family history of
dental caries. Overuse of fluoride can result in “fluorosis,” (Item C101A) which includes white
spots, pitting, or staining of teeth.
Decisions about fluoride use for children should be based on a child’s risk for caries.
Individual and family risk factors include poverty, lack of access to dental care, non-white races,
recent immigration, family history of dental caries, and residence in a community that does not
have fluoridated water.
The Task Force on Community Preventive Service concluded that fluoridated water reduced
tooth decay by 30% to 50% relative to what would be expected in those not consuming
fluoridated water. Information about the fluoride content of the community water supply in the area
to which the family in the vignette has moved should be available from the local health or water
department. Fluoride supplementation may be prescribed accordingly (Item C101B).
Supplemental fluoride may be discontinued when the child reaches an age when fluoride
varnish may be applied to the teeth (by a medical or dental professional) and when the child may
use fluoridated toothpaste.
Guidelines from the American Academy of Pediatrics, published in 1995 and updated in 2002,
lowered the recommended fluoride supplement dose for children younger than 6 years of age and
eliminated supplementation for infants younger than 6 months of age. These changes were made
in recognition of the risk of fluorosis related to excess fluoride exposure during permanent tooth
development.
The period of highest susceptibility to fluorosis for permanent teeth is estimated to be between
15 and 24 months of age for boys and 21 and 30 months of age for girls. This is a good age for
fluoride varnish application. Early toothbrushing (ie, before 2 to 3 years of age) with fluoride paste
may lead to an unacceptable risk of fluorosis, especially in combination with the use of fluoridated
water and fluoride varnish.
Children living in families who use exclusively bottled water for meal preparation and mixing
formula and juice for their children should receive fluoride supplementation because most bottled
water does not contain fluoride. However, the Centers for Disease Control and Prevention (CDC)
have raised concern that the use of fluoridated water to prepare infant formula may be associated
with an increased risk of fluorosis. The CDC has suggested that parents of infants fed primarily
formula from concentrate or powder who are concerned about fluorosis consider the use of
bottled water for formula preparation.
References:
Centers for Disease Control and Prevention. Water Fluoridation. 2006. Available at:
http://www.cdc.gov/fluoridation/safety/infant_formula.htm
Gleiner S, Lewis CW. Question from the clinician: fluoride supplementation and dental caries.
Pediatr Rev. 2002;23:186-187. Available at:
http://pedsinreview.aappublications.org/cgi/content/full/23/5/186
Lewis C, Lynch H, Richardson L. Fluoride varnish use in primary care: what do providers think?
Pediatrics. 2005;115:e69-e76. Available at:
http://pediatrics.aappublications.org/cgi/content/full/115/1/e69
Truman BI, Gooch FH, Sulemana I, et al. Reviews of evidence on interventions to prevent dental
caries, oral and pharyngeal cancers, and sports-related craniofacial injuries. Am J Prev Med.
2002;23(1 suppl):21—54. Abstract available at:
http://www.ncbi.nlm.nih.gov/entrez/query.fcgi?db=pubmed&cmd=Retrieve&dopt=AbstractPlus&list
_uids=12091093
Yeung CA, Hitchings JL, Macfarlane TV, Threlfall AG, Tickle M, Glenny AM. Fluoridated milk for
preventing dental caries. Cochrane Database Syst Rev. 2005;3:CD003876. Available at:
http://www.mrw.interscience.wiley.com/cochrane/clsysrev/articles/CD003876/frame.html
Critique: 101
Fluorosis (mottled enamel) is characterized by white-to-brown patches on the teeth. In this child it
is most evident on the maxillary central incisors.
Courtesy of M. Rimsza
Critique: 101
Question: 102
A 6-month-old previously healthy girl is brought to your office because she has not been eating
well today. The mother reports that the baby is interested in taking the bottle, but stops feeding
within 1 minute and seems to have trouble breathing. She is irritable but consolable in her
mother’s arms. She is pale, afebrile, and has a respiratory rate of 70 breaths/min. Her heart rate
is too fast to count, she has palpable pulses in all extremities, and her perfusion is fair, with a
capillary refill time of 2 to 3 seconds.
Of the following, the MOST likely additional finding expected in this child is
B. conjunctivitis
C. hepatomegaly
D. nuchal rigidity
References:
Doniger SJ, Sarieff GQ. Pediatric dysrhythmias. Pediatr Clin North Am. 2006;53:85-105.
http://www.ncbi.nlm.nih.gov/entrez/query.fcgi?db=pubmed&cmd=Retrieve&dopt=AbstractPlus&li
st_uids=16487786
Kaltman J, Shah M. Evaluation of the child with an arrhythmia. Pediatr Clin North Am.
2004;51:1537-1551.
http://www.ncbi.nlm.nih.gov/entrez/query.fcgi?db=pubmed&cmd=Retrieve&dopt=AbstractPlus&li
st_uids=15561172
Talner NS, McGovern JJ, Carboni MP. Congestive heart failure. In: Moller JH, Hoffman JIE, eds.
Pediatric Cardiovascular Medicine. Philadelphia, Pa: Churchill Livingstone; 2000:817-832
Vetter V. Arrhythmias. In: Moller JH, Hoffman JIE, eds. Pediatric Cardiovascular Medicine.
Philadelphia, Pa: Churchill Livingstone; 2000:833-884
Question: 103
A 10-year-old boy in whom you diagnosed attention-deficit/hyperactivity disorder (ADHD) has
been receiving stimulant medication daily since he was 7 years old. About 2 years ago, he
developed persistent, repetitive throat clearing, and subsequently he has had waxing and
waning motor and vocal tics that include eye rolling, grimacing, head bobbing, sniffing, and
humming. For the past 2 months, his tics have worsened; he now has a loud squeak and head-
jerking tic. His mother decided to stop his stimulant medication last week, fearing it might be
causing his tics to worsen. Now he is much more hyperactive and is having more behavioral
and attentional difficulties at school and home. His tics have not improved.
References:
Bloch MH, Peterson BS, Scahill L, Otka J, et al. Adulthood outcome of tic and obsessive-
compulsive symptom severity in children with Tourette syndrome. Arch Pediatr Adolesc Med.
2006;160:65-69. Abstract available at:
http://www.ncbi.nlm.nih.gov/entrez/query.fcgi?db=pubmed&cmd=Retrieve&dopt=AbstractPlus&li
st_uids=16389213
Filloux FM, McMahon WM. Tic disorders. In: Maria BL, ed. Current Management in Child
Gilbert DL. Treatment of children and adolescents with tics and Tourette syndrome. J Child
Neurol. 2006;21:690-700. Abstract available at:
http://www.ncbi.nlm.nih.gov/entrez/query.fcgi?db=pubmed&cmd=Retrieve&dopt=AbstractPlus&li
st_uids=16970870
Leckman JF, Zhang H, Vitale A, et al. Course of tic severity in Tourette syndrome: the first two
decades. Pediatrics. 1998;102:14-19. Available at:
http://pediatrics.aappublications.org/cgi/content/full/102/1/14
Singer HS. Treatment of Tourette syndrome. In: Singer HS, Kossoff EH, Hartman AL, Crawford
TO, eds. Treatment of Pediatric Neurologic Disorders. Boca Raton, Fla: Taylor & Francis
Group; 2005:125-132
Question: 104
A 7-year-old patient who has Down syndrome is brought to the clinic by her mother, who is
worried that the child has an increasingly abnormal gait and worsening clumsiness. At age 3
years, she was screened for cervical instability with flexion and extension cervical spine films,
which showed normal results. On physical examination today, you note that she has an
unsteady gait, and she has brisk deep tendon reflexes diffusely. These findings represent a
significant change from 9 months ago when your neurologic examination showed only slightly
diminished tone.
Of the following, the MOST likely cause of these symptoms and signs in a child who has Down
syndrome is
A. cerebellar medulloblastoma
B. Chiari I malformation
References:
American Academy of Pediatrics Committee on Genetics. Health supervision for children with
Down syndrome. Pediatrics. 2001;107:442-449. Available at:
http://pediatrics.aappublications.org/cgi/content/full/107/2/442
American Academy of Pediatrics Committee on Sports Medicine and Fitness. Medical conditions
affecting sports participation. Pediatrics. 2001;107:1205-1209. Available at:
http://pediatrics.aappublications.org/cgi/content/full/107/5/1205
Cohen WI, ed. Health care guidelines for individuals with Down syndrome: 1999 revision. Down
Syndrome Quarterly. 1999;4(3).
Pueschel SM. Should children with Down syndrome be screened for atlantoaxial instability? Arch
Pediatr Adolesc Med. 1998;152:123-125. Abstract available at:
http://www.ncbi.nlm.nih.gov/entrez/query.fcgi?db=pubmed&cmd=Retrieve&dopt=AbstractPlus&li
st_uids=9491036
Question: 105
A 17-year-old young woman comes to your clinic after having been diagnosed with pelvic
inflammatory disease the preceding day. She was prescribed doxycycline 100 mg orally twice a
day for 14 days and given ceftriaxone 250 mg intramuscularly in a single dose at the time of
diagnosis. Since this visit, she vomited the doxycycline, has been unable to retain any fluids, has
developed a fever, and has had worsening abdominal pain. External genital examination findings
are normal.
A. administer a repeat dose of the oral doxycycline and send the patient home
B. administer benzathine penicillin G 2.4 million units intramuscularly in a single dose and send
the patient home
C. change the oral medication to azithromycin 1 g given in a single dose and send the patient
home
D. hospitalize the patient and begin intravenous cefotetan 2 g plus doxycycline 100 mg every 12
hours
E. hospitalize the patient for observation and add acyclovir 400 mg orally TID for 7 to 10 days
References:
American Academy of Pediatrics. Pelvic inflammatory disease. In: Pickering LK, Baker CJ, Long
SS, McMillan JA, eds. Red Book: 2006 Report of the Committee on Infectious Diseases. 27th ed.
Elk Grove Village, Ill: American Academy of Pediatrics; 2006:493-498
Centers for Disease Control and Prevention. Sexually transmitted diseases treatment guidelines,
2006. MMWR Recomm Rep. 2006;55(RR-11):1-94. Available at:
http://www.cdc.gov/mmwr/preview/mmwrhtml/rr5511a1.htm
Question: 106
During the annual health supervision visit of a 9-year-old boy, you note that he has grown very
little in the past year (Item Q106). He has been otherwise well. On physical examination, he has
slightly increased abdominal fat and decreased muscle mass.
A. celiac disease
C. craniopharyngioma
D. hypochondroplasia
E. renal insufficiency
Question: 106
Courtesy of L. Levitsky
References:
Recht LD, Marcus KJ. Craniopharyngioma. UpToDate Online 14.3. Available for subscription at:
http://www.utdol.com/utd/content/topic.do?topicKey=brain_ca/2447&type=A&selectedTitle=1~14
Richmond EJ, Rogol AD. Diagnosis of growth hormone deficiency in children. UpToDate Online
14.3. Available for subscription at:
http://www.utdol.com/utd/content/topic.do?topicKey=pediendo/5869&type=P&selectedTitle=20~3
7
Rogol AD. Causes of short stature. UpToDate Online 14.3. Available for subscription at:
http://www.utdol.com/utd/content/topic.do?topicKey=pediendo/2279&type=A&selectedTitle=1~61
van Rijn JC, Grote FK, Oostdijk W, Wit JM. Short stature and the probability of celiac disease, in
the absence of gastrointestinal symptoms. Arch Dis Child. 2004;89:882-883. Available at:
http://adc.bmj.com/cgi/content/full/89/9/882
Critique: 106
Growth curve in a child who has celiac disease documenting slowing of linear growth and weight
loss.
Courtesy of L. Levitsky
Question: 107
The parents of 9-month-old twins ask you if they should be concerned about the vast behavioral
differences between the children. They explain that the boy whimpers when he is hungry, but the
girl has a vigorous scream. The boy plays quietly while his diaper is changed, but the girl is
constantly moving. Finally, the girl tends to display her emotions with strong intensity, while the
boy is more easygoing.
B. the behaviors the twins display are due to their different temperaments
C. the girl’s behavior is indicative of a developmental disorder that should be monitored closely
References:
Boyle MP. Evolving parenthood: a developmental perspective. In: Levine MD, Carey WB,
Crocker AC, eds. Developmental-Behavioral Pediatrics. 3rd ed. Philadelphia, Pa: WB Saunders
Co; 1999:80-88
Gurian A. Parenting styles/children’s temperaments: the match. NYU Child Study Center. 2007.
Available at:
http://www.aboutourkids.org/articles/parentingstyles.html
Question: 108
You are taking your family on a safari in Africa. Everyone has received hepatitis A and typhoid
immunization. Your husband is worried about “catching malaria” and asks how you are planning
to prevent this.
A. atovaquone
B. chloroquine
C. dapsone
D. mefloquine
E. no prophylactic medications
References:
American Academy of Pediatrics. Malaria. In: Pickering LK, Baker, CJ, Long SS, McMillon JA,
eds. Red Book: 2006 Report of the Committee on Infectious Diseases. 27th ed. Elk Grove
Village, Ill: American Academy of Pediatrics; 2006:435-441
Maples HD, Schutze GE. Consultation with the specialist: malaria prophylaxis in children. Pediatr
Rev. 2006;27:346-350. Available at:
http://pedsinreview.aappublications.org/cgi/content/full/27/9/346
Question: 109
A 14-year-old previously healthy girl presents with a 3-day history of knee and ankle pain and a
maculopapular lacy rash (Item Q109A) that is most prominent on her face (Item Q109B), trunk,
and thighs. Ten days prior to the development of the rash and the arthralgias, she had a
temperature of 102°F (38.9°C), malaise, and a headache that resolved after 2 days.
A. cell culture
Question: 109
Courtesy of D. Krowchuk
Question: 109
Courtesy of D. Krowchuk
References:
American Academy of Pediatrics. Parvovirus B19 (erythema infectiosum, Fifth disease). In:
Pickering LK, Baker CK, Long SS, McMillan JA, eds. Red Book: 2006 Report of the Committee
on Infectious Diseases. 27th ed. Elk Grove Village, Ill: American Academy of Pediatrics;
2006:484-487
Gill VJ, Fedorko DP, Witebsky FG. The clinician and the microbiology laboratory. In: Mandell GL,
Bennett JE, Dolin R, eds. Mandell, Douglas and Bennett’s Principles and Practice of Infectious
Diseases. 6th ed. Philadelphia, Pa: Elsevier Churchill Livingstone; 2005:203-241
Question: 110
An 8-year-old boy presents with tingling in his lower extremities. He had been treated surgically
for osteogenic sarcoma and remains on a chemotherapeutic regimen that includes cisplatin. He
reports intermittent vomiting and loose stools during his chemotherapy treatment. Physical
examination shows no other findings of note. Among the results of laboratory evaluation are a
magnesium value of 0.9 mg/dL (0.37 mmol/L) and a potassium value of 2.7 mEq/L (2.7 mmol/L).
Of the following, the MOST likely explanation for this boy’s electrolyte imbalance is
B. dietary deficiency
References:
Brock PR, Koliouskas DE, Barratt TM, Yeomans E, Pritchard J. Partial reversibility of cisplatin
nephrotoxicity in children. J Pediatr. 1991;118:531-534. Abstract available at:
http://www.ncbi.nlm.nih.gov/entrez/query.fcgi?db=pubmed&cmd=Retrieve&dopt=AbstractPlus&li
st_uids=2007926
Chesney RW, Jones DP. Nephrotoxins. In: Avner ED, Harmon WE, Niaudet P, eds. Pediatric
Nephrology. 5th ed. Philadelphia, Pa: Lippincott Williams & Wilkins; 2004:987-1006
Rose BD, Post TW. Hypokalemia. In: Clinical Physiology of Acid-base and Electrolyte Disorders.
5th ed. New York, NY: McGraw-Hill Medical Publishing Division; 2001:836-887
Question: 111
A 4-year-old boy presents with a 2-year history of persistent bilateral nasal congestion. His
parents are worried because at night he snores loudly and has had pauses in his breathing. His
symptoms occur daily and have not improved with the administration of oral decongestants,
nasal corticosteroids, oral antihistamines, or antibiotics. The boy denies ocular pruritus,
sneezing, or rhinorrhea. On physical examination, a low-pitched inspiratory noise is audible, and
there is “cobblestoning” of his posterior pharynx. Findings on the remainder of the physical
examination, including the tonsils, nose (by nasal speculum examination), and neck, are
unremarkable.
A. adenoidal hypertrophy
B. allergic rhinitis
C. choanal atresia
D. chronic sinusitis
References:
Lewis J. Kass LJ. Sleep problems. Pediatr Rev. 2006;27:455-462. Available at:
http://pedsinreview.aappublications.org/cgi/content/full/27/12/455
Matiz A, Roman EA, Adam HM. In brief: apnea. Pediatr Rev. 2003;24:32-34. Available at:
http://pedsinreview.aappublications.org/cgi/content/full/24/1/32
Rosen CL, Kass LJ, Haddad GG. Obstructive sleep apnea and hypoventilation. In: Behrman
RE, Kliegman RM, Jenson HB, eds. Nelson Textbook of Pediatrics. 17th ed. Philadelphia, Pa:
Saunders; 2004:1397-1400
Critique: 111
Critique: 111
Courtesy of D. Mulvihill
Critique: 111
Courtesy of D. Kirse
Critique: 111
Courtesy of M. Boston
Question: 112
A 10-year-old soccer player sustained tibial and fibular fractures of his left leg yesterday during
a soccer game and was placed in a long leg posterior splint in the emergency department. His
mother calls you today because he is complaining of worsening pain in his lower leg over the last
4 hours that has not improved with acetaminophen with codeine and loosening of the splint. In
addition, he says that he no longer can move the toes on his left foot, and the toes look white.
D. lack of tenseness on palpation of the lower extremity rules out the diagnosis
E. measurement of the pressures in the muscle compartments of the lower leg is the diagnostic
test of choice
References:
Kostler W, Strohm PC, Sudkamp NP. Acute compartment syndrome of the limb. Injury.
2005;36:992-998. Abstract available at:
http://www.ncbi.nlm.nih.gov/entrez/query.fcgi?db=pubmed&cmd=Retrieve&dopt=AbstractPlus&li
st_uids=16372396
McQueen MM, Court-Brown CM. Compartment monitoring in tibial fractures. The pressure
threshold for decompressions. J Bone Joint Surg Br. 1996;78:99-104. Abstract available at:
http://www.ncbi.nlm.nih.gov/entrez/query.fcgi?db=pubmed&cmd=Retrieve&dopt=AbstractPlus&li
st_uids=8898137
Mubarak SJ, Hargens AR. Acute compartment syndromes. Surg Clin North Am. 1983;63:539-
565. Abstract available at:
http://www.ncbi.nlm.nih.gov/entrez/query.fcgi?db=pubmed&cmd=Retrieve&dopt=AbstractPlus&li
st_uids=6346542
Critique: 112
Cross-sectional drawing illustrating the muscle compartments of the leg. Increased pressure
within these closed compartments may lead to decreased muscle perfusion and ischemia.
Courtesy of A. Johnson
Question: 113
A 16-year-old girl requests treatment for acne. She has used a nonprescription medication
containing benzoyl peroxide without significant benefit. Physical examination reveals
inflammatory lesions and open and closed comedones on the face (Item Q113) and
inflammatory lesions on the chest and back; there is no scarring. She has no known allergies to
medications.
B. clindamycin topically
D. isotretinoin orally
E. tretinoin topically
Question: 113
Courtesy of D. Krowchuk
References:
James WD. Clinical practice. Acne. N Engl J Med. 2005;352:1463-1472. Abstract available at:
http://www.ncbi.nlm.nih.gov/entrez/query.fcgi?db=pubmed&cmd=Retrieve&dopt=AbstractPlus&li
st_uids=15814882
Krowchuk DP. Managing adolescent acne. A guide for pediatricians. Pediatr Rev. 2005;26:250-
261. Available at: http://pedsinreview.aappublications.org/cgi/content/full/26/7/250
Krowchuk DP, Mancini AJ, eds. Acne vulgaris. In: Pediatric Dermatology. A Quick Reference
Guide. Elk Grove Village, Ill: American Academy of Pediatrics; 2007:41-51
Weston WL, Lane AT, Morelli JG. Acne. In: Color Textbook of Pediatric Dermatology. 3rd ed. St.
Louis, Mo: Mosby; 2002:89:15-25
Zaenglein AL, Thiboudot DM. Expert committee recommendations for acne management.
Pediatrics. 2006;118:1188-1199. Available at:
http://pediatrics.aappublications.org/cgi/content/full/118/3/1188
Critique: 113
Courtesy of D. Krowchuk
Critique: 113
Blackheads and whiteheads (skin-colored or white papules) are evidence of follicular obstruction
in acne.
Courtesy of D. Krowchuk
Critique: 113
Moderate mixed acne, including numerous inflammatory lesions, blackheads, and whiteheads.
Courtesy of D. Krowchuk
Question: 114
A 5-year-old child presents to your office with a history of recurrent rectal prolapse that occurs
at the time of bowel movements. Both the mother and child are very concerned when the rectal
tissue protrudes from the anus, but the prolapse typically resolves without treatment.
A. celiac disease
B. cystic fibrosis
D. functional constipation
E. rectal polyps
References:
Johnson S, Jaksic T. Benign perianal lesions. In: Walker WA, Goulet O, Kleinman RE, Sherman
PM, Shneider BL, Sanderson IR, eds. Pediatric Gastrointestinal Disease: Pathophysiology,
Diagnosis, Management. 4th ed. Hamilton, Ontario, Canada: BC Decker; 2004:598-603
Madiba TE, BaigMK, Wexner SD. Surgical management of rectal prolapse. Arch Surg.
2005;140:63-73. Abstract available at:
http://www.ncbi.nlm.nih.gov/entrez/query.fcgi?db=pubmed&cmd=Retrieve&dopt=AbstractPlus&li
st_uids=15655208
Zempsky WT, Rosenstein BJ. The cause of rectal prolapse in children. Am J Dis Child.
1988;142:338-339. Abstract available at:
http://www.ncbi.nlm.nih.gov/entrez/query.fcgi?db=pubmed&cmd=Retrieve&dopt=AbstractPlus&li
st_uids=3344723
Question: 115
You are planning the discharge of an extremely low-birthweight infant from the neonatal intensive
care unit (NICU) who requires home oxygen therapy for chronic lung disease. She also requires
anticonvulsant therapy for a seizure disorder that is related to her having a grade III
intraventricular hemorrhage with posthemorrhagic hydrocephalus and a ventriculoperitoneal
shunt.
Of the following, a TRUE statement about the psychosocial effects of home care and equipment
use by her parents is that
A. all parents are challenged by the stressors of home care and equipment use for infants who
were in the NICU
B. mothers are more challenged than fathers about the provision of home care and equipment
use
C. mothers who have visited regularly in the NICU have no problems with home medical
equipment
D. parents who are health-care professionals can be expected to provide optimal care of their
infant at home
E. the psychosocial stressors of home care and equipment use by parents require a psychiatric
consultation
References:
Carnevale FA, Alexander E, Davis M, Rennick J, Troini R. Daily living with distress and
enrichment: the moral experience of families with ventilator-assisted children at home.
Pediatrics. 2006;117:e48-e60. Available at:
http://pediatrics.aappublications.org/cgi/content/full/117/1/e48
Melnyk BM, Alpert-Gillis L, Feinstein NF, et al. Creating opportunities for parent empowerment:
program effects on the mental health/coping outcomes of critically ill young children and their
mothers. Pediatrics. 2004;113:e597-e607. Available at:
http://pediatrics.aappublications.org/cgi/content/full/113/6/e597
Melnyk BM, Feinstein NF, Alpert-Gillis L, et al. Reducing premature infants’ length of stay and
improving parents’ mental health outcomes with the Creating Opportunities for Parent
Empowerment (COPE) neonatal intensive care unit program: a randomized, controlled trial.
Pediatrics. 2006;118:e1414-e1427. Available at:
http://pediatrics.aappublications.org/cgi/content/full/118/5/e1414
Raina P, O’Donnell M, Rosenbaum P, et al. The health and well-being of caregivers of children
with cerebral palsy. Pediatrics. 2005;115:e626-e636. Available at:
http://pediatrics.aappublications.org/cgi/content/full/115/6/e626
Siegel R, Gardner SL, Merenstein GB. Families in crisis: theoretical and practical considerations.
In: Merenstein GB, Gardner SL, eds. Handbook of Neonatal Intensive Care. 6th ed. St.Louis,
Mo: Mosby Elsevier; 2006:863-913
Question: 116
The parents of a 6-month-old boy call you in the middle of the night because he is coughing and
has nasal congestion. You had diagnosed a viral upper respiratory tract infection when you saw
him earlier today. The parents are frustrated that the cough is persistent and request medicine
for their boy so they can sleep.
A. chlorpheniramine/pseudoephedrine combination
B. codeine
C. dextromethorphan
D. guaifenesin
References:
Infant deaths associated with cough and cold medications— two states, 2005. MMWR Morbid
Mortal Wkly Rep. 2007;56:1-4. Available at: http://www.cdc.gov/mmwr/PDF/wk/mm5601.pdf
Kelly LF. Pediatric cough and cold preparations. Pediatr Rev. 2004;25:115-123. Available at:
http://pedsinreview.aappublications.org/cgi/content/full/25/4/115
Paul IM, Yoder KE, Crowell KR, et al. Effect of dextromethorphan, diphenhydramine, and placebo
on nocturnal cough and sleep quality for coughing children and their parents. Pediatrics.
2004;114:e85-e90. Available at: http://pediatrics.aappublications.org/cgi/content/full/114/1/e85
Question: 117
One of your patients underwent enucleation of one eye for treatment of retinoblastoma. He now
presents at age 5 years for his first preparticipation sports physical for t-ball.
Of the following, the MOST important advice for him and his parents is that
B. he should be taught to slide head first into base so he can better see where he is going
C. he should not participate in any sports because he has only one eye
D. only children who have eye problems need to wear goggles when participating in sports
References:
American Academy of Pediatrics. Committee on Sports Medicine and Fitness. Policy statement:
protective eyewear for young athletes. Pediatrics. 2004;113:619-622. Available at:
http://pediatrics.aappublications.org/cgi/content/full/113/3/619
American Academy of Pediatrics. Committee on Sports Medicine and Fitness. Risk of injury
from baseball and softball in children. Pediatrics. 2001;107:782-784. Available at:
http://pediatrics.aappublications.org/cgi/content/full/107/4/782
Committee on Sports Medicine and Fitness. Medical conditions affecting sports participation.
Pediatrics. 1994;94:757-760. Available at:
http://pediatrics.aappublications.org/cgi/reprint/94/5/757
Listman DA. Paintball injuries in children: more than meets the eye. Pediatrics. 2004;113:e15-
e18. Available at: http://pediatrics.aappublications.org/cgi/content/full/113/1/e15
Critique: 117
An example of specially designed glasses with polycarbonate lenses that are recommended for
athletes who have one functional eye.
Courtesy of D. Krowchuk
Question: 118
You are seeing a 10-year-old boy for a health supervision visit. His brother and sister
accompany him and his mother to the visit. The mother reports that her husband recently
underwent a heart transplant for a “thick heart.” As you explore the family history in more detail,
you learn that the boy’s paternal uncle and grandfather both have been diagnosed with
hypertrophic cardiomyopathy. The boy has never had chest pain, palpitation, shortness of
breath, dizziness, or syncope. He participates in sports and activities without any problems.
Findings on physical examination are within normal limits.
References:
Maron BJ, Chaitman BR, Ackerman MJ, et al. Recommendations for physical activity and
recreational sports participation for young patients with genetic cardiovascular diseases.
Circulation. 2004;109:2807-2816. Available at:
http://circ.ahajournals.org/cgi/content/full/109/22/2807
Question: 119
A 10-year-old boy presents to the emergency department with confusion. He is febrile. While
you are examining him, his eyes glaze over and deviate to the right, he has automatic chewing
movements, and he is completely unresponsive for 30 seconds, after which he is very sleepy.
Emergent head computed tomography scan shows low density in the right temporal lobe.
E. emergent electroencephalography
References:
Bale JF. Meningitis and encephalitis. In: Maria BL, ed. Current Management in Child Neurology.
3rd ed. Hamilton, Ontario, Canada: BC Decker; 2005:509-514
Kohl S. Herpes simplex virus. In: Behrman RE, Kliegman RM, Jenson HB, eds. Nelson Textbook
of Pediatrics. 17th ed. Philadelphia, Pa: Saunders; 2004:1051-1056
Tyler KL. Update on herpes simplex encephalitis. Rev Neurol Dis. 2004;1:169-178. Abstract
available at:
http://www.ncbi.nlm.nih.gov/entrez/query.fcgi?db=pubmed&cmd=Retrieve&dopt=AbstractPlus&li
st_uids=16400278
Whitley RJ, Kimberlin DW. Herpes simplex encephalitis: children and adolescents. Semin Pediatr
Infect Dis. 2005;16:17-23. Abstract available at:
http://www.ncbi.nlm.nih.gov/entrez/query.fcgi?db=pubmed&cmd=Retrieve&dopt=AbstractPlus&li
st_uids=15685145
Critique: 119
Herpes simplex virus encephalitis: Axial computed tomography scan performed 2 weeks after the
onset of illness reveals necrosis in the right temporal lobe (arrow).
Courtesy of D. Gilbert
Question: 120
The parents of a child in your practice call you for advice after learning that the husband’s
newborn niece has been diagnosed with a rare inborn error of metabolism. The mother of your
patient is 16 weeks pregnant with their second child. They ask you whether their new baby will
be affected. These parents are not related to one another, and the wife is not related to the
mother of the affected newborn. You refer them for genetic counseling.
Of the following, the risk for this same metabolic condition to affect the unborn child is CLOSEST
to
A. <1%
B. 5%
C. 10%
D. 25%
E. 50%
References:
Genetic variation in populations. In: Nussbaum RL, McInnes RR, Willard HF, eds. Thompson &
Thompson Genetics in Medicine. 6th ed. Philadelphia, Pa: Saunders; 2004:95-109
Patterns of single-gene inheritance. In: Nussbaum RL, McInnes RR, Willard HF, eds. Thompson
& Thompson Genetics in Medicine. 6th ed. Philadelphia, Pa: Saunders; 2004:51-77
Question: 121
A 16-year-old boy comes to your clinic with complaints of a penile discharge and dysuria. He
reports no fever or scrotal tenderness. On physical examination, he is afebrile. He has a thick
yellowish discharge at the penile meatus, but no genital rashes or lesions, no scrotal tenderness
or swelling, and no inguinal adenopathy.
Of the following, the MOST likely causative organism for his symptoms is
A. herpes simplex 2
B. human papillomavirus
C. Neisseria gonorrhoeae
D. Treponema pallidum
E. Trichomonas vaginalis
References:
Centers for Disease Control and Prevention. Sexually transmitted diseases treatment guidelines,
2006. MMWR Recomm Rep. 2006;55(RR-11):1-94. Available at:
http://www.cdc.gov/mmwr/preview/mmwrhtml/rr5511a1.htm
Fortenberry JD, Neinstein LS. Overview of sexually transmitted diseases. In: Neinstein LS, ed.
Adolescent Health Care: A Practical Guide. 4th ed. Philadelphia, Pa: Lippincott Williams &
Wilkins; 2002:1085-1117
Question: 122
A 7-year-old boy comes to your office for his annual health supervision visit. On physical
examination, he has Sexual Maturity Rating 3 pubic hair, his penis is 5 cm in stretched length,
his testes are 2 mL in volume, the scrotum is rugated, and there is no thinning of scrotal skin.
His weight is at the 90th percentile, and his height is at the 50th percentile (Item Q122).
D. premature adrenarche
Question: 122
Courtesy of L. Levitsky
References:
Rosenfield RL, Qin K. Premature adrenarche. UpToDate Online 14.3. Available for subscription
at:
http://www.utdol.com/utd/content/topic.do?topicKey=pediendo/12377&type=A&selectedTitle=1~8
Question: 123
The mother of a 2-year-old girl is very concerned that her daughter is developmentally delayed.
She explains that the girl speaks in two- to three-word phrases. She can feed herself with a
spoon, but is unable to button her clothing. She can follow simple two-step commands and can
climb stairs. However, she is not yet toilet trained. Findings on physical examination are
unremarkable.
E. schedule a 6-month follow-up evaluation to see if the child has reached the milestones
References:
Needlman R. The first year: In : Berman RE, Kliegman RM, Jenson HB, eds. Nelson Textbook of
Pediatrics. 17th ed. Philadelphia, Pa: Saunders; 2004:31-37
Parker S. Toliet training. In: Parker P, Zuckerman B, Augustyn M. Developmental and Behavioral
Pediatrics: A Handbook for Primary Care. 2nd ed. Philadelphia, Pa : Lippincott Williams &
Williams; 2005: 355-357
Shelov S. Ages two to three years: growth and development. In: The American Academy of
Pediatrics Caring for Your Baby and Young Child: Birth to age 5. Elk Grove Village, Ill: American
Academy of Pediatrics; 2004:301-315
Shelov S. The second year: growth and development. In: The American Academy of Pediatrics
Caring for Your Baby and Young Child: Birth to age 5. Elk Grove Village, Ill: American Academy
of Pediatrics; 2004:265-277
Question: 124
A 5-day-old infant who was found in a trash dumpster is brought to you for evaluation. Because
you have no history on the baby, you decide to send serology specimens to test for syphilis,
human immunodeficiency virus (HIV), hepatitis B, hepatitis C, and rubella. One week later, all of
the test results are negative, with the exception of the presence of antibodies to rubella and HIV.
Of the following, the MOST appropriate next step in the evaluation and treatment of this infant is
to
A. begin zidovudine
References:
American Academy of Pediatrics. Human immunodeficiency virus infection. In: Pickering LK,
Baker CJ, Long SS, McMillan JA, eds. Red Book: 2006 Report of the Committee on Infectious
Diseases. 27th ed. Elk Grove Village, Ill: American Academy of Pediatrics; 2006:378-401
Centers for Disease Control and Prevention. U.S. Public Health Service task force
recommendations for use—of antiretroviral drugs in pregnant HIV-1-infected women for
maternal health and interventions to reduce perinatal HIV-1 transmission in the United States.
MMWR Morbid Mortal Wkly Rep. 2002;51(RR18):1-38. Available at:
http://www.cdc.gov/mmwr/preview/mmwrhtml/rr5118a1.htm
Question: 125
You are speaking to a group of medical students about the clinical symptoms, physical
examination findings, and laboratory diagnosis of viral organisms that can cause acute
encephalitis.
Of the following, the virus that can be identified MOST easily by culture techniques is
References:
American Academy of Pediatrics. Herpes simplex. In: Pickering LK, Baker CJ, Long SS,
McMillan JA, eds. Red Book: 2006 Report of the Committee on Infectious Diseases. 27th ed. Elk
Grove Village, Ill: American Academy of Pediatrics; 2006:361-371
Gill VJ, Fedorko DP, Witebsky FG. The clinician and the microbiology laboratory. In: Mandell GL,
Bennett JE, Dolin R, eds. Mandell, Douglas and Bennett’s Principles and Practice of Infectious
Diseases. 6th ed. Philadelphia, Pa: Elsevier Churchill Livingstone; 2005:203-241
Question: 126
An 18-month-old female presents with failure to thrive, polydipsia, and photophobia. Her weight is
8 kg and height is 70 cm (both <5th percentile). On physical examination, she appears pale and
small for stated age, and she closes her eyes when you attempt to perform ophthalmoscopy.
She has tacky mucous membranes and capillary refill of 2 to 3 seconds. Pertinent findings on
laboratory evaluation include:
C. ophthalmologic examination
References:
Foreman JW. Cystinosis and Fanconi syndrome. In: Avner ED, Harmon WE, Niaudet P, eds.
Pediatric Nephrology. 5th ed. Philadelphia, Pa: Lippincott Williams & Wilkins; 2004:789-806
Gahl WA, Thoene JG, Schneider JA. Cystinosis. N Engl J Med. 2002;347:111-121. Abstract
available at:
http://www.ncbi.nlm.nih.gov/entrez/query.fcgi?db=pubmed&cmd=Retrieve&dopt=AbstractPlus&li
st_uids=12110740
Question: 127
A 14-year-old girl comes to the clinic with a temperature of 100.6°F (38.1°C), myalgias,
arthralgias, and urticaria for the past 48 hours. She recently completed a 10-day course of
amoxicillin for a sinus infection. Her symptoms developed shortly after completing the course of
the antibiotic. On physical examination, she has a blanchable, erythematous rash on her hands
(Item Q127) and feet. There are no target lesions, and the lips and oral mucosa are not involved.
She denies rhinorrhea, cough, or recent sick contacts.
A. erythema multiforme
D. serum sickness
Question: 127
Reprinted with permission from Krowchuk DP, Mancini AJ, eds. Pediatric Dermatology. A Quick
Reference Guide. Elk Grove Village, Ill: American Academy of Pediatrics; 2007
References:
Sicherer SH, Leung DYM. Serum sickness. In: Behrman RE, Kliegman RM, Jenson HB, eds.
Nelson Textbook of Pediatrics. 17th ed. Philadelphia, Pa: Saunders; 2004:782-783
Wolf R, Orion E, Marcos B, Matz H. Life-threatening acute adverse cutaneous drug reactions.
Clin Dermatol. 2005;23:171-181. Abstract available at:
http://www.ncbi.nlm.nih.gov/entrez/query.fcgi?db=pubmed&cmd=Retrieve&dopt=AbstractPlus&li
st_uids=15802211
Critique: 127
The rash of serum sickness or a serum sickness-like reaction often appears as urticarial
plaques that have lilac or blue centers.
Reprinted with permission from Krowchuk DP, Mancini AJ, eds. Pediatric Dermatology. A Quick
Reference Guide. Elk Grove Village, Ill: American Academy of Pediatrics; 2007
Critique: 127
The lesions of erythema multiforme are erythematous papules or plaques that develop central
violaceous discoloration.
Reprinted with permission from Krowchuk DP, Mancini AJ, eds. Pediatric Dermatology. A Quick
Reference Guide. Elk Grove Village, Ill: American Academy of Pediatrics; 2007
Critique: 127
Reprinted with permission from Krowchuk DP, Mancini AJ, eds. Pediatric Dermatology. A Quick
Reference Guide. Elk Grove Village, Ill: American Academy of Pediatrics; 2007
Critique: 127
Courtesy of D. Krowchuk
Question: 128
A 2-year-old boy presents with bloody drainage from the left ear. According to his mother, he
has had upper respiratory tract infection symptoms for the past 3 days, and last night he was
crying and holding his left ear. His pain seemed to improve after she cleaned out the ear with a
cotton swab, but this morning there was blood on his pillow and around his left ear. On physical
examination of his ears, you are unable to see the left tympanic membrane because of
seropurulent fluid in the external auditory canal.
Of the following, the MOST likely cause of the bloody ear drainage is
C. otitis externa
References:
Bauer CA, Jenkins HA. Otologic symptoms and syndromes. In: Cummings CW, Haughey BH,
Thomas JR, et al, eds. Cummings Otolaryngology: Head and Neck Surgery. 4th ed.
Philadelphia, Pa: Mosby; 2005:2867-2871
Haddad J Jr. The ear: clinical manifestations. In: Behrman RE, Kleigman RM, Jenson HB, eds.
Nelson Textbook of Pediatrics. 17th ed. Philadelphia, Pa: Saunders; 2004:2127-2128
Question: 129
You are evaluating a 7-year-old girl who has a 2-day history of a rash without fever or other
symptoms. The only notable findings on physical examination are round, erythematous, thin
plaques, each of which has a central violaceous discoloration or blister (Item Q129). The lesions
are concentrated on the extremities, including the hands and feet, with relative sparing of the
trunk.
A. erythema migrans
B. erythema multiforme
C. Stevens-Johnson syndrome
E. urticaria
Question: 129
Reprinted with permission from Krowchuk DP, Mancini AJ, eds. Pediatric Dermatology. A Quick
Reference Guide. Elk Grove Villaage, Ill: American Academy of Pediatrics; 2007
References:
Krowchuk DP, Mancini AJ, eds. Erythema multiforme (EM)/Stevens-Johnson syndrome (SJS).
In: Pediatric Dermatology. A Quick Reference Guide. Elk Grove Village, Ill: American Academy
of Pediatrics; 2007:267-270
Paller AS, Mancini AJ. The hypersensitivity syndromes. In: Hurwitz Clinical Pediatric
Dermatology. 3rd ed. Philadelphia, Pa: Elsevier Saunders; 2006:525-556.
Weston WL, Lane AT, Morelli JG. Bullous diseases and mucocutaneous syndromes. In: Color
Textbook of Pediatric Dermatology. 3rd ed. St. Louis, Mo: Mosby; 2002:89:155-167
Critique: 129
Reprinted with permission from Krowchuk DP, Mancini AJ, eds. Pediatric Dermatology. A Quick
Reference Guide. Elk Grove Village, Ill: American Academy of Pediatrics; 2007Courtesy of D.
Krowchuk
Critique: 129
In Stevens-Johnson syndrome and toxic epidermal necrolysis, target lesions are uncommon.
Rather, patients develop erythematous macules or patches.
Courtesy of D. Krowchuk
Critique: 129
In Stevens-Johnson syndrome and toxic epidermal necrolysis, blisters develop (arrow) and
subsequently rupture, leaving denuded areas.
Courtesy of D. Krowchuk Reprinted with permission from Krowchuk DP, Mancini AJ, eds.
Pediatric Dermatology. A Quick Reference Guide. Elk Grove Village, Ill: American Academy of
Pediatrics; 2007
Critique: 129
Reprinted with permission from Bullen LK, Zenel JA. Visual diagnosis: a 15-year-old who has
cough, rash, and painful swallow. Pediatr Rev. 2005;26:176-181
Critique: 129
Swelling, ulceration, and crusting of the lips is a feature of Stevens-Johnson syndrome and toxic
epidermal necrolysis.
Courtesy of D. Krowchuk
Critique: 129
Multiple erythema migrans lesions may be observed in the early disseminated phase of Lyme
disease.
Courtesy of dermatlas.org
Critique: 129
The wheals of urticaria vary in size and often have unusual shapes.
Courtesy of D. Krowchuk
Question: 130
A 15-year-old girl who has Crohn disease has had poor appetite and chronic diarrhea despite
her medical treatment. In evaluating her nutritional state, you document a low plasma zinc
concentration and initiate zinc supplementation.
References:
Giles E, Doyle LW. Copper in extremely low-birthweight or very preterm infants. NeoReviews.
2007;8:e159-e164. Available at: http://neoreviews.aappublications.org/cgi/content/short/8/4/e159
Giles E, Doyle LW. Zinc in extremely low-birthweight or very preterm infants. NeoReviews.
2007;8:e165-e172. Available at: http://neoreviews.aappublications.org/cgi/content/short/8/4/e165
Krebs NF, Hambidge KM. Trace elements. In: Walker WA, Watkins JB, Duggan C, eds. Nutrition
in Pediatrics: Basic Science and Clinical Applications. 3rd ed. Hamilton Ontario, Canada: BC
Decker; 2003:86-110
Maverakis E, Fung MA, Lynch PJ, et al. Acrodermatitis enteropathica and an overview of zinc
metabolism. J Am Acad Dermatol. 2007;56:116-124. Abstract available at:
http://www.ncbi.nlm.nih.gov/entrez/query.fcgi?db=pubmed&cmd=Retrieve&dopt=AbstractPlus&li
st_uids=17190629
Critique: 130
Question: 131
You admitted a 2,000-g term newborn to the neonatal intensive care unit. Her Apgar scores
were 4 and 6 at 1 and 5 minutes, respectively. She exhibits seizure activity at 24 hours of age.
Physical examination reveals microcephaly (Item Q131), respiratory distress requiring assisted
ventilation, hepatosplenomegaly, cutaneous petechiae, and lethargy. Laboratory tests
demonstrate thrombocytopenia, anemia, elevated liver transaminases, and hyperbilirubinemia.
Of the following, the viral agent MOST likely to cause these findings is
A. adenovirus
B. Coxsackievirus B
C. cytomegalovirus
D. hepatitis C virus
Question: 131
Courtesy of B. Carter
References:
American Academy of Pediatrics. Cytomegalovirus infection. In: Pickering LK, Baker CJ, Long
SS, McMillan JA, eds. Red Book: 2006 Report of the Committee on Infectious Diseases. 27th ed.
Elk Grove Village, Ill: American Academy of Pediatrics; 2006:273-277
Pan ES, Cole FS, Weintrub PS. Viral infections of the fetus and newborn. In: Taeusch HW,
Ballard RA, Gleason CA, eds. Avery’s Diseases of the Newborn. 8th ed. Philadelphia, Pa:
Elsevier Saunders; 2005:495-529
Rathore MH. CMV infection in very low birth weight infants via breast milk. AAP Grand Rounds.
2002;7:29
Stehel E, Sánchez PJ. Cytomegalovirus infection in the fetus and neonate. NeoReviews.
2005;6:e38-e45. Available at: http://neoreviews.aappublications.org/cgi/content/full/6/1/e38
Critique: 131
Courtesy of B. Carter
Critique: 131
Reprinted with permission from Stehel E, Sánchez PJ. Cytomegalovirus infection in the fetus and
neonate. NeoReviews. 2005;6:e38-e45
Critique: 131
Infants who have congenital cytomegalovirus infection have hepatomegaly and splenomegaly
(arrows show lines on the abdomen that denote the positions of the edges of the liver and
spleen).
Reprinted with permission from Stehel E, Sánchez PJ. Cytomegalovirus infection in the fetus and
neonate. NeoReviews. 2005;6:e38-e45
Question: 132
A 2-year-old boy comes to the emergency department because of a barking cough. His mother
reports that he has no fever or shortness of breath, but you note a barking, seal-like cough
(Item Q132). His respiratory rate is 20 breaths/min, and there is no stridor. His lungs are clear,
and other findings on the physical examination are normal.
B. helium/oxygen mixture
C. nebulized albuterol
E. oral antibiotic
References:
Johnson DW, Jacobson S, Edney PC, Hadfield P, Mundy ME, Schuh S. A comparison of
nebulized budesonide, intramuscular dexamethasone, and placebo for moderately severe
croup. N Engl J Med. 1998;339:498-503. Abstract available at: Abstract available at:
http://www.ncbi.nlm.nih.gov/entrez/query.fcgi?db=pubmed&cmd=Retrieve&adopt=AbstractPlus&l
ist_uids=9709042
Malhotra A, Krilov LR. Viral croup. Pediatr Rev. 2001;22:5-12. Available at:
http://pedsinreview.aappublications.org/cgi/content/full/22/1/5
Moore M, Little P. Humidified air inhalation for treating croup. Cochrane Database Syst Rev.
2007;1:CD002870. Available at:
http://www.mrw.interscience.wiley.com/cochrane/clsysrev/articles/CD002870/frame.html
Roosevelt GE. Acute inflammatory upper airway obstruction. In: Behrman RE, Kliegman RM,
Jenson HB, eds. Nelson Textbook of Pediatrics. 17th ed. Philadelphia, Pa: Saunders; 2004:1405-
1409
Question: 133
A 15-year-old high school baseball player presents for his annual sports physical. As you
complete the adolescent interview, he acknowledges using chewing tobacco “because all the
other guys do it and the pros do it.”
Of the following, the BEST anticipatory guidance is to tell this teen that
E. you plan to inform his parents and coach of his tobacco use
References:
Bize R, Burnand B, Mueller Y, Cornuz J. Biomedical risk assessment as an aid for smoking
cessation. Cochrane Database Syst Rev. 2005;4:CD004705. Available at:
http://www.mrw.interscience.wiley.com/cochrane/clsysrev/articles/CD004705/frame.html
Escher SA, Tucker AM, Lundin TM, Grabiner MD. Smokeless tobacco, reaction time and
strength in athletes. Med Sci Sports Exerc. 1998;30:1548-1551. Abstract available at:
http://www.ncbi.nlm.nih.gov/entrez/query.fcgi?db=pubmed&cmd=Retrieve&adopt=AbstractPlus&l
ist_uids=9789857
Klein JD, Camenga DR. Tobacco prevention and cessation in pediatric patients. Pediatr Rev.
2004;25:17-26. Available at: http://pedsinreview.aappublications.org/cgi/content/full/25/1/17
Robertson PB, Walsh MM, Greene JC. Oral effects of smokeless tobacco use by professional
baseball players. Adv Dent Res. 1997;11:307-312. Abstract available at:
http://www.ncbi.nlm.nih.gov/entrez/query.fcgi?db=pubmed&cmd=Retrieve&adopt=AbstractPlus&l
ist_uids=9524430
Walker JF, Collins LC, Rowell PP, Goldsmith LJ, Moffatt RJ, Stamford BA. The effect of smoking
on energy expenditure and plasma catecholamine and nicotine levels during light physical
activity, Nicotine Tob Res. 1999;1:365-370. Abstract available at:
http://www.ncbi.nlm.nih.gov/entrez/query.fcgi?db=pubmed&cmd=Retrieve&adopt=AbstractPlus&l
ist_uids=11072434
Question: 134
You are evaluating a 3-year-old boy in the urgent care clinic for fever. His mother tells you that
he had been well until yesterday, when he had a temperature that she recorded by oral
thermometer at 103°F (39.5°C). He has had a clear nasal discharge, cough, and one episode of
emesis. At the time of your evaluation, the patient is eating ice chips from a cup that he is holding
while he is sitting on the bed. He has a temperature of 102.7°F (39.3°C), a heart rate of 140
beats/min, a respiratory rate of 30 breaths/min, and a blood pressure of 110/66 mm Hg. He has
coarse breath sounds with good air movement bilaterally. His pulses are strong throughout. His
capillary refill time is between 3 and 4 seconds in his hands and 2 seconds in his feet.
E. repetition of the perfusion examination with the patient supine and hands warmed
References:
Bengur AR, Meliones JN. Cardiogenic shock. New Horiz. 1998;6:139-149. Abstract available at:
http://www.ncbi.nlm.nih.gov/entrez/query.fcgi?db=pubmed&cmd=Retrieve&dopt=AbstractPlus&li
st_uids=9654321
Lister G. Poor systemic perfusion and circulatory shock. In: Rudolph CD, Rudolph AM, eds.
Rudolph’s Pediatrics. 21st ed. New York, NY: McGraw-Hill Medical Publishing Division; 2003:285-
292
Question: 135
A 12-year-old girl in your practice had been born preterm and presented in early childhood with
developmental delay. She was diagnosed with cerebral palsy and has been given physical,
occupational, and speech therapy. Magnetic resonance imaging at age 3 years showed white
matter volume loss, particularly adjacent to the ventricles. The parents are concerned their
daughter’s condition may be degenerating because she has fallen progressively further behind
her peers, and she has become increasingly anxious and oppositional. She has not developed
seizures. Her recent special education re-evaluation at school revealed verbal and performance
intelligence quotients in the 70s, unchanged from 3 years ago. On physical examination, you
note dolichocephaly, hyperreflexia at the knees, and two beats of clonus at each ankle.
D. referral to genetics
E. sleep-deprived electroencephalography
References:
Johnston MV. Encephalopathies. In: Behrman RE, Kliegman RM, Jenson HB, eds. Nelson
Textbook of Pediatrics. 17th ed. Philadelphia, Pa: Saunders; 2004:2023-2028
Johnston MV. Neurodegenerative disorders of childhood. In: Behrman RE, Kliegman RM,
Jenson HB, eds. Nelson Textbook of Pediatrics. 17th ed. Philadelphia, Pa: Saunders; 2004:2029-
2034
Kolodny EH, Fattqal-Valevski A. Degenerative disorders. In: Maria BL, ed. Current Management
in Child Neurology. 3rd ed. Hamilton, Ontario, Canada: BC Decker; 2005:265-276
Question: 136
A neighbor approaches you regarding her concerns about her newborn grandson. Her 23-year-
old daughter, the mother of the baby, had an uneventful pregnancy, labor, and delivery, and “all
of her prenatal tests were normal.” The baby has been in the normal newborn nursery for 3
days, and he is feeding poorly. He “just can’t seem to get the hang of sucking.” She thinks he is
probably just “lazy,” and she describes him as feeling like a “rag doll” when she holds him. She
has noticed that he has “a lot of skin” at the back of his neck, and his fifth fingers are “crooked”
(Item Q136). She thinks his penis is small. You suggest that she alert the baby’s physician to
her concerns.
A. 22q11 deletion
B. achondroplasia
C. cri-du-chat syndrome
D. Down syndrome
E. Prader-Willi syndrome
Question: 136
Deformity of the fifth fingers, as described for the infant in the vignette.
Courtesy of M. Rimsza
References:
Achondroplasia. In: Jones KL. Smith’s Recognizable Patterns of Human Malformation. 6th ed.
Philadelphia, Pa: Elsevier Saunders; 2006:390-397
Deletion 22q11.2 syndrome. In: Jones KL. Smith’s Recognizable Patterns of Human
Malformation. 6th ed. Philadelphia, Pa: Elsevier Saunders; 2006:298-301
Deletion 5p syndrome. In: Jones KL. Smith’s Recognizable Patterns of Human Malformation. 6th
ed. Philadelphia, Pa: Elsevier Saunders; 2006:40-43
Down syndrome. In: Jones KL. Smith’s Recognizable Patterns of Human Malformation. 6th ed.
Philadelphia, Pa: Elsevier Saunders; 2006:7-12
Critique: 136
Clinodactyly (incurving) of the fifth fingers occurs in 50% of children who have Down syndrome.
Courtesy of M. Rimsza
Critique: 136
Dysmorphic features of Down syndrome include a flat facial profile and upslanting palpebral
fissures.
Courtesy of M. Rimsza
Critique: 136
A 3-month-old who has Prader-Willi syndrome: Note the almond-shaped eyes and downturned
mouth.
Reprinted with permission from Jonas JM, Demmer LA. Genetic syndromes determined by
alterations in genomic imprinting pathways. NeoReviews 2007;8:e120-e126
Critique: 136
Infants who have achondroplasia exhibit macrocephaly and a depressed nasal bridge. Another
feature observed in this infant is a small chest compared with the abdomen.
Courtesy of M. Rimsza
Critique: 136
For patients who have achondroplasia, tapered fingers and a relatively large gap between the
third and fourth fingers may lead to a "trident" appearance of the hand.
Courtesy of T. Jewett
Question: 137
During the health supervision visit for a 15-year-old girl, her mother states that her daughter is
currently suspended from school for fighting and that she was previously caught carrying a knife
to school. You decide to counsel them both about adolescent violence.
D. risk factors for school violence include attendance at small rural schools
References:
Eaton DK, Kann L, Kinchen S, et al. Youth risk behavior surveillance-United States, 2005.
MMWR Morbid Mortal Wkly Rep Surv Summ. 2006;55(SS05):1-108. Available at:
http://www.cdc.gov/mmwr/preview/mmwrhtml/ss5505a1.htm
McIntosh G. Adolescents and violence. In: Osborn LM, DeWitt TG, First LR, Zenel JA, eds.
Pediatrics. Philadelphia, Pa: Elsevier Mosby; 2005:1512-1518
National Center for Injury Prevention and Control. Best Practices of Youth Violence Prevention:
A Sourcebook for Communmity Action. Atlanta. Ga: Centers for Disease Control and Prevention.
2002. Available at: http://www.cdc.gov/ncipc/dvp/bestpractices.htm806Download
Question: 138
You are caring for a 12-year-old boy who has been receiving 125 mcg/d of thyroxine for
treatment of congenital hypothyroidism. He has been growing well and doing well in school. His
thyroxine and thyroid-stimulating hormone (TSH) values have been normal and stable for the
past year. At this visit, you measure a free thyroxine (fT4) concentration of 2.5 ng/dL (32.3
pmol/L) (normal, 0.8 to 2.0 ng/dL [10.3 to 25.7 pmol/L]) and a TSH concentration of 12.6 mIU/L
(12.6 mU/L) (normal, 0.5 to 5.0 mIU/L [0.5 to 5.0 mU/L]).
Of the following, the MOST likely reason for these abnormal laboratory study results is
References:
Ross DS. Laboratory assessment of thyroid function. UpToDate Online 14.3. Available for
subscription at:
http://www.utdol.com/utd/content/topic.do?topicKey=thyroid/9466&type=A&selectedTitle=1~23
Question: 139
The mother of one of your patients is in the process of getting a divorce and has just moved into
a small apartment. Due to her new work schedule, she has not been able to unpack the boxes
left in the kitchen. She brings her 4-year-old daughter to your office because the child cut herself
with a knife trying to open one of the unpacked boxes. The mother is visibly upset. After you
bandage the daughter’s injured hand, you sit down to talk with the mother.
A. explore what support system the mother has to help her family settle into their new home
B. recommend that the mother place her child in time-out for playing with the knife
E. tell the mother that you are obligated to report her to the child welfare agency
References:
About CAPTA: a legislative history. Washington, DC: Child Welfare Information Gateway; 2004.
Available at: http://www.childwelfare.gov/pubs/factsheets/about.cfm
Christopherson ER. Behavioral management: theory and practice. In: Parker P, Zuckerman B,
Augustyn M. Developmental and Behavioral Pediatrics: A Handbook for Primary Care. 2nd ed.
Philadelphia, Pa: Lippincott Williams & Williams; 2005:55-60
Shelov S. Age three to five years: sleeping. In: The American Academy of Pediatrics Caring for
Your Baby and Young Child: Birth to Age 5. Elk Grove Village, Ill: American Academy of
Pediatrics; 2004:375-388
Question: 140
During your morning nursery rounds, you find you have a new patient who was born to a mother
infected with human immunodeficiency virus (HIV). You introduce yourself to the mother, and
she asks you about any precautions she needs to take in the care of her newborn due to her
HIV infection.
Of the following, you are MOST likely to tell the mother that she should
B. avoid breastfeeding
References:
American Academy of Pediatrics. Human immunodeficiency virus infection. In: Pickering LK,
Baker CJ, Long SS, McMillan JA, eds. Red Book: 2006 Report of the Committee on Infectious
Diseases. 27th ed. Elk Grove Village, Ill: American Academy of Pediatrics; 2006:378-401
Thior I, Lockman S, Smeaton LM, et al. Breastfeeding plus infant zidovudine prophylaxis for 6
months vs formula feeding plus infant zidovudine for 1 month to reduce mother-to-child HIV
transmission in Botswana: a randomized trial: the Mashi study. JAMA. 2006;296:794-805.
Abstract available at:
http://www.ncbi.nlm.nih.gov/entrez/query.fcgi?db=pubmed&cmd=Retrieve&dopt=AbstractPlus&li
st_uids=16905785
Question: 141
A 3-year-old patient who has acute lymphoblastic leukemia is admitted to the pediatric intensive
care unit after developing severe sepsis due to Pseudomonas aeruginosa. She is intubated,
ventilated, and requires intensive vasopressor support.
Of the following, the MOST appropriate antibiotic regimen for the treatment of this patient is an
aminoglycoside plus
A. cefazolin
B. cefdinir
C. ceftazidime
D. ceftriaxone
E. cefuroxime
References:
Pier GB, Ramphal R. Pseudomonas aeruginosa. In: Mandell GL, Bennett JE, Dolin R, eds.
Mandell, Douglas and Bennett’s Principles and Practice of Infectious Diseases. 6th ed.
Philadelphia, Pa: Elsevier Churchill Livingstone; 2005:2587-2614
Prince AS. Pseudomonas aeruginosa. In: Long SS, Pickering LK, Prober CG, eds. Principles
and Practice of Pediatric Infectious Diseases. 2nd ed. New York, NY: Churchill Livingstone;
2003:857-860
Question: 142
You are seeing an 8-month-old infant for a health supervision visit. He was born at 28 weeks’
gestation, weighed 1,200 g at birth, and has bronchopulmonary dysplasia. His only medication is
furosemide, which he has been receiving for several months.
A. hypercalciuria
B. hypermagnesemia
C. hypocalcemia
D. hyponatremic dehydration
E. metabolic acidosis
References:
Eades SK, Christensen ML. The clinical pharmacology of loop diuretics in the pediatric patient.
Pediatr Nephrol. 1998;12:603-616. Abstract available at:
http://www.ncbi.nlm.nih.gov/entrez/query.fcgi?db=pubmed&cmd=Retrieve&dopt=AbstractPlus&li
st_uids=9761364
Rose BD, Post TW. Clinical use of diuretics. In: Clinical Physiology of Acid-base and Electrolyte
Disorders. 5th ed. New York, NY: McGraw-Hill Medical Publishing Division; 2001:447-477
Question: 143
A 17-year-old girl presents for a health supervision visit before leaving for college. As you review
her medical history, you note that she has marked “penicillin allergy” on the school health form.
She remarks that her mother told her she had a rash after amoxicillin when she was 2 years old.
Of the following, the BEST statement regarding penicillin drug reactions is that
B. negative skin testing to major and minor determinants of penicillin can exclude almost all
immunoglobulin (Ig) E-mediated reactions
C. nonpruritic maculopapular rash that occurs in patients who receive amoxicillin during
mononucleosis is a contraindication for future penicillin therapy
E. the incidence of IgE-mediated penicillin allergy among patients who have this history is
greater than 20%
References:
Boguniewicz M. Adverse reactions to drugs. In: Behrman RE, Kliegman RM, Jenson HB, eds.
Nelson Textbook of Pediatrics. 17th ed. Philadelphia, Pa: Saunders; 2004:783-785
Gruchalla RS. 10. Drug allergy. J Allergy Clin Immunol. 2003;111(suppl):S548-S559. Abstract
available at:
http://www.ncbi.nlm.nih.gov/entrez/query.fcgi?db=pubmed&cmd=Retrieve&dopt=AbstractPlus&li
st_uids=12592301
Critique: 143
A generalized macular and papular eruption can follow the administration of a penicillin in those
who have infectious mononucleosis.
Courtesy of D. Krowchuk
Question: 144
You are on the sidelines of a girl’s high school lacrosse game when one of the players is struck
in the mouth with the ball. She does not lose consciousness but runs off the field complaining of
severe mouth pain. On inspection of her mouth, you observe profuse bleeding, lacerations of the
upper alveolar mucosa, and avulsion of the right upper central incisor. Her teammate finds the
missing tooth on the field and brings it to you. You apply direct pressure to the bleeding gums
and make arrangements for the player to be transported to the emergency department.
Of the following, the MOST appropriate method for preparing the tooth for transport with the
patient is to
D. rub all debris from the tooth and place it in a dry plastic bag
References:
Martof A. Consultation with the specialist: dental care. Pediatr Rev. 2001;22:13-15. Available at:
http://pedsinreview.aappublications.org/cgi/content/full/22/1/13
McTigue DJ. Diagnosis and management of dental injuries in children. Pediatric Clin North Am.
2000;47;1067-1084. Abstract available at:
http://www.ncbi.nlm.nih.gov/entrez/query.fcgi?db=pubmed&cmd=Retrieve&dopt=AbstractPlus&li
st_uids=11059350
Question: 145
You are evaluating a newborn who has complete heart block and several 1.5-cm erythematous
macules and annuli located on the forehead, behind the ears, and in the scalp (Item Q145).
Of the following, the test MOST likely to confirm the infant’s diagnosis is
B. creatine kinase
D. platelet count
E. urinalysis
Question: 145
Courtesy of D. Krowchuk
References:
Horii KA, Nopper AJ, Sharma V. Picture of the month. Neonatal lupus erythematosus. Arch
Pediatr Adolesc Med. 2006;160:189-190. Abstract available at:
http://www.ncbi.nlm.nih.gov/entrez/query.fcgi?db=pubmed&cmd=Retrieve&dopt=AbstractPlus&li
st_uids=16461876
Krowchuk DP, Mancini AJ, eds. Systemic lupus erythematosus (SLE). In: Pediatric
Dermatology. A Quick Reference Guide. Elk Grove Village, Ill: American Academy of Pediatrics;
2007:483-493
Paller AS, Mancini AJ. Collagen vascular disorders. In: Hurwitz Clinical Pediatric Dermatology.
3rd ed. Philadelphia, Pa: Elsevier Saunders; 2006:573-608
Weston WL, Lane AT, Morelli JG. Papulosquamous disorders. In: Color Textbook of Pediatric
Dermatology. 3rd ed. St. Louis, Mo: Mosby; 2002:119-143
Critique: 145
Lesions of neonatal lupus erythematosus are erythematous annular plaques located in sun-
exposed areas.
Question: 146
A 15-year-old girl presents with an episode of “feeling faint” and melena. On physical
examination, you note a gallop rhythm and mild, nonspecific abdominal tenderness. Stool is
guaiac-positive. Laboratory analysis demonstrates anemia, with a hematocrit of 18% (0.18). You
administer fluid resuscitation and packed red blood cells, and the patient’s hemodynamic status
stabilizes.
A. angiography
References:
Ge ZZ, Chen HY, Gao YJ, Gu JL, Hu YB, Xiao SD. Clinical application of wireless capsule
endoscopy in pediatric patients for suspected small bowel diseases. Eur J Pediatr.
2006;epublished before print. Abstract available at:
http://www.ncbi.nlm.nih.gov/entrez/query.fcgi?db=pubmed&cmd=Retrieve&dopt=AbstractPlus&li
st_uids=17103187
Question: 147
A woman who has chorioamnionitis and had a positive group B streptococcal (GBS) screening
culture at 36 weeks’ gestation delivers an infant at term. The infant becomes ill in the first 4
hours after birth, demonstrating tachypnea, inability to maintain temperature, and poor perfusion.
He is admitted to the intensive care nursery. On physical examination, there are retractions of
the chest wall, coarse and shallow breath sounds, and delayed capillary refill. There is no heart
murmur or cyanosis, but arterial oxygen saturation determined by pulse oximetry is only 80% on
room air.
Of the following, the clinical manifestation that is observed MORE often in early-onset than late-
onset GBS infection is
A. cellulitis
B. meningitis
C. osteomyelitis
D. pneumonia
E. septic arthritis
References:
American Academy of Pediatrics. Group B streptococcal infections. In: Pickering LK, Baker CJ,
Long SS, McMillan JA, eds. Red Book: 2006 Report of the Committee on Infectious Diseases.
27th ed. Elk Grove Village, Ill: American Academy of Pediatrics; 2006:620-627
Baker CJ. Group B streptococcal disease. In: McMillan JA, Feigin RD, DeAngelis C,
Jones MD, eds. Oski's Pediatrics: Principles & Practice. 4th ed. Philadelphia, Pa:
Lippincott Williams & Wilkins; 2006:501-507
Fluegge K, Siedler A, Heinrich B, et al for the German Pediatric Surveillance Unit Study Group.
Incidence and clinical presentation of invasive neonatal group B streptococcal infections in
Germany. Pediatrics. 2006;117:e1139-e1145. Available at:
http://pediatrics.aappublications.org/cgi/content/full/117/6/e1139
Venkatesh M, Merenstein GB, Adams KM, Weisman LE. Infection in the neonate. In: Merenstein
GB, Gardner SL, eds. Handbook of Neonatal Intensive Care. 6th ed. St.Louis, Mo: Mosby
Elsevier; 2006:569-593
Question: 148
An 18-month-old girl is brought to the emergency department by her mother, who reports that
the girl has had a high fever and difficulty breathing for 1 day. She has had a barking cough for
the last 3 days but had been afebrile and breathing comfortably until today. Her immunizations
are up to date. Her temperature is 104ºF (40°C), her respiratory rate is 50 breaths/min, she
exhibits inspiratory stridor and a brassy cough, and she appears ill. Lung examination reveals
decreased breath sounds bilaterally with transmitted upper airway sounds. An anterior neck
radiograph shows subglottic narrowing and a ragged tracheal air column.
B. angioneurotic edema
C. bacterial tracheitis
D. epiglottitis
E. retropharyngeal abscess
References:
Malhotra A, Krilov LR. Viral croup. Pediatr Rev. 2001;22:5-12. Available at:
http://pedsinreview.aappublications.org/cgi/content/full/22/1/5
Roosevelt GE. Acute inflammatory upper airway obstruction. In: Behrman RE, Kliegman RM,
Jenson HB, eds. Nelson Textbook of Pediatrics. 17th ed. Philadelphia, Pa: Saunders; 2004:1405-
1409
Critique: 148
In epiglottitis, a lateral radiograph of the neck reveals a swollen epiglottis (the "thumb" sign).
Critique: 148
Retropharyngeal abscess: Lateral radiograph of the neck shows widening of the prevertebral soft-
tissue space (defined by arrows). Normally, this space should measure no more than one half
the width of the fifth cervical vertebral body.
Courtesy of B. Specter
Critique: 148
Retropharyngeal abscess: Computed tomography scan of the neck at the level of the angle of the
mandible shows soft-tissue swelling that impinges on the airway (arrow) and an abscess.
Courtesy of B. Specter
Question: 149
A 17-year-old girl presents to your school-based clinic with her 3-day-old newborn for an early
discharge follow-up visit. The young mother looks very tired and is tearful. She states that she is
trying to breastfeed but has to supplement the feeding with a bottle because she “doesn’t have
any milk.”
E. suggest that she attend prenatal classes for her next pregnancy
References:
Arora S, McJunkin C, Wehrer J, Kuhn P. Major factors influencing breastfeeding rates: mother's
perception of father's attitude and milk supply. Pediatrics. 2000;106:e67. Available at:
http://pediatrics.aappublications.org/cgi/content/full/106/5/e67
Escobar GJ, Braveman PA, Ackerson L, et al. A randomized comparison of home visits and
hospital-based group follow-up visits after early postpartum discharge. Pediatrics. 2001;108:719-
727. Available at: http://pediatrics.aappublications.org/cgi/content/full/108/3/719
Lieu TA, Wikler C, Capra AM, Martin KE, Escobar GJ, Braveman PA. Clinical outcomes and
maternal perceptions of an updated model of perinatal care. Pediatrics. 1998;102:1437-1444.
Available at: http://pediatrics.aappublications.org/cgi/content/full/102/6/1437
Question: 150
You are evaluating a 17-year-old boy whom you have known since early childhood. He is
complaining of headaches over the past 2 weeks. He has a history of asthma, which has been
well controlled, and he is an otherwise healthy member of the varsity football team at school. He
has had a significant weight gain of 30 lb (13.5 kg) since his visit to you 1 year ago. He denies
using illicit or prescription drugs. On physical examination, he appears very muscular and has a
blood pressure of 180/120 mm Hg. You repeat the measurement using a leg cuff to ensure
adequate cuff size and obtain the same result.
References:
Chobanian AV, Bakris GL, Black HR, et al; Joint National Committee on Prevention, Detection,
Evaluation, and Treatment of High Blood Pressure. National Heart, Lung, and Blood Institute;
National High Blood Pressure Education Program Coordinating Committee. Seventh report of the
Joint National Committee on Prevention, Detection, Evaluation and Treatment of High Blood
Ettinger LM, Spitzer A. Hypertension. eMedicine Pediatrics Cardiology. 2007. Available at:
http://www.emedicine.com/ped/topic1097.htm
National High Blood Pressure Education Program Working Group on High Blood Pressure in
Children and Adolescents. The fourth report on the diagnosis, evaluation, and treatment of high
blood pressure in children and adolescents. Pediatrics. 2004;114:555–576. Available at:
http://pediatrics.aappublications.org/cgi/content/full/114/2/S2/555
Question: 151
A mother brings her 3-year-old boy to the emergency department. She explains that the boy
suddenly stopped paying attention, stared, and had jerking of his arms and legs for about 1
minute. His lips turned blue, and he became incontinent of urine. After the episode, he appeared
confused and became very sleepy. On physical examination, he has a temperature of 104°F
(40°C). Following administration of acetaminophen, his temperature has decreased to 98.6°F
(37°C). He is alert, interactive with his parents, and has normal findings on physical examination.
References:
Baumann RJ, Duffner PK. Treatment of children with simple febrile seizures: the AAP practice
parameter. American Academy of Pediatrics. Pediatr Neurol. 2000;23:11-17. Abstract available
at:
http://www.ncbi.nlm.nih.gov/entrez/query.fcgi?db=pubmed&cmd=Retrieve&dopt=AbstractPlus&li
st_uids=10963965
Johnston MV. Seizures in childhood. In: Behrman RE, Kliegman RM, Jenson HB, eds. Nelson
Textbook of Pediatrics. 17th ed. Philadelphia, Pa: Saunders; 2004: 1993-2008
Waruiru C, Appleton R. Febrile seizures: an update. Arch Dis Child. 2004;89:751-756. Abstract
available at:
http://www.ncbi.nlm.nih.gov/entrez/query.fcgi?db=pubmed&cmd=Retrieve&dopt=AbstractPlus&li
st_uids=15269077
Question: 152
You care for a 3-year-old girl who was born with spina bifida. The child is otherwise normally
formed and is developmentally appropriate with respect to her social, fine motor, and language
skills. During a routine visit, her mother tells you that she is contemplating another pregnancy.
You mention the important role of folic acid in the prevention of neural tube defects, and you
recommend that she discuss this with her obstetrician.
Of the following, the recommendation, implemented before and during pregnancy, that should
have the GREATEST impact on reducing this woman’s risk for having a future child affected with
spina bifida is to
C. measure the woman’s serum folic acid level and supplement accordingly
References:
Centers for Disease Control and Prevention. Folic acid and prevention of spina bifida and
anencephaly: 10 years after the U.S. Public Health Services Recommendation. MMWR Morbid
Mortal Wkly Rep. 2002;51:1-3. Available at:
http://www.cdc.gov/mmwr/preview/mmwrhtml/rr5113a1.htm
Williams LJ, Rasmussen SA, Flores A, Kirby RS, Edmonds LD. Decline in the prevalence of
spina bifida and anencephaly by race/ethnicity: 1995-2002. Pediatrics. 2005;116:580-586.
Available at: http://pediatrics.aappublications.org/cgi/content/full/116/3/580
Question: 153
You are seeing a 15-year-old girl for her first health supervision visit to your practice. In
explaining your practice’s policies, you discuss confidentiality.
A. adolescents are more likely to seek health care for sensitive issues if they believe that their
parents will be informed
B. billing policies of an outpatient or inpatient facility are always confidential in regard to sexually
transmitted disease infection testing for adolescents
D. parents have access to all of an adolescent’s health information through the Health Insurance
Portability and Accountability Act of 1996 (HIPAA)
E. state laws mandate that adolescents in all states may receive confidential treatment for
alcohol and other drug use disorders
References:
Center for Adolescent Health and the Law. Policy Compendium on Confidential Health Services
for Adolescents. 2nd ed. 2005.
http://www.cahl.org/PDFs/Policy%20CompendiumPDFs/PolicyCompendium.pdf
English A, Kenney KE. State Minor Consent Laws: A Summary. 2nd ed. Chapel Hill, NC: Center
for Adolescent Health and the Law; 2003. Abstract available at:
http://www.cahl.org/MC%20Monograph.htm
Joffe A. Legal and ethical issues in adolescent health care. In: Osborn LM, DeWitt TG, First LR,
Zenel JA, eds. Pediatrics. Philadelphia, Pa: Elsevier Mosby; 2005:1428-1430
Standards for privacy of individually identifiable health information; final rule. Fed Reg.
2000;65:82461-82510.
Weddle M, Kokotailo P. Adolescent substance abuse: confidentiality and consent. Pediatr Clin
North Am. 2002;49:301-315. Abstract available at:
http://www.ncbi.nlm.nih.gov/entrez/query.fcgi?db=pubmed&cmd=Retrieve&dopt=AbstractPlus&li
st_uids=11993284
Weddle M, Kokotailo PK. Confidentiality and consent in adolescent substance abuse: an update.
Virtual Mentor: Ethics Journal of the American Medical Association. 2005: 7(3).
Question: 154
You are discussing treatment choices with the family of a 13-year-old girl in whom you have just
diagnosed hyperthyroidism. You include antithyroid drug therapy, surgery, and radioactive iodine
treatment in your discussion. They choose antithyroid drug therapy with methimazole for initial
therapy.
A. agranulocytosis
B. cholestatic jaundice
C. headache
D. hematuria
E. rash
References:
Ferry RJ Jr, Levitsky LL. Graves disease. eMedicine Pediatrics Endocrinology. Available at:
http://www.emedicine.com/ped/topic899.htm
Rivkees S. Radioactive iodine use in childhood Graves' disease: time to wake up and smell the I-
131. J Clin Endocrinol Metab. 2004;89:4227-4228. Available at:
http://jcem.endojournals.org/cgi/content/full/89/9/4227
Ross DS. Pharmacology and toxicity of thionamides. UpToDate. Online 14.3. Available for
subscription at:
http://www.utdol.com/utd/content/topic.do?topicKey=thyroid/5248&type=A&selectedTitle=1~28
Question: 155
One of your 2-year-old patients has prolonged crying and screaming episodes every time her
parents deny her access to something she desires. The mother reports that the girl often throws
herself on the floor, kicking and thrashing about for long periods of time. She asks you how she
should handle her daughter’s behavior.
A. consider giving in to the girl only when she is outside of the home to avoid a major tantrum
C. move the girl to a safe place if needed and ignore her when she has a tantrum
References:
Boyce WT, Shonkoff JP. Developmental and behavioral pediatrics. In: Rudolph CD, Rudolph AM,
eds. Rudolph’s Pediatrics. 21st ed. New York, NY: McGraw-Hill Medical Publishing Division;
2003:401-532
Zuckerman BS, Frank DA, Augustyn M. Infancy and toddler years. In: Levine MD, Carey WB,
Crocker AC, eds. Developmental-Behavioral Pediatrics. 3rd ed. Philadelphia, Pa: WB Saunders
Co; 1999:24-37
Question: 156
A 3-year-old girl presents to the clinic with the complaint of a recurrent abscess on her buttock.
The area has been indurated for 3 to 4 days, but the mother reports that some “nasty” drainage
began last night. On physical examination, the girl is afebrile and has an erythematous, indurated
area on her right buttock of approximately 2 cm in diameter. With pressure, it drains purulent
material.
A. amoxicillin/clavulanate
B. cephalexin
C. clindamycin
D. observation
E. trimethoprim-sulfamethoxazole
References:
American Academy of Pediatrics. Staphylococcal infections. In: Pickering LK, Baker CJ, Long
SS, McMillan JA, eds. Red Book: 2006 Report of the Committee on Infectious Diseases. 27th ed.
Elk Grove Village, Ill: American Academy of Pediatrics; 2006:598-610
Lee MC, Rios AM, Aten MF, et al. Management and outcome of children with skin and soft tissue
abscesses caused by community-acquired methicillin-resistant Staphylococcus aureus. Pediatr
Infect Dis J. 2004;23:123-127. Abstract available at:
http://www.ncbi.nlm.nih.gov/entrez/query.fcgi?db=pubmed&cmd=Retrieve&dopt=AbstractPlus&li
st_uids=14872177
Question: 157
You are talking to a group of medical students about the antibiotic vancomycin and its
associated adverse effects. One of the students asks you to explain “red man syndrome.”
Of the following, the MOST appropriate response is that “red man syndrome” is
References:
Cunha BA. Vancomycin. Med Clin North Am. 1995;79:817-831. Abstract available at:
http://www.ncbi.nlm.nih.gov/entrez/query.fcgi?db=pubmed&cmd=Retrieve&dopt=AbstractPlus&li
st_uids=7791425
O’Sullivan TL, Ruffing MJ, Lamp KC, Warbasse LH, Rybak MJ. Prospective evaluation of red
Reynolds PE. Structure, biochemistry and mechanism of action of glycopeptide antibiotics. Eur J
Clin Microbiol Infect Dis. 1989;8:943-950. Abstract available at:
http://www.ncbi.nlm.nih.gov/entrez/query.fcgi?db=pubmed&cmd=Retrieve&dopt=AbstractPlus&li
st_uids=2532132
Wallace MR, Mascola JR, Oldfield EC 3rd. Red man syndrome: incidence, etiology, and
prophylaxis. J Infect Dis. 1991;164:1180-1185. Abstract available at:
http://www.ncbi.nlm.nih.gov/entrez/query.fcgi?db=pubmed&cmd=Retrieve&dopt=AbstractPlus&li
st_uids=1955716
Wilhelm MP. Vancomycin. Mayo Clin Proc. 1991;66:1165-1170. Abstract available at:
http://www.ncbi.nlm.nih.gov/entrez/query.fcgi?db=pubmed&cmd=Retrieve&dopt=AbstractPlus&li
st_uids=1943250
Question: 158
You are evaluating a 10-year-old girl for a health supervision visit. Her weight and height are at
the 50th percentile for age, her blood pressure is 108/64 mm Hg, and there are no unusual
findings on physical examination. A screening urinalysis shows a specific gravity of 1.030, pH of
6.5, 2+ blood, and no protein. Urine microscopy reveals 5 to 10 red blood cells/high-power field.
References:
Fitzwater DS, Wyatt RJ. Hematuria. Pediatr Rev. 1994;15:102-108. Available at:
http://pedsinreview.aappublications.org/cgi/reprint/15/3/102
Kalia A, Travis LB. Hematuria, leukocyturia, and cylindruria. In: Edelmann CM Jr, ed. Pediatric
Kidney Disease. 2nd ed. Boston, Mass: Little, Brown and Company; 1992:553-563
Question: 159
A 10-year-old boy presents for evaluation of hives that have occurred daily over the past 4
months. His parents are frustrated by the lack of change in their son’s symptoms despite
changing soap, fabric softener, and detergent. They would like to have their son seen by a
specialist for more testing. They describe the hives as raised, erythematous, pruritic 1- to 2-cm
lesions that involve the trunk and extremities. The hives resolve spontaneously within a few
hours and seem to occur at any time of the day or night. The child is otherwise healthy and is
only taking an over-the-counter antihistamine to help with itching.
Of the following, the MOST likely cause for this child’s hives is
E. systemic mastocytosis
References:
Brunetti L, Francavilla R, Miniello VL, et al. High prevalence of autoimmune urticaria in children
with chronic urticaria. J Allergy Clin Immunol. 2004;114:922-927. Abstract available at:
http://www.ncbi.nlm.nih.gov/entrez/query.fcgi?db=pubmed&cmd=Retrieve&dopt=AbstractPlus&li
st_uids=15480336
Critique: 159
Courtesy of D. Krowchuk
Question: 160
A 15-year-old basketball player presents to your office with an acute shoulder injury. He reports
that an opponent ran into his left, posteriorly outstretched arm as he was guarding him. He
developed pain in his left shoulder and has been unable to move his arm since the injury.
Physical examination reveals asymmetry of his shoulders, with increased shoulder slope on the
left. The left arm is externally rotated and slightly abducted. Fullness is palpable inferior to the left
mid-clavicle, and there is no pinprick sensation over the lateral deltoid. You place the patient’s
left arm in a sling, administer acetaminophen and codeine, and send him for radiographs.
Of the following, the MOST helpful radiographic view for diagnosing his injury is
References:
Anterior instability of the shoulder. In: Wheeless’ Textbook of Orthopaedics. Duke University
Medical Center's Division of Orthopaedic Surgery. Available at:
www.wheelessonline.com/ortho/anterior_instability_of_the_shoulder
Carson S, Woolridge DP, Colletti J, Kilgore K. Pediatric upper extremity injuries. Pediatr Clin
North Am. 2006;53:41-67. Abstract available at:
http://www.ncbi.nlm.nih.gov/entrez/query.fcgi?db=pubmed&cmd=Retrieve&dopt=AbstractPlus&li
st_uids=16487784
Deitch J, Mehlman CT, Foad SL, Obbehat A, Mallory M. Traumatic anterior shoulder dislocation
in adolescents. Am J Sports Med. 2003;31:758-763. Abstract available at:
http://www.ncbi.nlm.nih.gov/entrez/query.fcgi?db=pubmed&cmd=Retrieve&dopt=AbstractPlus&li
st_uids=12975198
te Slaa RL, Wijffels MP, Brand R, Marti RK. The prognosis following acute primary glenohumeral
dislocation. J Bone Joint Surg Br. 2004;86:58-64. Abstract available at:
http://www.ncbi.nlm.nih.gov/entrez/query.fcgi?db=pubmed&cmd=Retrieve&dopt=AbstractPlus&li
st_uids=14765867
Critique: 160
In the normal shoulder (top) the humeral head is adjacent to the glenoid fossa. In an anterior
shoulder dislocation (bottom), the humeral head is positioned anterior to the glenoid fossa.
Courtesy of D. Mulvihill
Question: 161
A 14-year-old girl presents for evaluation of areas of skin thickening, tightness, and discoloration
that developed 2 months ago. Physical examination reveals shiny, hypopigmented patches with
brown borders on the leg and ankle (Item Q161). The affected skin is immobile, firm, and has a
“bound-down” feeling.
B. linear scleroderma
C. pityriasis alba
E. vitiligo
Question: 161
Courtesy of D. Krowchuk
References:
Paller AS, Mancini AJ. Collagen vascular disorders. In: Hurwitz Clinical Pediatric Dermatology.
3rd ed. Philadelphia, Pa: Elsevier Saunders; 2006:573-608
Weston WL, Lane AT, Morelli JG. Immobile and hypermobile skin. In: Color Textbook of Pediatric
Dermatology. 3rd ed. St. Louis, Mo: Mosby; 2002:89:264-269
Zulian F. Scleroderma in children. Pediatr Clin North Am. 2005;52:521-545. Abstract available at:
http://www.ncbi.nlm.nih.gov/entrez/query.fcgi?db=pubmed&cmd=Retrieve&dopt=AbstractPlus&li
st_uids=15820378
Critique: 161
Linear scleroderma affecting the ankle: The skin has a waxy appearance, and there is central
hypopigmentation and peripheral hyperpigmentation.
Courtesy of D. Krowchuk
Critique: 161
Lichen sclerosus et atrophicus produces hypopigmentation and atrophy that usually affect the
genitalia. (There is a suture at the site of a biopsy.)
Courtesy of D. Krowchuk
Critique: 161
Courtesy of D. Krowchuk
Critique: 161
Vitiligo causes depigmented macules or patches that have well-defined borders. In this patient,
some areas have begun to repigment (arrow).
Courtesy of D. Krowchuk
Question: 162
A 16-year-old girl presents with a 4-month history of right upper quadrant abdominal pain. The
pain occurs at different times, but seems to strike primarily after meals, more frequently after
she eats fatty foods. In your office, she complains of intermittent pain to deep palpation of the
right upper quadrant. Complete blood count, alanine aminotransferase, alkaline phosphatase,
serum bilirubin, amylase, and lipase findings are normal. Abdominal ultrasonography shows no
evidence of stones or gallbladder thickening. Upper endoscopy and biopsy results are normal,
with no evidence of ulcers or gastritis.
References:
Hadigan C, Fishman SJ, Connolly LP, Treves ST, Nurko S. Stimulation with fatty meal (Lipomul)
to assess gallbladder emptying in children with chronic acalculous cholecystitis. J Pediatr
Gastroenterol Nutr. 2003;37:178-182. Abstract available at:
http://www.ncbi.nlm.nih.gov/entrez/query.fcgi?db=pubmed&cmd=Retrieve&dopt=AbstractPlus&li
st_uids=12883305
Question: 163
An infant who was born at 26 weeks’ gestation, weighing 700 g, is nearing 37 weeks corrected
age. He is receiving nasal cannula oxygen (0.2 L/min) and being treated with diuretics for
chronic lung disease. He has no intracranial hemorrhage and is growing well on enteral feedings
via a nasogastric tube. Attempts at oral feeding have been unsuccessful because of frequent
oxygen desaturation, bradycardia, and concerns over the inability to coordinate sucking-
swallowing. The mother asks why you are continuing to feed her son via a feeding tube.
Of the following, the condition that BEST explains why her son has feeding problems is
A. apnea of prematurity
C. gastroesophageal reflux
D. necrotizing enterocolitis
E. tracheoesophageal fistula
References:
Allen MC. Risk assessment and neurodevelopmental outcomes. In: Taeusch HW, Ballard RA,
Gleason CA, eds. Avery’s Diseases of the Newborn. 8th ed. Philadelphia, Pa: Elsevier
Saunders; 2005:1026-1042
Bos AF, Dibiasi J, Tiessen AH, Bergman KA. Treating preterm infants at risk for chronic lung
disease with dexamethasone leads to an impaired quality of general movements. Biol Neonate.
2002;82:155-158. Abstract available at:
http://www.ncbi.nlm.nih.gov/entrez/query.fcgi?db=pubmed&cmd=Retrieve&dopt=AbstractPlus&li
st_uids=12373065
Gewolb IH, Vice FL. Abnormalities in the coordination of respiration and swallow in preterm
infants with bronchopulmonary dysplasia. Dev Med Child Neurol. 2006;48:595-599. Abstract
available at:
http://www.ncbi.nlm.nih.gov/entrez/query.fcgi?db=pubmed&cmd=Retrieve&dopt=AbstractPlus&li
st_uids=16780630
Mercado-Deane MG, Burton EM, Harlow SA, et al. Swallowing dysfunction in infants less than 1
year of age. Pediatr Radiol. 2001;31:423-428. Abstract available at:
http://www.ncbi.nlm.nih.gov/entrez/query.fcgi?db=pubmed&cmd=Retrieve&dopt=AbstractPlus&li
st_uids=11436889
Vohr BR, Wright LL, Dusick AM, et al. Neurodevelopmental and functional outcomes of
extremely low birth weight infants in the National Institute of Child Health and Human
Development Neonatal Research Network, 1993-1994. Pediatrics. 2000;105:1216-1226.
Available at: http://pediatrics.aappublications.org/cgi/content/full/105/6/1216
Wood NS, Costeloe K, Gibson AT, Hennessy EM, Marlow N, Wilkinson AR; The EPICure Study
Group. The EPICure study: associations and antecedents of neurological and developmental
disability at 30 months of age following extremely preterm birth. Arch Dis Child Fetal Neonatal
Question: 164
You are asked to consult on a 9-month-old boy who has been hospitalized five times for
wheezing. His history reveals occasional coughing with feedings, but results of a pH probe
performed during his last admission were normal. His weight and height are at the 50th
percentile. Except for scattered wheezes with good aeration bilaterally, results of his physical
examination are normal.
Of the following, the test that is MOST likely to reveal the cause of his recurrent wheezing is
B. immunoglobulin panel
References:
Newman LA, Keckley C, Petersen MC, Hamner A. Swallowing function and medical diagnoses in
infants suspected of dysphagia. Pediatrics. 2001;108:e106-e110. Available at:
http://pediatrics.aappublications.org/cgi/content/full/108/6/e106
Sheikh S, Allen E, Shell R, et al. Chronic aspiration without gastroesophageal reflux as a cause
of chronic respiratory symptoms in neurologically normal infants. Chest. 2001;120:1190-1195.
Abstract available at:
http://www.ncbi.nlm.nih.gov/entrez/query.fcgi?db=pubmed&cmd=Retrieve&adopt=AbstractPlus&l
ist_uids=11591559
Critique: 164
Normal and abnormal swallowing: In normal swallowing (left), the food bolus enters the
esophagus. Laryngeal penetration occurs when food enters the airway above the vocal cords
(center). In aspiration (right), food travels below the vocal cords into the trachea.
Courtesy of A. Johnson
Question: 165
A 6-year-old boy has complained of periumbilical abdominal pain for the past 2 months. He has a
history of mild constipation that his mother has managed successfully by increasing juice and
fruits in his diet. The mother is frustrated because the boy has been sent home from school
frequently due to complaints of pain. She reports that he is being evaluated for dyslexia. The
pain occasionally occurs on weekends and school vacation breaks.
E. school phobia
References:
Campo JV, Bridge J, Ehmann M, et al. Recurrent abdominal pain, anxiety, and depression in
primary care. Pediatrics 2004;113:817-824. Available at:
http://pediatrics.aappublications.org/cgi/content/full/113/4/817
Question: 166
Included in your rounds today is a 36-hour-old boy who was born at term by normal,
spontaneous vaginal delivery. His respiratory rate is 80 breaths/min and heart rate is 168
beats/min. He has easily palpable, bounding pulses in all four extremities, and his blood pressure
is 72/30 mm Hg. Precordial examination reveals a lift and a 3/6 systolic ejection murmur at the
upper left sternal border (Item Q166). You also note a murmur over the anterior fontanelle.
B. aortic insufficiency
References:
Kallfelz C. Arteriovenous fistulae and allied lesions. In: Moller JH, Hoffman JIE, eds. Pediatric
Cardiovascular Medicine. Philadelphia, Pa: Churchill Livingstone; 2000:649-664
Talner NS, McGovern JJ, Carboni MP. Congestive heart failure. In: Moller JH, Hoffman JIE, eds.
Pediatric Cardiovascular Medicine. Philadelphia, Pa: Churchill Livingstone; 2000:817-829
Question: 167
An 8-year-old girl is brought to the emergency department via ambulance. On the playground,
she suddenly stopped playing, bent forward and fell to the ground, and had jerking of her arms
and legs. She drooled excessively and was unresponsive. Afterwards, she was confused, her
speech was slurred, and she was somewhat combative for about 30 minutes. In the emergency
department, she is responding appropriately, is afebrile, and has normal findings on general and
neurologic examinations. Her mother states that she has always been healthy and is an average
student. Review of systems reveals no headaches or recent illness.
Of the following, the MOST appropriate next step prior to discharge from the emergency
department is to
References:
Bluvstein JS, Moshé SL. First unprovoked seizure. In: Maria BL, ed. Current Management in
Child Neurology. 3rd ed. Hamilton, Ontario, Canada: BC Decker; 2005:83-88
Camfield P, Gordon K, Camfield C, Tibbles J, Dooley J, Smith B. EEG results are rarely the
same if repeated within six months in childhood epilepsy. Can J Neurol Sci. 1995;22:297-300.
Abstract available at:
http://www.ncbi.nlm.nih.gov/entrez/query.fcgi?db=pubmed&cmd=Retrieve&dopt=AbstractPlus&li
st_uids=8599774
Freeman JM. The evaluation of a child with a first seizure. In: Singer HS, Kossoff EH, Hartman
AL, Crawford TO, eds. Treatment of Pediatric Neurologic Disorders. Boca Raton, Fla: Taylor &
Francis Group; 2005:55-60
Gilbert DL, Buncher CR. An EEG should not be obtained routinely after first unprovoked seizure
in childhood. Neurology. 2000;54:635-641. Abstract available at:
http://www.ncbi.nlm.nih.gov/entrez/query.fcgi?db=pubmed&cmd=Retrieve&dopt=AbstractPlus&li
st_uids=10680796
Hirtz D, Ashwal S, Berg A, Bettis D, et al. Practice parameter: evaluating a first nonfebrile
seizure in children: report of the Quality Standards Subcommittee of the American Academy of
Neurology, The Child Neurology Society, and The American Epilepsy Society. Neurology.
2000;55:616-623. Abstract available at:
http://www.ncbi.nlm.nih.gov/entrez/query.fcgi?db=pubmed&cmd=Retrieve&dopt=AbstractPlus&li
st_uids=10980722
Johnston MV. Seizures in childhood. In: Behrman RE, Kliegman RM, Jenson HB, eds. Nelson
Textbook of Pediatrics. 17th ed. Philadelphia, Pa: Saunders; 2004:1993-2008
Musicco M, Beghi E, Solari A, Viani F. Treatment of first tonic-clonic seizure does not improve
the prognosis of epilepsy. First Seizure Trial Group (FIRST Group). Neurology. 1997;49:991-
998. Abstract available at:
http://www.ncbi.nlm.nih.gov/entrez/query.fcgi?db=pubmed&cmd=Retrieve&dopt=AbstractPlus&li
st_uids=9339678
Question: 168
A 12-year-old girl comes with her mother to your office for a health supervision visit. When you
meet with the mother alone, she asks you about sexuality education for early adolescents.
Of the following, you are MOST likely to counsel the mother that
A. office-based intervention alone is the best approach to promoting sexual health among young
people
D. talking about sexuality to children increases their risk of becoming sexually active
E. talking early and often with children about sexuality is an effective parental counseling
strategy
References:
Klein JD and the Committee on Adolescence. Clinical report: adolescent pregnancy: current
trends and issues. Pediatrics. 2005;116:281-286. Available at:
http://pediatrics.aappublications.org/cgi/content/full/116/1/281
Sieving RE, Oliphant JA, Blum RW. Adolescent sexual behavior and sexual health. Pediatr Rev.
2002;23:407-416. Available at: http://pedsinreview.aappublications.org/cgi/content/full/23/12/407
Question: 169
You are evaluating a 15-year-old girl who complains of malaise, fatigue, and occasional
abdominal discomfort. You diagnosed hypothyroidism due to chronic lymphocytic thyroiditis
(Hashimoto thyroiditis) 6 years ago. She has normal serum immunoglobulin A concentrations. A
tissue transglutaminase antibody study was negative 1 month before this visit, and free
thyroxine and thyroid-stimulating hormone (TSH) values were normal at that time. She has
normal menses. She reports that she has been eating poorly and has lost 5 lb since you saw
her at the beginning of the summer, but she obviously has had a good summer and has a tan.
Of the following, the MOST important laboratory studies to obtain at this time are
References:
Neimann LK. Causes of primary adrenal insufficiency (Addison’s disease). UpToDate Online
14.3. Available for subscription at:
http://www.utdol.com/utd/content/topic.do?topicKey=adrenal/7188&type=A&selectedTitle=4~23
Wilson TA, Speiser P. Adrenal insufficiency. eMedicine Pediatrics Endocrinology. 2006. Available
at: http://www.emedicine.com/ped/topic47.htm
Critique: 169
Patients who have Addison disease may have increased pigmentation that involves the skin and
mucosa (arrow).
Question: 170
A 3-year-old boy has a history of biting his parents’ cheeks when he does not get his way, which
they have always considered cute, calling it “love bites.” His child care teacher has informed the
parents that he is frequently biting other children. The boy’s parents are concerned that he may
be removed from his child care program and ask for your advice about how to stop this
behavior.
D. take the child aside and explain that his behavior is not acceptable and may cause him to be
removed from child care
E. tell the parents it is acceptable to gently bite the child or tap his backside to stop this behavior
quickly
References:
Biters: why they do it and what to do about it. KidSource OnLine™. Washington, DC. National
Association for the Education of Young Children; 1997. Available at:
http://www.kidsource.com/kidsource/content3/biters.p.t.4.html
Shelov S. Emergencies: bites. In: The American Academy of Pediatrics Caring for Your Baby
and Young Child: Birth to Age 5. Elk Grove Village, Ill: American Academy of Pediatrics;
2004:495-496
Question: 171
A mother brings in her child because she found a tick on the girl’s shoulder yesterday and is
worried about Lyme disease. The mother found a site on the Internet that suggests her daughter
needs an antibiotic called ceftriaxone. You assure the mother that only very few children who
suffer tick bites actually develop a tick-associated disease.
Of the following, the manifestation of Lyme disease for which ceftriaxone administration is MOST
appropriate is
A. acute arthritis
B. carditis
E. peripheral neuropathy
References:
Steer AC. Lyme disease. N Engl J Med. 2001;345:115-125. Abstract available at:
http://www.ncbi.nlm.nih.gov/entrez/query.fcgi?db=pubmed&cmd=Retrieve&dopt=AbstractPlus&li
st_uids=11450660
Critique: 171
Question: 172
A 5-year-old boy who has neuroblastoma is admitted to the pediatric intensive care unit for
treatment of fever, neutropenia, and severe hypotension due to Klebsiella pneumoniae sepsis.
Over the last several months, he has received multiple courses of vancomycin and ceftazidime
to treat fever and neutropenia. Antibiotic susceptibility testing of the pathogen shows it to be
susceptible only to the carbapenem and aminoglycoside classes of antibiotics.
References:
Gold HS, Moellering RC Jr. Antimicrobial drug resistance. N Engl J Med. 1996;335:1445-1453
Opal SM, Medeiros AA. Molecular mechanisms of antibiotic resistance in bacteria. In: Mandell
GL, Bennett JE, Dolin R, eds. Mandell, Douglas and Bennett’s Principles and Practice of
Infectious Diseases. 6th ed. Philadelphia, Pa: Elsevier Churchill Livingstone; 2005:253-270
Polk RE, Fishman NO. Antimicrobial management: cost and resistance. In: Mandell GL, Bennett
JE, Dolin R, eds. Mandell, Douglas and Bennett’s Principles and Practice of Infectious Diseases.
6th ed. Philadelphia, Pa: Elsevier Churchill Livingstone; 2005:611-618
Rahal JJ, Urban C, Horn D, et al. Class restriction of cephalosporin use to control total
cephalosporin resistance in nosocomial Klebsiella. JAMA. 1998;280:1233-1237. Abstract
available at:
http://www.ncbi.nlm.nih.gov/entrez/query.fcgi?db=pubmed&cmd=Retrieve&dopt=AbstractPlus&li
st_uids=9786372
Question: 173
One of your patients is a 6-month-old boy who had unilateral hydronephrosis detected
prenatally. Ultrasonography shows moderate hydronephrosis on the right and a normal left
kidney, and voiding cystourethrography shows no reflux, with a normal bladder and urethra.
During a MAG 3/furosemide renal scan, shortly after the furosemide is administered, the infant
becomes extremely fussy and difficult to console.
C. nephrolithiasis
References:
Chevalier RL, Roth JA. Obstructive uropathy. In: Avner ED, Harmon WE, Niaudet P, eds.
Pediatric Nephrology. 5th ed. Philadelphia, Pa: Lippincott Williams & Wilkins; 2004:1049-1078
Critique: 173
Fluoroscopic image from voiding cystourethrography showing a large mass within the contrast-
filled bladder consistent with an ureterocele. There is reflux into both lower pole collecting
systems of duplex kidneys.
Courtesy of D. Mulvihill
Critique: 173
Courtesy of D. Mulvihill
Critique: 173
A fluoroscopic spot film from voiding cystourethrography shows a significant change in urethral
caliber at the level of the posterior urethral folds consistent with valves.
Courtesy of D. Mulvihill
Question: 174
A 14-year-old boy presents to the emergency department with a severe asthma exacerbation
and respiratory failure. Despite intubation and aggressive resuscitation, he develops severe
acidosis, pulmonary edema, and hypoxic encephalopathy. His condition worsens over the next
week, and the parents decide to withdraw care.
Of the following, the risk factor MOST associated with fatal asthma is
A. Caucasian race
References:
Guill MF. Asthma update: epidemiology and pathophysiology. Pediatr Rev. 2004;25:299-305.
Available at: http://pedsinreview.aappublications.org/cgi/content/full/25/9/299
Lasley MV. New treatments for asthma. Pediatr Rev. 2003;24:222-231. Available at:
http://pedsinreview.aappublications.org/cgi/content/full/24/7/222
Critique: 174
Question: 175
A 16-year-old boy is brought to the emergency department because he had a seizure after
returning home from being out with friends. His mother reports that he was agitated and
aggressive when he first arrived home about 45 minutes ago and then became unresponsive
and developed generalized extremity shaking that lasted for approximately 5 minutes. His
friends reported that he “smoked weed,” but they denied any other drug or substance use. In
the emergency department, he is diaphoretic and difficult to arouse, his heart rate is 140
beats/min, his respiratory rate is 30 breaths/min, his blood pressure is 135/95 mm Hg, and his
pupils are mid-sized, equal, and sluggishly reactive. Nystagmus is noted.
Of the following, the MOST likely explanation for his symptoms is that
References:
Endom EE. Inhalant abuse in children and adolescents. UpToDate. Online 14.3. Available at:
http://www.utdol.com/utd/content/topic.do?topicKey=ped_tox/6962&type=P&selectedTitle=3~5
Haynes JF Jr. Medical management of adolescent drug overdoses. Adolesc Med Clin.
2006;17:353-379. Abstract available at:
http://www.ncbi.nlm.nih.gov/entrez/query.fcgi?db=pubmed&cmd=Retrieve&dopt=AbstractPlus&li
st_uids=16814698
Jonker J. Jimson weed. Cornell University Poisonous Plants Informational Database. Available
at: http://www.ansci.cornell.edu/plants/jimsonweed/jimsonweed.html
Marijuana-related emergency department visits by youth. The DAWN Report. Rockville, Md:
Department of Health and Human Services, Substance Abuse and Mental Health Services
Administration, Office of Applied Studies; August 2003. Available at:
http://oas.samhsa.gov/MJ2k3ED.pdf
Shukla P. Marijuana use in children and adolescents. UpToDate. Online 14.3. Available at:
http://www.utdol.com/utd/content/topic.do?topicKey=ped_tox/6529&type=P&selectedTitle=20~24
Critique: 175
Jimson weed (Datura stramonium) contains high concentrations of anticholinergic alkaloids. The
flowers are trumpet-shaped and white to purple. The leaves are large and have irregular teeth
similar to oak leaves.
Courtesy of M. Rimsza
Question: 176
A 5-year-old boy has been ill for 2 days with fever, decreased appetite, and a rash. On physical
examination, you note ulcers on the tongue (Item Q176A) and soft palate, but the gingivae are
spared. You also see oval vesicles with surrounding erythema on the hands (Item Q176B).
A. aphthae
B. hand-foot-and-mouth disease
C. herpangina
D. herpetic gingivostomatitis
E. thrush
Question: 176
Reprinted with permission from Krowchuk DP, Mancini AJ, eds. Pediatric Dermatology. A Quick
Reference Guide. Elk Grove Villaage, Ill: American Academy of Pediatrics; 2007
Question: 176
Courtesy of D. Krowchuk
References:
Abzug MJ. Nonpolio enteroviruses. In: Behrman RE, Kliegman RM, Jenson HB, eds. Nelson
Textbook of Pediatrics. 17th ed. Philadelphia, Pa: Saunders; 2004:1042-1048
Krowchuk DP, Mancini AJ, eds. Hand-foot-and-mouth disease (HFMD)/herpangina. In: Pediatric
Dermatology. A Quick Reference Guide. Elk Grove Village, Ill: American Academy of Pediatrics;
2007:95-98
Paller AS, Mancini AJ. The exanthematous diseases of childhood. In: Hurwitz Clinical Pediatric
Dermatology. 3rd ed. Philadelphia, Pa: Elsevier Saunders; 2006:423-448
Weston WL, Lane AT, Morelli JG. Viral infections. In: Color Textbook of Pediatric Dermatology.
3rd ed. St. Louis, Mo: Mosby; 2002:89-118
Critique: 176
In hand-foot-and-mouth disease, ulcers may appear on the tongue, soft palate, uvula, and
tonsillar pillars.
Reprinted with permission from Krowchuk DP, Mancini AJ, eds. Pediatric Dermatology. A Quick
Reference Guide. Elk Grove Village, Ill: American Academy of Pediatrics; 2007
Critique: 176
Oval or round vesicles that have surrounding erythema occur on the palms and soles in hand-
foot-and-mouth disease.
Courtesy of D. Krowchuk
Critique: 176
Aphthae are painful erosions or ulcers that appear on the lips, gingivae, tongue, palate, or buccal
mucosa.
Courtesy of dermatlas.org
Critique: 176
Critique: 176
Vesicles and ulcers located on the soft palate, tongue, gingivae, lips, and chin are characteristic
of herpetic gingivostomatitis.
Reprinted with permission from Krowchuk DP, Mancini AJ, eds. Pediatric Dermatology. A Quick
Reference Guide. Elk Grove Village, Ill: American Academy of Pediatrics; 2007
Critique: 176
Thrush produces white plaques, often on an erythematous base, located on the tongue and
buccal mucosa.
Courtesy of D. Krowchuk
Question: 177
A 13-month-old infant presents with a 1-month history of chronic diarrhea and weight loss. The
baby tolerated cow milk formula well, but the diarrhea began around the time he was transitioned
to whole milk. There is a family history of multiple food allergies. Physical examination
demonstrates a thin infant whose weight is at the 10th percentile and height is at the 50th
percentile. Stool cultures for enteric pathogens and viruses are negative. Results of complete
blood count, chemistries, and serum immunoglobulin (Ig) A measurement are normal. Celiac
serologies demonstrate a positive antigliadin IgG, negative antiendomysial antibodies, and
negative tissue transglutaminase antibody. A small bowel biopsy demonstrates increased
cellularity of the intestinal lamina propria and partial villous atrophy.
Of the following, a TRUE statement regarding the patient’s small bowel biopsy is that the findings
B. are nonspecific
References:
Murch SH. Protracted diarrhea. In: Wyllie R, Hyams JS, eds. Pediatric Gastrointestinal and Liver
Disease. Philadelphia, Pa: WB Saunders; 2006:492-505
Phillips AD, Smith VV. Intestinal biopsy. In: Walker WA, Goulet O, Kleinman RE, Sherman PM,
Shneider BL, Sanderson IR, eds. Pediatric Gastrointestinal Disease: Pathophysiology,
Diagnosis, Management. 4th ed. Hamilton, Ontario, Canada: BC Decker; 2004:1766-1785
Critique: 177
Top: Normal duodenum, with the villi (long fingerlike projections) (arrow) appearing normal.
Courtesy of J. Goldsmith
Bottom: Duodenal biopsy from a patient who has celiac disease shows atrophy of the villi (arrow)
and increased cellular infiltrate in the lamina propria.
Courtesy of A. Bousvaros
Question: 178
A 9-month-old infant who had been born at 25 weeks’ gestation is receiving daily diuretics and
nasal cannula oxygen with a baseline of 0.1 L/min flow. His mother called this morning, reporting
that he had a temperature of 100.5°F (38.1°C), nasal congestion, increased work of breathing
with a rapid respiratory rate, and “wheezing” cough. You instructed her to increase the oxygen
flow rate to 0.5 L/min and advised her to bring him to your office. On arrival at the clinic, pulse
oximetry reveals an oxygen saturation of 85% at rest. On physical examination, you note
intercostal and subcostal retractions, a respiratory rate of 80 breaths/min, a heart rate of 140
beats/min, and a prolonged expiratory phase with audible wheezing. A copious clear nasal
discharge is present. There is no heart murmur or gallop.
Of the following, the BEST explanation for this child’s presenting signs of respiratory distress is
B. acute sinusitis
C. gastroesophageal reflux
References:
Egreteau L, Pauchard J-Y, Semama DA, et al. Chronic oxygen dependency in infants born at
less than 32 weeks' gestation: incidence and risk factors. Pediatrics. 2001;108:e26-e30.
Available at: http://pediatrics.aappublications.org/cgi/content/full/108/2/e26
Hall CB. Respiratory syncytial virus and parainfluenza virus. N Engl J Med.
2001;344:1917–1928
Haworth SG, Hislop AA. Lung development-the effects of chronic hypoxia. Semin Neonatol.
2003;8:1-8. Abstract available at:
http://www.ncbi.nlm.nih.gov/entrez/query.fcgi?db=pubmed&cmd=Retrieve&dopt=AbstractPlus&li
st_uids=12667825
Kennedy KA, Warshaw JB. Bronchopulmonary dysplasia. In: McMillan JA, Feigin
RD, DeAngelis C, Jones MD, eds. Oski's Pediatrics: Principles & Practice. 4th ed.
Philadelphia, Pa: Lippincott Williams & Wilkins; 2006:321-324
McCarthy CA, Breese Hall C. Respiratory syncytial virus: concerns and control. Pediatr Rev.
2003;24:301-309. Available at: http://pedsinreview.aappublications.org/cgi/content/full/24/9/301
Critique: 178
Respiratory syncytial virus bronchiolitis in which there is hyperinflation and bibasilar and right
upper lobe atelectasis.
Courtesy of D. Krowchuk
Critique: 178
Mortality and chronic oxygen dependency according to gestational age at 28 days after birth
(days of life) and 36, 42, and 48 weeks' postconceptional age. For each band of gestational age,
black bars indicate deceased patients, shaded bars indicate survivors who have chronic oxygen
dependency (COD), and open bars indicate survivors who do not have COD.
Reprinted with permission from Egreteau L, Pauchard J-Y, Semama DA, et al. Chronic oxygen
dependency in infants born at less than 32weeks' gestation: incidence and risk factors.
Pediatrics. 2001;108:e26-e30
Question: 179
An 8-week-old breastfed boy is brought to the clinic for his health supervision visit. His mother
thinks he may be more pale than her other children, but he has otherwise been healthy. Findings
on physical examination and vital signs are normal. He does not appear pale to you. A complete
blood count reveals hemoglobin of 9 g/dL (90 g/L) and a mean cell volume of 85 fL. The
remainder of the complete blood count is normal.
References:
Glader B. Anemias of inadequate production. In: Behrman RE, Kliegman RM, Jenson HB, eds.
Nelson Textbook of Pediatrics. 17th ed. Philadelphia, Pa: Saunders; 2004:1606-1616
Segel GB, Hirsh MG, Feig SA. Managing anemia in pediatric office practice: part 1. Pediatr Rev.
2002;23:75-84. Available at: http://pedsinreview.aappublications.org/cgi/content/full/23/3/75
Question: 180
A 6-month-old infant is hospitalized for recurrent apneic events. Notes in the history indicate
multiple hospitalizations and emergency department visits, although no apnea alarms have been
recorded in the presence of medical personnel. The alarms have occurred only when the
mother is alone with the baby. The respiratory therapist now reports that she walked into the
room as the alarm was going off and found the mother holding her hands over the baby’s mouth
and nose.
A. arrange for video camera surveillance of the child’s hospital room to document any repeat
incidents
B. confront the mother after safely removing the infant from her care
References:
Hall DE, Eubanks L, Meyyazhagan S, Kenney RD, Cochran Johnson S. Evaluation of covert
video surveillance in the diagnosis of Munchausen syndrome by proxy: lessons from 41 cases.
Pediatrics. 2000;105:1305-1312. Available at:
http://pediatrics.aappublications.org/cgi/content/full/105/6/1305
Libow JA. Child and adolescent illness falsification. Pediatrics. 2000;105:336-342. Available at:
http://pediatrics.aappublications.org/cgi/content/full/105/2/336
Sharif I. In brief: Munchausen syndrome by proxy. Pediatr Rev. 2004;25:215-216. Available at:
http://pedsinreview.aappublications.org/cgi/content/full/25/6/215
Question: 181
You are evaluating a 2-year-old girl for fever and fatigue. Her parents report that she has had a
fever for 3 days, a progressive degree of fatigue, loss of appetite, and irritability. On
examination, she has a temperature of 102.3°F (39.1°C), a heart rate of 160 beats/min, a
respiratory rate of 40 breaths/min, and a blood pressure of 90/60 mm Hg. She has dry mucous
membranes, mild intercostal retractions, and a 3/6 holosystolic murmur (Item Q181) at the
cardiac apex. Her liver is palpable 3 cm below the costal margin. Her pulses are weak but
palpable in all extremities.
Of the following, the MOST likely cause of this patient’s clinical presentation is
B. Kawasaki disease
C. meningitis
D. myocarditis
E. rheumatic fever
References:
Kuhn B, Shapiro ED, Walls TA, Friedman AH. Predictors of outcome of myocarditis. Pediatr
Cardiol. 2004;25:379-384. Abstract available at:
http://www.ncbi.nlm.nih.gov/entrez/query.fcgi?db=pubmed&cmd=Retrieve&dopt=AbstractPlus&li
st_uids=15085306
Towbin JA. Cardiomyopathies. In: Moller JH, Hoffman JIE, eds. Pediatric Cardiovascular
Medicine. Philadelphia, Pa: Churchill Livingstone; 2000:753-7617
Question: 182
An 18-month-old developmentally delayed child presents to your office for follow-up after an
emergency department visit the previous day for a first seizure. The seizure began with jerking
of the left arm, followed quickly by loss of responsiveness and jerking of the entire body. It lasted
about 2 minutes and was followed by sleepiness for 4 hours. In the emergency department, the
child was difficult to arouse. Head computed tomography scan showed no acute changes, and
findings on lumbar puncture were normal. The child was afebrile. The examining physician
diagnosed an ear infection and advised the family to follow up with you today. The child was born
at 26 weeks’ gestation. At present, he can sit but is not yet standing. Physical examination
reveals a head circumference of 43 cm, and the head shape is dolichocephalic. Other
remarkable findings on physical examination include hyperreflexia with crossed adduction at
both knees and ankle clonus.
Of the following, you are MOST likely to advise the parents that
B. recurrent seizures in their child most likely will cause brain damage
C. the risk of seizure recurrence for their child is about one in three
D. the risk of seizure recurrence in their child is increased because of his developmental delay
and hyperreflexia
References:
Bluvstein JS, Moshé SL. First unprovoked seizure. In: Maria BL, ed. Current Management in
Child Neurology. 3rd ed. Hamilton, Ontario, Canada: BC Decker; 2005:89-92
Freeman JM. The evaluation of a child with a first seizure. In: Singer HS, Kossoff EH, Hartman
AL, Crawford TO, eds. Treatment of Pediatric Neurologic Disorders. Boca Raton, Fla: Taylor &
Francis Group; 2005:55-60
Johnston MV. Seizures in childhood. In: Behrman RE, Kliegman RM, Jenson HB, eds. Nelson
Textbook of Pediatrics. 17th ed. Philadelphia, Pa: Saunders; 2004:1993-2008
Shinnar S, Berg AT, Moshe SL, et al. The risk of seizure recurrence after a first unprovoked
afebrile seizure in childhood: an extended follow-up. Pediatrics. 1996;98:216-225. Available at:
http://pediatrics.aappublications.org/cgi/content/abstract/98/2/216
Shinnar S, O’Dell C, Mitnick R, Berg AT, Moshe SL. Neuroimaging abnormalities in children with
an apparent first unprovoked seizure. Epilepsy Res. 2001;43:261-269. Abstract available at:
http://www.ncbi.nlm.nih.gov/entrez/query.fcgi?db=pubmed&cmd=Retrieve&dopt=AbstractPlus&li
st_uids=11248538
Stroink H, Brouwer OF, Arts WF, Geerts AT, Peters AC, van Donselaar CA. The first
unprovoked, untreated seizure in childhood: a hospital based study of the accuracy of the
diagnosis, rate of recurrence, and long term outcome after recurrence. Dutch study of epilepsy
in childhood. J Neurol Neurosurg Psychiatry. 1998;64:595-600. Abstract available at:
http://www.ncbi.nlm.nih.gov/entrez/query.fcgi?db=pubmed&cmd=Retrieve&dopt=AbstractPlus&li
st_uids=9598673
Question: 183
You are seeing a 14-year-old boy for a physical examination, which he needs to have completed
to attend summer camp. In screening him for safety issues, you find that he does not use a seat
belt.
Of the following, a TRUE statement about seat belt use by adolescents is that
B. male teens are more likely to use seatbelts than female teens
C. most high school students report that they rarely or never wear a seatbelt when riding as a
passenger in a car
D. most youth who died as occupants in passenger vehicle crashes were not wearing seatbelts
E. motor vehicle collisions rank behind cancer and suicide as the leading cause of death for
teenagers
References:
Committee on Injury and Poison Prevention and Committee on Adolescence. The teenage
driver. Pediatrics. 1996;98:987-990. Available at:
http://pediatrics.aappublications.org/cgi/reprint/98/5/987
Eaton DK, Kann L, Kinchen S, et al. Youth risk behavior surveillance-United States, 2005.
MMWR Morbid Mortal Wkly Surv Summ. 2006;55(SS05):1-108. Available at:
http://www.cdc.gov/mmwr/preview/mmwrhtml/ss5505a1.htm
Funk-Zbinden JM, McIntosh GC, Burns DA, Peterson NM, Katcher ML. Child restraint systems:
an update for physicians. WMJ. 2002;100:47-52. Abstract available at:
http://www.ncbi.nlm.nih.gov/entrez/query.fcgi?db=pubmed&cmd=Retrieve&dopt=AbstractPlus&li
st_uids=11419372
National Highway Traffic Safety Administration. Traffic Safety Facts 2005. Washington, DC:
National Center for Statistics and Analysis. US Department of Transportation; 2005. Available at:
http://www-nrd.nhtsa.dot.gov/pdf/nrd-30/NCSA/TSFAnn/TSF2005.pdf
Question: 184
A 17-year-old boy has had poorly controlled type 1 diabetes (DM1) since the age of 2 years.
When you saw him 2 months ago, his hemoglobin A1c result was 11.3%. He complains of
burning and tingling in his feet, which his diabetologist has told him is sensory diabetic
neuropathy. He asks you about treatment for the burning.
Of the following, you are MOST likely to explain that neuropathic pain will be improved by
References:
Boulton AJ, Vinik AI, Arezzo JC, et al; American Diabetes Association. Diabetic neuropathies: a
statement by the American Diabetes Association. Diabetes Care. 2005;28:956-962. Available at:
http://care.diabetesjournals.org/cgi/content/full/28/4/956
Feldman EL, McCulloch DK. Treatment of diabetic neuropathy. UpToDate Online 14.3. Available
for subscription at:
http://www.utdol.com/utd/content/topic.do?topicKey=neuropat/6605&type=A&selectedTitle=9~37
Glastras SJ, Mohsin F, Donaghue KC. Complications of diabetes mellitus in childhood. Pediatr
Clin North Am. 2005;52:1735-1753. Abstract available at:
http://www.ncbi.nlm.nih.gov/entrez/query.fcgi?db=pubmed&cmd=Retrieve&dopt=AbstractPlus&li
st_uids=16301091
Martin CL, Albers J, Herman WH, et al; DCCT/EDIC Research Group. Neuropathy among the
diabetes control and complications cohort 8 years after trial completion. Diabetes Care.
2006;29:340-344. Available at: http://care.diabetesjournals.org/cgi/content/full/29/2/340
Question: 185
An 11-year-old boy was caught by his parents setting fire to paper in his bedroom wastebasket.
He denied ever doing this before, but his parents found a used box of matches under his bed.
He also is rough with the family pet, often pulling out the dog’s fur. He once grabbed the dog by
his ears and lifted him off the ground. His parents ask your advice on how to handle the boy’s
behavior.
E. suggest that the parents restrict the boy’s extracurricular activities if he misbehaves again
References:
Disorders usually first diagnosed in infancy, childhood or adolescence. In: Diagnostic and
Statistical Manual of Mental Disorders. 4th ed. Text Revision. Arlington, Va: American
Psychiatric Association; 2000:39-134
National Center for Injury Prevention. Youth violence: fact sheet. Atlanta, Ga: Centers for
Disease Control and Prevention; 2006. Available at:
http://www.cdc.gov/safeusa/youthviolence.htm
Sadock BJ, Sadock VA. Disruptive behavior disorders. In: Kaplan and Sadock’s Synopsis of
Psychiatry. 9th ed. Philadelphia, Pa: Lippincott Williams & Wilkins; 2003:1232-1240
Thomas CR, Meyer WJ III, Kaye DL. Aggressive behavior. In: Kaye DL, Montgomery ME,
Munson SW, eds. Child and Adolescent Mental Health. Philadelphia, Pa: Lippincott Williams &
Wilkins; 2002:149-165
Question: 186
You evaluated a child in your clinic 3 days ago for a protracted course of diarrhea. Findings on
your initial evaluation were unremarkable, but you did send stool samples for culture. Today the
laboratory reports that the stool samples are growing Yersinia enterocolitica. When you tell the
mother of these results, she is convinced that her daughter contracted the organism from
contaminated food. She asks what types of foods transmit this organism.
Of the following, infections due to Y enterocolitica MOST likely are associated with the
consumption of
A. beef
B. chicken
C. fish
D. pork
E. shrimp
References:
Question: 187
You are called by the director of a preschool regarding an outbreak of gastroenteritis that
occurred in the 2- and 3-year-old toddler classes. She states that 8 of 20 children developed
vomiting and diarrhea associated with fever several days after the school picnic. All the children
recovered from their illness after about 1 week.
A. adenovirus
B. Bacillus cereus
C. Coxsackievirus
D. rotavirus
E. Staphylococcus aureus
References:
American Academy of Pediatrics. Adenovirus infections. In: Pickering LK, Baker CJ, Long SS,
McMillan JA, eds. Red Book: 2006 Report of the Committee on Infectious Diseases. 27th ed. Elk
Grove Village, Ill: American Academy of Pediatrics; 2006:202-204
American Academy of Pediatrics. Caliciviruses. In: Pickering LK, Baker CJ, Long SS, McMillan
JA, eds. Red Book: 2006 Report of the Committee on Infectious Diseases. 27th ed. Elk Grove
Village, Ill: American Academy of Pediatrics; 2006:239-240
American Academy of Pediatrics. Rotavirus infections. In: Pickering LK, Baker CJ, Long SS,
McMillan JA, eds. Red Book: 2006 Report of the Committee on Infectious Diseases. 27th ed. Elk
Grove Village, Ill: American Academy of Pediatrics; 2006:572-574
Salerno DA, Arnoff SC. Nonbacterial food poisoning. In: Behrman RE, Kliegman RM, Jenson HB,
eds. Nelson Textbook of Pediatrics. 17th ed. Philadelphia, Pa; Saunders: 2004:2375-2377
Thielman NM, Guerrant RL. Clinical practice. Acute infectious diarrhea. N Engl J Med.
2004;350:38-47
Critique: 187
Question: 188
During the routine examination of a 1-day-old term infant, you palpate an abdominal mass. His
growth parameters and blood pressure are within normal limits.
A. bowel duplication
C. neuroblastoma
E. Wilms tumor
References:
Goodyer PR. Renal dysplasia/hypoplasia. In: Avner ED, Harmon WE, Niaudet P, eds. Pediatric
Nephrology. 5th ed. Philadelphia, Pa: Lippincott Williams & Wilkins; 2004:83-92
Schwartz MZ and Shaul DB. Abdominal masses in the newborn. Pediatr. Rev. 1989;11:172-179.
Critique: 188
Courtesy of D. Mulvihill
Question: 189
An 11-year-old girl presents with a 6-month history of coughing, wheezing, and chest tightness.
She usually has these symptoms three times a week during the day, but also wakes up at night
once a month with the same symptoms. The symptoms have improved when she has used her
mother’s beta-2 agonist inhaler, but her parents are worried that she sometimes misses school
because of her difficulty breathing. You suspect asthma.
Based on the frequency of her symptoms, the BEST categorization of this girl’s asthma severity
is
A. exercise-induced asthma
References:
Guill MF. Asthma update: clinical aspects and management. Pediatr Rev. 2004;25:335-344.
Available at: http://pedsinreview.aappublications.org/cgi/content/full/25/10/335
Lasley MV. New treatments for asthma. Pediatr Rev. 2003;24:222-231. Available at:
http://pedsinreview.aappublications.org/cgi/content/full/24/7/222
Critique: 189
Question: 190
During a busy morning, a mother runs into your office with her 4-month-old infant, who is limp
and cyanotic. She tells you that the baby was having difficulty breathing overnight, and on the
way to the office this morning, he stopped breathing in the car. On physical examination, the
infant has apnea and no pulses. You begin bag-valve-mask ventilation with 100% oxygen, and
your nurse begins chest compressions.
Of the following, the MOST appropriate approach to cardiopulmonary resuscitation for this
patient is
References:
Critique: 190
Compression (top) and release (bottom) phases of chest compression using the two-thumb
encircling hands technique.
Reprinted with permission from Kattwinkel J, ed. Textbook of Neonatal Resuscitation. 5th ed. Elk
Grove Village, Ill: American Academy of Pediatrics and American Heart Association; 2006
Question: 191
During the examination of a 2-year-old child, you observe an abnormality of the labia minora
(Item Q191). The patient’s mother reports that the girl has had several episodes of perineal
discomfort and pruritus during the past 6 months. The child’s past medical history is
unremarkable, and she is otherwise well.
Question: 191
Courtesy of S. Sinal
References:
Leung AK, Robson WL, Kao CP, Liu EK, Fong JH. Treatment of labial fusion with topical
estrogen therapy. Clin Pediatr (Phila). 2005;44:245-247. Abstract available at:
http://www.ncbi.nlm.nih.gov/entrez/query.fcgi?db=pubmed&cmd=Retrieve&dopt=AbstractPlus&li
st_uids=15821849
Omar HA. Management of labial adhesions in prepubertal girls. J Pediatr Adolesc Gynecol.
2000;13:183-185. Abstract available at:
http://www.ncbi.nlm.nih.gov/entrez/query.fcgi?db=pubmed&cmd=Retrieve&dopt=AbstractPlus&li
st_uids=11173021
Sanfilippo JS. Vulvovaginitis. In: Behrman RE, Kliegman RM, Jenson HB, eds. Nelson Textbook
of Pediatrics. 17th ed. Philadelphia, Pa: Saunders; 2004:1828-1832
Critique: 191
Labial adhesions (arrows) may result from inflammation due to local irritation.
Courtesy of S. Sinal
Question: 192
You are evaluating a 1-month-old term infant who has persistent jaundice. The parents explain
that his stools were green 2 weeks ago and now are pale yellow. Physical examination findings
are unremarkable, except for a liver that is palpable 2 cm below the costal margin. The infant’s
total bilirubin is 6.1 mg/dL (104.3 mcmol/L) and direct bilirubin is 4.2 mg/dL (71.8 mcmol/L).
Alanine aminotransferase is 240 U/L, and aspartate aminotransferase is 160 U/L. A hepatobiliary
iminodiacetic (HIDA) nuclear medicine scan demonstrates absence of excretion of tracer into
the bowel (Item Q192).
Of the following, the MOST definitive diagnostic test to establish the diagnosis is
A. intraoperative cholangiography
Question: 192
Courtesy of D. Mulvihill
References:
Campbell KM, Bezerra JA. Biliary atresia. In: Walker WA, Goulet O, Kleinman RE, Sherman PM,
Shneider BL, Sanderson IR, eds. Pediatric Gastrointestinal Disease: Pathophysiology,
Diagnosis, Management. 4th ed. Hamilton, Ontario, Canada: BC Decker; 2004:1122-1138
Critique: 192
Anteroposterior views of a hepatobiliary iminodiacetic acid (HIDA) scan. In a normal study (top),
there is tracer activity in the liver and in the bowel, indicating an intact biliary system. In biliary
atresia (bottom), there is intense accumulation of tracer in the liver but not in the bowel. A small
amount of tracer is seen in the bladder.
Courtesy of D. Mulvihill
Question: 193
You are seeing a term 4,500-g large-for-gestational age (LGA) infant in the nursery at 2 hours of
age. His delivery was complicated by a difficult vaginal extraction with forceps assistance, and
he had a shoulder dystocia. Physical examination reveals a large infant who is well perfused and
in no respiratory distress. There is no crepitus along the clavicles. You elicit an asymmetric
Moro reflex with inability to raise the right arm at the shoulder. The infant holds his right arm in
adduction and internal rotation, with pronation of the forearm (Item Q193). His left hand displays
a normal grasp.
Of the following, a TRUE statement about this infant’s condition and prognosis is that
A. brachial plexus injury with palsies affecting the lower arm and hand have a poorer prognosis
than those with isolated upper arm palsy
B. brachial plexus injury with palsies affecting the lower arm and hand is due most commonly to
in utero nerve damage acquired in infants of diabetic mothers and will be permanent
C. clavicular fracture often complicates brachial plexus injury in LGA infants and results in a long-
term shoulder drop
D. complete avulsion of the brachial plexus is the most common injury and requires
microsurgical repair by 3 months of age
E. Erb palsy is the least common form of brachial plexus injury, involves the lower arm and
hand, and requires several months to heal
Question: 193
Courtesy of B. Carter
References:
Chauhan SP, Rose CH, Gherman RB, Magann EF, Holland MW, Morrison JC. Brachial plexus
injury: a 23-year experience from a tertiary center. Am J Obstet Gynecol. 2005;192:1795-1800.
Abstract available at:
http://www.ncbi.nlm.nih.gov/entrez/query.fcgi?db=pubmed&cmd=Retrieve&dopt=AbstractPlus&li
st_uids=15970811
Gurewitsch ED, Johnson E, Hamzehzadeh S, Allen RH. Risk factors for brachial plexus injury
with and without shoulder dystocia. Am J Obstet Gynecol. 2006;194:486-492. Abstract available
at:
http://www.ncbi.nlm.nih.gov/entrez/query.fcgi?db=pubmed&cmd=Retrieve&dopt=AbstractPlus&li
st_uids=16458651
Joyner B, Soto MA, Adam HM. In brief: brachial plexus injury. Pediatr Rev. 2006;27:238-239.
Available at: http://pedsinreview.aappublications.org/cgi/content/full/27/6/238
Mehta SH, Blackwell SC, Bujold E, Sokol RJ. What factors are associated with neonatal injury
following shoulder dystocia? J Perinatol. 2006;26:85-88. Abstract available at:
http://www.ncbi.nlm.nih.gov/entrez/query.fcgi?db=pubmed&cmd=Retrieve&dopt=AbstractPlus&li
st_uids=16407959
Paige PL, Moe PC. Neurologic disorders. In: Merenstein GB, Gardner SL, eds. Handbook of
Neonatal Intensive Care. 6th ed. St. Louis, Mo: Mosby Elsevier; 2006:773-811
Sponseller PD. Bone, joint, and muscle problems. In: McMillan JA, Feigin RD,
DeAngelis C, Jones MD, eds. Oski's Pediatrics: Principles & Practice. 4th ed.
Philadelphia, Pa: Lippincott Williams & Wilkins; 2006:2470-2504
Critique: 193
In Erb palsy, the Moro reflex is absent on the affected (right) side. Additionally, there is adduction
and internal rotation of the arm and pronation of the forearm.
Courtesy of B. Carter
Question: 194
An African-American mother brings her 4-week-old daughter to the emergency department
because of progressive shortness of breath and pallor. She had mild anemia and jaundice
shortly after birth, which was believed to be due to ABO incompatibility (mother is type O-, infant
is type A-). She required phototherapy for 1 day and was discharged without further
complications. Her respiratory rate today is 65 breaths/min and heart rate is 170 beats/min. She
appears pale and mildly icteric and has mild-to-moderate respiratory distress. Complete blood
count reveals a hemoglobin of 5 mg/dL (50 g/L) and a mean cell volume of 95 fL.
Of the following, the MOST likely cause of this child’s severe anemia is
A. ABO incompatibility
B. Diamond-Blackfan anemia
C. hemoglobin SS disease
D. iron deficiency
References:
Bizzarro MJ, Colson E, Ehrenkranz RA. Differential diagnosis and management of anemia in the
newborn. Pediatr Clin North Am. 2004;51:1087-1107. Abstract available at:
http://www.ncbi.nlm.nih.gov/entrez/query.fcgi?db=pubmed&cmd=Retrieve&adopt=AbstractPlus&l
ist_uids=15275990
Stoll BJ, Kliegman RM. Blood disorders. In Behrman RE, Kliegman RM, Jenson HB, eds. Nelson
Textbook of Pediatrics. 17th ed. Philadelphia, Pa: Saunders; 2004:599-606
Question: 195
A 4-year-old boy is brought in for evaluation by his mother. He has had “intoeing” all of his life,
and his grandmother took him to a doctor who prescribed expensive corrective shoes, but he
has not improved. Physical examination reveals no metatarsus adductus or tibial torsion. You
note that he likes to sit on his knees with his legs behind him in a “W” shape.
Of the following, you are MOST likely to find on completely unclothed examination
A. bowed legs
B. equinovarus deformity
C. femoral anteversion
E. waddling gait
References:
Craig CL, Goldberg MJ. Foot and leg problems. Pediatr Rev. 1993;14:395-400. Available at:
http://pedsinreview.aappublications.org/cgi/reprint/14/10/395
Scherl SA. Common lower extremity problems in children. Pediatr Rev. 2004;25:52-62. Available
at: http://pedsinreview.aappublications.org/cgi/content/full/25/2/52
Critique: 195
Metatarsus adductus represents an incurving of the forefoot. Normally, a line bisecting the heel
passes between the second and third toes (left). In metatarsus adductus, the line bisecting the
heel passes more laterally (right).
Reprinted with permission from Scherl SA. Common lower extremity problems in children. Pediatr
Rev. 2004;25:52-62
Critique: 195
To measure hip rotation, the patient is placed in the prone position with the knee flexed. The hip
then is rotated internally and externally.
Reprinted with permission from Gurko LY, ed. Essentials of Musculoskeletal Care. 3rd ed.
Rosemont, Ill: American Academy of Orthopaedic Surgeons; 2005
Critique: 195
Reprinted with permission from Scherl SA. Common lower extremity problems in children. Pediatr
Rev. 2004;25:52-62
Critique: 195
Courtesy of D. Krowchuk
Critique: 195
In Blount disease, there is fragmentation and downsloping of the medial metaphyses of the
proximal tibiae.
Courtesy of E. Anthony
Critique: 195
Courtesy of M. Rimsza
Question: 196
You are conducting a preparticipation evaluation of a 14-year-old girl who is trying out for her
school volleyball program. She is athletic and has never had any health problems. On physical
examination, her height is at the 90th percentile and weight is at the 50th percentile for her age.
Her lungs are clear, and cardiac examination reveals a systolic click and an apical systolic
murmur that is late systolic and graded at 2/6 with radiation to the left axilla (Item Q196).
References:
Lucas RV Jr. Mitral valve prolapse. In: Moller JH, Hoffman JIE, eds. Pediatric Cardiovascular
Medicine. Philadelphia, Pa: Churchill Livingstone; 2000:673-686
Moller J. Mitral valve prolapse. In: Rudolph CD, Rudolph AM, eds. Rudolph’s Pediatrics. 21st ed.
New York, NY: McGraw-Hill Medical Publishing Division; 2003:1797-1798
Critique: 196
When the septum is viewed from the right side of the heart, a small ventricular septal defect is
apparent in the muscular, trabeculized portion of the septum near the apex.
Courtesy of P. Lynch
Question: 197
At 8 am, your nurse urgently calls you to see a child in the waiting room. You come out and
observe a 5-year-old boy whose eyes are glassy and staring off to the right. He is making
chewing movements and has urinated. He is not responding to his mother’s calls or touch. He
then blinks several times and begins to respond, but is clearly confused. His mother explains
that her son has been diagnosed with epilepsy, but she ran out of medication 2 days ago.
Of the following, the MOST appropriate maintenance antiseizure medication for this child is
A. carbamazepine
B. diazepam
C. ethosuximide
D. phenobarbital
E. phenytoin
References:
Friedman MJ, Sharieff GQ. Seizures in children. Pediatr Clin North Am. 2006;53:257-277.
Abstract available at:
http://www.ncbi.nlm.nih.gov/entrez/query.fcgi?db=pubmed&cmd=Retrieve&dopt=AbstractPlus&li
st_uids=16574525
Johnston MV. Seizures in childhood. In: Behrman RE, Kliegman RM, Jenson HB, eds. Nelson
Textbook of Pediatrics. 17th ed. Philadelphia, Pa: Saunders; 2004:1993-2008
Pellock JM, Duchowny M. Partial seizures. In: Maria BL, ed. Current Management in Child
Neurology. 3rd ed. Hamilton, Ontario, Canada: BC Decker; 2005:99-104
Critique: 197
Question: 198
A 13-year-old boy comes to your office for a preparticipation sports physical examination. He is
interested in starting power lifting of heavy weights to prepare himself to play football in a
community recreational league. On his physical examination, you find he is at Sexual Maturity
Rating 2.
Of the following, you are MOST likely to explain to the boy that
A. adolescents should avoid power lifting and body building until they reach physical and skeletal
maturity
B. resistance training should be used alone for the best general health benefits
C. strength training programs for adolescents should not include warm-up and cool-down
periods
References:
American Academy of Pediatrics Committee on Sports Medicine and Fitness. Strength training
by children and adolescents. Pediatrics. 2001;107:1470-1472. Available at:
http://pediatrics.aappublications.org/cgi/content/full/107/6/1470
Council on Sports Medicine and Fitness and Council on School Health. Policy statement: active
healthy living: prevention of childhood obesity through increased physical activity. Pediatrics.
2006;117:1834-1842. Available at:
http://pediatrics.aappublications.org/cgi/content/full/117/5/1834
Question: 199
You are reviewing the growth chart (Item Q199) of a 10-year-old boy who has had type 1
diabetes for 4 years and whose hemoglobin A1c is 8.1% (normal, 3.8% to 6.4%) during his
annual health supervision visit. His mother tells you that he has been eating poorly, and she
thinks he may have lost some weight in the past 6 months. On physical examination, he appears
well but somewhat tired, he has no abdominal masses, and his liver is palpable 1 cm below the
right costal margin.
A. adrenal insufficiency
B. anorexia nervosa
C. celiac disease
D. hypothyroidism
Question: 199
Courtesy of L. Levitsky
References:
Daneman D, Drash AL, Lobes LA, Becker DJ, Baker LM, Travis LB. Progressive retinopathy
with improved control in diabetic dwarfism (Mauriac’s syndrome). Diabetes Care. 1981;4:360-
365. Abstract available at:
http://www.ncbi.nlm.nih.gov/entrez/query.fcgi?db=pubmed&cmd=Retrieve&dopt=AbstractPlus&li
st_uids=7047112
Glastras SJ, Craig ME, Verge CF, Chan AK, Cusumano JM, Donaghue KC. The role of
autoimmunity at diagnosis of type 1 diabetes in the development of thyroid and celiac disease
and microvascular complications. Diabetes Care. 2005;28:2170-2175. Available at:
http://care.diabetesjournals.org/cgi/content/full/28/9/2170
Kaspers S, Kordonouri O, Schober E, et al; German Working Group for Pediatric Diabetology.
Anthropometry, metabolic control, and thyroid autoimmunity in type 1 diabetes with celiac
disease: a multicenter survey. J Pediatr. 2004;145:790-795. Abstract available at:
http://www.ncbi.nlm.nih.gov/entrez/query.fcgi?db=pubmed&cmd=Retrieve&dopt=AbstractPlus&li
st_uids=15580203
Levitsky LL, Misra M. Associated autoimmune disorders in children and adolescents with type 1
diabetes mellitus. UpToDate Online 14.3. Available for subscription at:
http://www.utdol.com/utd/content/topic.do?topicKey=pediendo/19209&type=P&selectedTitle=91~
115
Question: 200
An 8-month-old infant often falls asleep while his mother is feeding him. He tends to sleep longest
during the day and wakes frequently during the night. The parents are sleep-deprived and ask
for your assistance in getting the infant to sleep more during the night.
A. instruct the parents to feed the infant promptly when he awakes at night
B. instruct the parents to keep the child awake more during the day
D. reassure the parents that this is a phase that will pass and recommend a follow-up evaluation
in 2 months
E. recommend that the infant sleep with the parents to minimize nighttime disturbances
References:
Anderson JE. Co-sleeping: can we ever put the issue to rest? Contemp Pediatr. 2000;17:98-
121.
Boyce WT, Shonkoff JP. Developmental-behavioral pediatrics. In: Rudolph CD, Rudolph AM,
eds. Rudolph’s Pediatrics. 21st ed. New York, NY: McGraw-Hill Medical Publishing Division;
2003:401-532
Cuthbertson J, Schevill S. From five to nine months. In: Helping Your Child Sleep Through the
Night: A Guide for Parents of Children From Infancy to Age Five. 1985 New York, NY: Main
Street Books, Doubleday; 1985:53-90
Davis KF, Parker KP, Montgomery GL. Sleep in infants and young children: part one: normal
sleep. J Pediatr Health Care. 2004;18:65-71. Abstract available at:
http://www.ncbi.nlm.nih.gov/entrez/query.fcgi?db=pubmed&cmd=Retrieve&dopt=AbsractPlus&lis
t_uids=15007289
Davis KF, Parker KP, Montgomery GL. Sleep in infants and young children: part two: common
sleep problems. J Pediatr Health Care. 2004;18:130-137. Abstract available at:
http://www.ncbi.nlm.nih.gov/entrez/query.fcgi?db=pubmed&cmd=Retrieve&dopt=AbsractPlus&lis
t_uids=15129213
Ferber R. Helping you child develop good sleep patterns. In: Solve Your Child’s Sleep Problems.
New York, NY: Fireside, Simon & Schuster, Inc: 1985:35-45
Question: 201
As you are examining a 2-year-old boy who is new to your practice, you note a swollen area on
the right side of his neck. You ask the mother about the swelling, and she starts to cry. She
states that this area has been swollen for 2 months despite courses of cephalexin, clindamycin,
and trimethoprim-sulfamethoxazole. Upon further questioning, you learn that the family recently
moved to “town” after living in a rural area for the past 10 years. She denies a history of animal
or tick exposure. Because her husband is highly allergic to cats, they won’t even let their son pet
one. The boy is afebrile, and other than a 3 x 3 cm swelling in the submandibular region of the
right neck, results of the physical examination are normal. The swelling moves freely but is
slightly tender to palpation.
A. Bartonella henselae
B. Francisella tularensis
C. Mycobacterium avium-intracellulare
D. Staphylococcus aureus
E. Streptococcus pyogenes
References:
Critique: 201
Question: 202
A 26-week-old preterm infant has developed line-associated Staphylococcus epidermidis
sepsis.
Of the following, the MOST effective antibiotic regimen for this infant is
A. ampicillin
B. ampicillin + gentamicin
C. ceftriaxone
D. oxacillin + gentamicin
E. vancomycin + rifampin
References:
Stoll BJ, Hansen N, Fanaroff AA, et al. Late-onset sepsis in very low birth weight neonates: the
experience of the NICHD Neonatal Research Network. Pediatrics. 2002;110:285-291. Available
at: http://pediatrics.aappublications.org/cgi/content/full/110/2/285
Question: 203
Voiding cystourethrography in a 9-month-old boy who has new-onset febrile urinary tract
infection reveals grade II vesicoureteral reflux (VUR). The parents ask you about their son’s
prognosis.
A. approximately 80% of children who have newly diagnosed febrile urinary tract infections have
VUR when tested
E. unilateral grade II reflux has a high likelihood of resolution within 5 years of the diagnosis
References:
Elmore JM, Scherz HC, Kirsch AJ. Dextranomer/hyaluronic acid for vesicoureteral reflux:
success rates after initial treatment failure. J Urol. 2006;175:712-715. Abstract available at:
http://www.ncbi.nlm.nih.gov/entrez/query.fcgi?db=pubmed&cmd=Retrieve&dopt=AbstractPlus&li
st_uids=16407036
Lee RS, Diamond DA, Chow JS. Applying the ALARA concept to the evaluation of vesicoureteric
reflux. Pediatr Radiol. 2006;36(supplement 14):185-191. Abstract available at:
http://www.ncbi.nlm.nih.gov/entrez/query.fcgi?db=pubmed&cmd=Retrieve&dopt=AbstractPlus&li
st_uids=16862421
Rushton HG Jr. Vesicoureteral reflux and scarring. In: Avner ED, Harmon WE, Niaudet P, eds.
Pediatric Nephrology. 5th ed. Philadelphia, Pa: Lippincott Williams & Wilkins; 2004:1027-1048
Skoog SJ, Belman AB, Majd M. A nonsurgical approach to the management of primary
vesicoureteral reflux. J Urol. 1987;138:941-946. Abstract available at:
http://www.ncbi.nlm.nih.gov/entrez/query.fcgi?db=pubmed&cmd=Retrieve&dopt=AbstractPlus&li
st_uids=3656575
Question: 204
An 8-year-old girl is admitted to the intensive care unit for a severe asthma exacerbation. As
part of her management, she is placed on continuous albuterol nebulization at 15 mg/h.
Of the following, the MOST likely electrolyte abnormality to expect in this girl is
A. hypercalcemia
B. hypermagnesemia
C. hypernatremia
D. hypoglycemia
E. hypokalemia
The most common adverse reactions to beta2-adrenergic agonists are tremor, tachycardia, and
palpitations. Administration of therapeutic or multiple doses of a beta2-adrenergic also may
cause transient decreases in PaO2, prolonged QTc interval, arrhythmias, and electrolyte
abnormalities.
Continuous administration of a beta2-adrenergic agonist, as described in the vignette, can
result in hyperglycemia due to glycogenolysis, hypomagnesemia, and hypokalemia.
Hypokalemia results from stimulation of the Na+-K+ pump and may be characterized by a
decrease in serum potassium concentrations by 0.4 to 0.9 mEq/L (0.4 to 0.9 mmol/L). Patients
who have preexisting hypoglycemia or relative hypoglycemia due to diuretic therapy may be at
higher risk for arrhythmias during continuous beta2-adrenergic agonist therapy.
Hypoglycemia and hypocalcemia have been reported in isolated cases, but
hypermagnesemia, hypercalcemia, hyponatremia, and hypernatremia have not been reported
during single or continuous beta2-adrenergic agonist administration.
References:
Habashy D, Lam LT, Browne GJ. The administration of beta2-agonists for paediatric asthma
and its adverse reaction in Australian and New Zealand emergency departments: a cross-
sectional survey. Eur J Emerg Med. 2003;10:219-224. Abstract available at:
http://www.ncbi.nlm.nih.gov/entrez/query.fcgi?db=pubmed&cmd=Retrieve&dopt=AbstractPlus&li
st_uids=12972899
Nelson HS. Beta-adrenergic agonists. In: Adkinson NF Jr, Yunginger JW, Busse WW, Bochner
BS, Holgate ST, Simons FER, eds. Middleton’s Allergy Principles and Practice. 6th ed.
Philadelphia, Pa: Mosby Inc; 2003:803-821.
Question: 205
A 2-year-old girl has a choking episode in the waiting room of your office while eating peanuts.
When you evaluate her in an examination room, she appears well, the coughing has resolved,
her respiratory rate is 24 breaths/min, and her oxygen saturation is 97%. Auscultation of her
lungs reveals coarse rhonchi at both lung bases with an end-expiratory wheeze in the right mid-
lung field. Posteroanterior, lateral, and decubitus chest radiographs appear normal.
Of the following, the next MOST appropriate step in the evaluation of this patient is
B. flexible laryngoscopy
D. no further evaluation
E. rigid bronchoscopy
References:
Eren S, Balci AE, Dikici B, Doblan M, Eren MN. Foreign body aspiration in children: experience
with 1160 cases. Ann Trop Pediatr. 2003;23:31-37. Abstract available at:
http://www.ncbi.nlm.nih.gov/entrez/query.fcgi?db=pubmed&cmd=Retrieve&dopt=AbstractPlus&li
st_uids=12648322
Rovin JD, Rodgers BM. Pediatric foreign body aspiration. Pediatr Rev. 2000;21:86-90. Available
at: http://pedsinreview.aappublications.org/cgi/content/full/21/3/86
Tan HK, Brown K, McGill T, Kenna MA, Lund DP, Healey GB. Airway foreign bodies (FB): a 10-
year review. Int J Pediatr Otorhinolaryngol. 2000;56:91-99. Abstract available at:
http://www.ncbi.nlm.nih.gov/entrez/query.fcgi?db=pubmed&cmd=Retrieve&dopt=AbstractPlus&li
st_uids=11115682
Critique: 205
Foreign body aspiration: Chest radiograph during inspiration (top) reveals only slightly greater
expansion of the left lung compared with the right. During expiration (bottom), the affected left lung
remains expanded due to air trapping. The right hemidiaphragm has moved upward, and the
heart and mediastinum have shifted to the unaffected right side.
Courtesy of D. Mulvihill
Question: 206
A 16-year-old boy is concerned about “bumps” on his penis that have been present for several
weeks (Item Q206).
A. observation
B. penicillin parenterally
C. permethrin topically
D. podofilox topically
E. surgical excision
Question: 206
Courtesy of D. Krowchuk
References:
Centers for Disease Control and Prevention. Sexually transmitted diseases treatment guidelines
2006. MMWR Recomm Rep. 2006;55(No. RR-11):62-67. Available at:
http://www.cdc.gov/mmwr/preview/mmwrhtml/rr5511a1.htm
Krowchuk DP, Mancini AJ, eds. Warts. In: Pediatric Dermatology. A Quick Reference Guide. Elk
Grove Village, Ill: American Academy of Pediatrics; 2007:79-83
Moscicki A-B. Impact of HPV infection in adolescent populations. J Adolesc Health. 2005;37(6
suppl):S3-S9. Abstract available at:
http://www.ncbi.nlm.nih.gov/entrez/query.fcgi?db=pubmed&cmd=Retrieve&dopt=AbstractPlus&li
st_uids=16310138
Wellington MA, Bonnez W. Consultation with the specialist: genital warts. Pediatr Rev.
2005;26:467-471. Available at: http://pedsinreview.aappublications.org/cgi/content/full/26/12/467
Critique: 206
In males, genital warts often are located on the shaft of the penis, appearing as papules that have
a pointed surface.
Courtesy of D. Krowchuk
Critique: 206
On mucosal surfaces, genital warts may appear as pedunculated growths that have multiple
finger-like projections (arrow).
Courtesy of M. Rimsza
Critique: 206
Pearly pink penile papules (arrow) are skin-colored papules that are symmetrically distributed on
the glans, usually the corona.
Courtesy of dermatlas.org
Critique: 206
Courtesy of D. Krowchuk
Critique: 206
The chancre of primary syphilis is a painless ulcer that has an indurated border.
Question: 207
A 3-month-old infant who was born at term has had persistent cholestasis since birth. At 6
weeks of age, he had acholic stools. At 2 months of age, intraoperative cholangiography
demonstrated patent bile ducts. Liver biopsy demonstrated globules in the hepatocytes on a
periodic acid-Schiff stain. Currently, his stools are pale yellow, and he is growing at the 10th
percentile for both weight and height. His parents are healthy, but his sister was diagnosed with
liver cirrhosis at 10 years of age. Physical examination reveals a healthy-appearing infant whose
lungs are clear. Cardiac evaluation documents a normal heart rate and rhythm and no murmurs
or gallops. The liver is mildly enlarged, with a span of 6 cm (2 cm below the costal margin), and
the spleen tip is palpable. The patient’s bilirubin is 5.4 mg/dL (92.3 mcmol/L), alanine
aminotransferase is 124 U/L, gamma-glutamyltranspeptidase is 300 U/L, prothrombin time is 12
seconds, and partial thromboplastin time is 29 seconds.
Of the following, this patient’s underlying condition places him at GREATEST risk for the
development of
A. choreoathetosis
B. cryoglobulinemia
C. diabetes mellitus
D. emphysema
E. pancreatitis
References:
Perlmutter DH. Alpha-1-antitrypsin deficiency. In: Walker WA, Goulet O, Kleinman RE, Sherman
PM, Shneider BL, Sanderson IR, eds. Pediatric Gastrointestinal Disease: Pathophysiology,
Diagnosis, Management. 4th ed. Hamiltonm Ontario, Canada: BC Decker; 2004:1376-1401
Perlmutter DH. Alpha-1-antitrypsin deficiency: diagnosis and treatment. Clin Liver Dis.
2004;8:839-859. Abstract available at:
http://www.ncbi.nlm.nih.gov/entrez/query.fcgi?db=pubmed&cmd=Retrieve&dopt=AbstractPlus&li
st_uids=15464658
Question: 208
You are preparing to attend the vaginal delivery of an infant at 40 weeks’ gestation. Artificial
rupture of the fetal membranes 8 hours ago revealed meconium-stained amniotic fluid. The
pediatric resident asks what she should plan to do.
A. always ask the delivering physician to suction the mouth and hypopharynx upon delivery of
the head
B. be prepared to suction the mouth and hypopharynx upon receipt of the infant on the radiant
warmer bed
C. intubate the trachea immediately and suction any meconium in all infants born through
meconium-stained fluid
D. provide bag-and-mask positive-pressure ventilation upon receipt of the infant on the radiant
warmer bed
References:
Care of the neonate. In: Guidelines for Perinatal Care. 5th ed. Elk Grove Village, Ill; Washington
DC: American Academy of Pediatrics; The American College of Obstetricians and
Gynecologists; 2002:187-235
Dargaville PA, Copnell B, for the Australian and New Zealand Neonatal Network. The
epidemiology of meconium aspiration syndrome: incidence, risk factors, therapies, and
outcome.Pediatrics. 2006;117:1712–1721. Available at:
http://pediatrics.aappublications.org/cgi/content/full/117/5/1712
Ehrenkranz RA. The newborn intensive care unit. In: McMillan JA, Feigin RD,
DeAngelis C, Jones MD, eds. Oski's Pediatrics: Principles & Practice. 4th ed.
Philadelphia, Pa: Lippincott Williams & Wilkins; 2006:201-219
Use of resuscitation devices for positive-pressure ventilation. In: Kattwinkel J, ed. Textbook of
Neonatal Resuscitation. 5th ed. Elk Grove Village, Ill; Dallas, Tex: American Academy of
Pediatrics; American Heart Association; 2006
Question: 209
You are evaluating a 5-year-old girl who has a urinary tract infection. She has had four lower
urinary tract infections in the last 2 years, all of which resolved completely with oral antibiotics.
She denies symptoms of urgency and frequency. The only significant finding on her medical
history is constipation. Results of renal ultrasonography and voiding cystourethrography are
normal. Her growth parameters and physical examination findings are normal. You prescribe
oral trimethoprim-sulfamethoxazole.
Of the following, the MOST appropriate additional step to help reduce the incidence of further
urinary tract infection is to
References:
Johnson CE. New advances in childhood urinary tract infections. Pediatr Rev. 1999;20:335-342.
Available at: http://pedsinreview.aappublications.org/cgi/content/full/20/10/335
Shortliffe LMD. Urinary tract infections in infants and children. In: Walsh PC, Retik AB, Baughan
ED, et al, eds. Campbell’s Urology. 8th ed. Philadelphia, Pa: Elsevier; 2002:1846-1884.
Question: 210
A 12-year-old boy presents with an itchy rash that you diagnose as scabies. As he leaves the
examination room, you note that he is limping. He is overweight, and his mother states he has
been playing football to get some exercise. She believes he is limping because he was injured
during football practice several weeks ago and has been complaining of left knee pain. Findings
on physical examination of the knee are normal, but he complains of pain with hip motion.
References:
Kocher MS, Bishop JA, Weed B, et al. Delay in diagnosis of slipped capital femoral epiphysis.
Pediatrics. 2004;113:e322-e325. Available at:
http://pediatrics.aappublications.org/cgi/content/full/113/4/e322
Scherl SA. Common lower extremity problems in children. Pediatr Rev. 2004;25:52-62. Available
at: http://pedsinreview.aappublications.org/cgi/content/full/25/2/52
Tse SML, Laxer RM. Approach to acute limb pain in childhood. Pediatr Rev. 2006;27:170-180.
Available at: http://pedsinreview.aappublications.org/cgi/content/full/27/5/170
Critique: 210
Slipped capital femoral epiphysis is characterized by an upward anterior movement of the femoral
neck on the capital epiphysis (which becomes displaced posteriorly and inferiorly). In the normal
hip (right), a line drawn along the superior margin of the femoral neck transects a portion of the
ossified epiphysis. This does not occur on the affected side (left).
Courtesy of D. Krowchuk
Question: 211
You are evaluating a 12-year-old boy as part of his annual health supervision visit. He has been
in good health. His heart rate is 75 beats/min and blood pressure is 132/82 mm Hg using the
appropriate-sized cuff. His weight is above the 95th percentile, and his height is at the 50th
percentile. He has strong pulses at the right brachial and right femoral regions.
Of the following, the MOST appropriate diagnostic evaluation to pursue at this time is
B. echocardiography
References:
Chobanian AV, Bakris GL, Cushman WC, et al; the National High Blood Pressure Education
Program Coordinating Committee. Seventh report of the Joint National Committee on Prevention,
Detection, Evaluation, and Treatment of High Blood Pressure. Hypertension. 2003;42:1206-
1252. Available at: http://hyper.ahajournals.org/cgi/content/full/42/6/1206
National Heart, Lung, and Blood Institute, National Institutes of Health. Blood Pressure Levels for
Boys and Girls by Age and Height Percentile. Available at:
http://www.nhlbi.nih.gov/guidelines/hypertension/child_tbl.pdf
National High Blood Pressure Education Program Working Group on High Blood Pressure in
Children and Adolescents. The fourth report on the diagnosis, evaluation, and treatment of high
blood pressure in children and adolescents. Pediatrics. 2004;114:555–576. Available at:
http://pediatrics.aappublications.org/cgi/content/full/114/2/S2/555
Question: 212
An 8-year-old boy presents to the emergency department following 3 days of progressive
difficulty walking. He says that his back hurts. Physical examination shows no abnormalities. On
neurologic examination, he is alert, makes good eye contact, and responds to questions
appropriately, but seems distressed. Cranial nerve examination results are normal. On motor
examination, strength is 4/5 in the arms and legs. Reflexes are absent. He cannot rise from the
floor unassisted. Sensory examination findings are normal.
B. electroencephalography
C. electromyography
E. lumbar puncture
References:
Birnkrant DJ. The assessment and management of the respiratory complications of pediatric
neuromuscular diseases. Clin Pediatr (Phila). 2002;41:301-308. Abstract available at:
http://www.ncbi.nlm.nih.gov/entrez/query.fcgi?db=pubmed&cmd=Retrieve&dopt=AbstractPlus&li
st_uids=12086195
Hughes RAC, Raphaël J-C, Swan AV, van Doorn PA. Intravenous immunoglobulin for Guillain-
Barré syndrome. Cochrane Database Syst Rev. 2006;1:CD002063. Available at:
http://www.mrw.interscience.wiley.com/cochrane/clsysrev/articles/CD002063/frame.html
http://www.ncbi.nlm.nih.gov/entrez/query.fcgi?db=pubmed&cmd=Retrieve&dopt=AbstractPlus&li
st_uids=15953319
Sarnat HB. Disorders of neuromuscular transmission and of motor neurons. In: Behrman RE,
Kliegman RM, Jenson HB, eds. Nelson Textbook of Pediatrics. 17th ed. Philadelphia, Pa:
Saunders; 2004:2072-2076
Question: 213
A 16-year-old girl is brought by ambulance to the emergency department with complaints of
vomiting, abdominal cramping, and diarrhea over the past day. She complains of both hot and
cold flashes and muscle aches, and she has had difficulty sleeping for the past several nights.
She was in a drug rehabilitation program 2 years ago, but she denies the use of alcohol or other
drugs now. On physical examination, her temperature is 101.3°F (38.5°C), heart rate is 120
beats/min, respiratory rate is 24 breaths/min, and blood pressure is 145/90 mm Hg. She is alert
but anxious, her pupils are dilated, and she has a slight tremor. Cardiovascular examination
results are normal except for tachycardia. Her lungs are clear to auscultation, and results of her
abdominal examination are within normal limits.
References:
Johnston LD, O’Malley PM, Bachman JG, Schulenberg JE. Monitoring the Future National
Survey Results on Drug Use, 1975-2005: Volume I: Secondary School Students. Bethesda, Md:
National Institute on Drug Abuse; 2006. Available at:
http://www.monitoringthefuture.org/pubs/monographs/vol1_2005.pdf
Schwartz B, Alderman EM. Peripheral brain: substances of abuse. Pediatr Rev. 1997;18:204-
215. Available at: http://pedsinreview.aappublications.org/cgi/content/full/18/6/204
Strasburger VC, Brown RT, Braverman PK, Rogers PD, Holland-Hall C, Coupey SM. Substance
abuse. In: Adolescent Medicine: A Handbook for Primary Care. Philadelphia, Pa: Lippincott
Williams & Wilkins; 2006:147-162
Question: 214
A 5-year-old boy who has nephrotic syndrome required 3 months of prednisone therapy (2
mg/kg per day) to induce remission. He now has been weaned off prednisone for 1 week.
Of the following, the symptom or sign that is MOST indicative of adrenal insufficiency is
A. headache
B. hyperbilirubinemia
C. nausea
D. pruritus
E. weight gain
References:
Lee MM-C, Levitsky LL. Disorders of the adrenal gland. In: Burg FD, Ingelfinger JR, Polin RA,
Gershon AA, eds. Current Pediatric Therapy. 18th ed. Philadelphia, Pa: Saunders Elsevier;
2006:972-979
Neiman LK. Clinical manifestations of adrenal insufficiency. UpToDate Online 14.3. Available for
subscription at:
http://www.utdol.com/utd/content/topic.do?topicKey=adrenal/5492&type=A&selectedTitle=2~100
Wilson TA, Speiser P. Adrenal insufficiency. eMedicine Pediatrics Endocrinology. 2006. Available
at: http://www.emedicine.com/ped/topic47.htm
Question: 215
The parents of a 17-year-old boy bring him to you because he has told his parents that he has a
“computer in his head” that tells him what to do. He refuses to attend school or go outside the
house because of the “scary faces” he sees at times. In your office, the boy is expressionless
but begins to giggle for no apparent reason when you speak to him. Results of his urine drug
screen are negative.
A. Asperger disorder
B. bipolar disorder
C. conduct disorder
D. depression
E. schizophrenia
References:
American Academy of Child and Adolescent Psychiatry. Practice parameter for the assessment
and treatment of children and adolescents with schizophrenia. J Am Acad Child Adolesc
Psychiatry. 2001;40(7 suppl):4S-23S. Abstract available at:
http://www.ncbi.nlm.nih.gov/entrez/query.fcgi?db=pubmed&cmd=Retrieve&adopt=AbsractPlus&li
st_uids=11434484
Groisman AE, Seminatore ML, Cheng TL. In brief: dhildren and adolescents who have
schizophrenia. Pediatr Rev. 2003:24:356-357. Available at:
http://pedsinreview.aappublications.org/cgi/content/full/24/10/356
Sadock BJ, Sadock VA. Disruptive behavior disorders. In: Kaplan and Sadock’s Synopsis of
Psychiatry. 9th ed. Philadelphia, Pa: Lippincott Williams & Wilkins; 2003:1232-1240
Sadock BJ, Sadock VA. Schizophrenia. In: Kaplan and Sadock’s Synopsis of Psychiatry. 9th ed.
Schizophrenia and other psychotic disorders. In: Diagnostic and Statistical Manual of Mental
Disorders. 4th ed. Revised Text. Washington, DC: American Psychiatric Association; 1994:297-
344
Question: 216
An 8-year-old girl presents with a 4-day history of fever, headache, and abdominal pain. Her
mother states that they live in a rural area and have multiple pets, including dogs, cats, horses,
cows, and a pet raccoon. There is no history of tick bites. On physical examination, the girl
appears mildly toxic, has a temperature of 102.2°F (39°C), and has a grade II/VI systolic ejection
murmur best heard on the left side of the sternal border. Her right upper quadrant is tender to
palpation, but there is no hepatosplenomegaly. Findings on her skin and extremity examination
are normal. A complete blood count reveals a white blood cell count of 1.2x103/mcL (1.2x109/L)
with 90% neutrophils and 10% lymphocytes. Her hemoglobin is 10 g/dL (100 g/L), and her
platelet count is 50x103/mcL (50x109/L). Her alanine aminotransferase is 600 U/L, and her
aspartate aminotransferase is 450 U/L. Her amylase and lipase values are normal. Serum
sodium is 133 mEq/L (133 mmol/L), but the remainder of her electrolyte values are normal.
B. Lyme disease
D. tularemia
E. typhus
References:
American Academy of Pediatrics. Ehrlichia and Anaplasma infections (human ehrlichiosis). In:
Pickering LK, Baker CJ, Long SS, McMillan JA, eds. Red Book: 2006 Report of the Committee
on Infectious Diseases. 27th ed. Elk Grove Village, Ill: American Academy of Pediatrics;
2006:281-284
Diagnosis and management of tickborne rickettsial diseases: Rocky Mountain spotted fever,
ehrlichioses, and anaplasmosis - United States: a practical guide for physicians and other health-
care professionals. MMWR Morbid Mortal Wkly Rep. 2006;55(RR04):1-27. Available at:
http://www.cdc.gov/mmwr/preview/mmwrhtml/rr5504a1.htm
Jacobs RF. Human monocytic ehrlichiosis: similar to Rocky Mountain spotted fever but different.
Pediatr Ann. 2002;31:180-184. Abstract available at:
http://www.ncbi.nlm.nih.gov/entrez/query.fcgi?db=pubmed&cmd=Retrieve&dopt=AbstractPlus&li
st_uids=11905291
Question: 217
You are evaluating a 1-hour-old term infant for respiratory distress. The infant was born by
spontaneous vaginal delivery to a 24-year-old para 1 gravida 1 mother who had premature
rupture of membranes 19 hours prior to delivery. The mother had a temperature of 101.5ºF
(38.6°C) at the time of delivery and received one dose of ampicillin. Her group B streptococcal
culture at 34 weeks of pregnancy was negative. You suspect group B streptococcal infection in
the infant.
Of the following, the MOST rapid method for diagnosis in the infant is
B. antigen detection
C. C-reactive protein
D. culture
References:
Baker CJ, Rench MA. Commercial latex agglutination for detection of group B streptococcal
antigen in body fluids. J Pediatr. 1983;102:393-395
Greenberg DN, Ascher DP, Yoder BA, Hensley DM, Heiman HS, Keith JF 3rd. Sensitivity and
specificity of rapid diagnostic tests for detection of group B streptococcal antigen in bacteremic
neonates. J Clin Microbiol. 1995;33:193-198. Available at:
http://jcm.asm.org/cgi/reprint/33/1/193?view=long&pmid=7699040
Lin FY, Azimi PH, Weisman LE, et al. Antibiotic susceptibility profiles for group B streptococci
isolated from neonates, 1995-1998. Clin Infect Dis. 2000;31:76-79. Available at:
http://www.journals.uchicago.edu/CID/journal/issues/v31n1/994414/994414.html
Question: 218
A mother brings in her 3-year-old daughter because of daytime urinary incontinence and
abdominal pain. The mother explained that the girl was toilet trained at 2 years of age. On
physical examination, growth parameters and vital signs are normal, although the girl has mild
suprapubic tenderness without associated costovertebral angle tenderness or sacral dimples.
Urinalysis shows a urine specific gravity of 1.025, pH of 6.5, 2+ blood, 1+ protein, 3+ leukocyte
esterase, and positive nitrite. Urine microscopy demonstrates 5 to 10 red blood cells/high-power
field, 20 to 50 white blood cells/high-power field, and 3+ bacteria.
A. Enterococcus faecalis
B. Escherichia coli
C. Klebsiella pneumoniae
D. Proteus mirabilis
E. Staphylococcus saprophyticus
References:
Craig JC, Hodson EM. Treatment of acute pyelonephritis in children. BMJ. 2004;328:179-180.
Abstract available at:
http://www.ncbi.nlm.nih.gov/entrez/query.fcgi?db=pubmed&cmd=Retrieve&dopt=AbstractPlus&li
st_uids=14739166
Hansson S, Jodal U. Urinary tract infection. In: Avner ED, Harmon WE, Niaudet P, eds. Pediatric
Nephrology. 5th ed. Philadelphia, Pa: Lippincott Williams & Wilkins; 2004:1007-1026
Jones KV, Asscher AW. Urinary tract infection and vesicoureteral reflux. In: Edelmann CM Jr,
ed. Pediatric Kidney Disease. 2nd ed. Boston, Mass: Little, Brown and Company; 1992:1943-
1992
Kimani K. Microbiology and infectious diseases. In: Robertson J, Shilkofski N, eds. The Harriet
Lane Handbook. 17th ed. Philadelphia, Pa: Elsevier Mosby; 2005:411-452
Omokaro SO. Nephrology. In: Robertson J, Shilkofski N, eds. The Harriet Lane Handbook. 17th
ed. Philadelphia, Pa: Elsevier Mosby; 2005:477-506
Question: 219
You have recently diagnosed mild persistent asthma in a 10-year-old boy and decide to initiate
low-dose inhaled corticosteroid therapy. During your discussion with the parents, you review the
potential benefits and adverse effects of inhaled corticosteroids.
A. adrenal suppression
B. cataracts
D. dysphonia
References:
Allen DB, Bielory L, Derendorf H, Dluhy R, Colice GL, Szefler SJ. Inhaled corticosteroids: past
lessons and future issues. J Allergy Clin Immunol. 2003;112(suppl):S1-S40. Abstract available
at:
http://www.jacionline.org/article/PIIS0091674903018591/fulltext?article_id=PIIS00916749030185
91&browse_volume=112&issue_key=S0091-6749%2800%29X0053-
X&issue_preview=no&select1=no&select1=no&vol=
Lasley MV. New treatments for asthma. Pediatr Rev. 2003;24:222-232. Available at:
http://pedsinreview.aappublications.org/cgi/content/full/24/7/222
Schielmer RP, Spahn JD, Covar R, Szefler SJ. Glucocorticoids. In: Adkinson NF Jr, Yunginger
JW, Busse WW, Bochner BS, Holgate ST, Simons FER, eds. Middleton’s Allergy Principles and
Practice. 6th ed. Philadelphia, Pa: Mosby Inc; 2003:887-913
Question: 220
A 5-year-old boy is brought to the office 4 hours after falling off his bicycle. His mother reports
that he was not wearing a helmet, and bystanders said that he did not lose consciousness.
When his friends brought him home, he was tearful and sleepy but was answering questions
appropriately. The mother noted an abrasion on the left side of his head and applied ice to a
small area of swelling on the left temple. Over the last several hours, however, he has become
increasingly confused and has had multiple episodes of vomiting. On physical examination, he is
difficult to arouse, and his right pupil is larger than his left.
Of the following, a TRUE statement regarding this patient’s likely diagnosis is that
E. the injury is caused by laceration of the veins that bridge the dural sinuses and the brain
References:
Dias MS. Traumatic brain and spinal cord injury. Pediatr Clin North Am. 2004;51:271-303.
Abstract available at:
http://www.ncbi.nlm.nih.gov/entrez/query.fcgi?db=pubmed&cmd=Retrieve&dopt=AbstractPlus&li
st_uids=15062672
McCrory P, Johnston K, Meeuwisse W, et al. Summary and agreement statement of the 2nd
International Conference on Concussion in Sport, Prague 2004. Clin J Sport Med. 2005;15:48-
55. Available at: http://multimedia.olympic.org/pdf/en_report_926.pdf
Critique: 220
Axial computed tomography scan (brain window) shows an area of increased attenuation (arrow)
with convex borders characteristic of an epidural hematoma.
Courtesy of D. Mulvihill
Question: 221
An 11-year-old boy presents for evaluation of recurrent oral ulcers (Item Q221), joint pain, and
weight loss for 3 months.
A. Crohn disease
B. cyclic neutropenia
C. hand-foot-and-mouth disease
D. herpangina
E. herpetic gingivostomatitis
Question: 221
Courtesy of dermatlas.org
References:
Hyams JS. Inflammatory bowel disease. In: Behrman RE, Kliegman RM, Jenson HB, eds.
Nelson Textbook of Pediatrics. 17th ed. Philadelphia, Pa: Saunders; 2004:1248-1255
Hyams JS. Inflammatory bowel disease. Pediatr Rev. 2000;21:291-295. Available at:
http://pedsinreview.aappublications.org/cgi/content/full/21/9/291
Critique: 221
Aphthae are painful ulcers or erosions that appear on the lips, gingivae, tongue, palate, or buccal
mucosa.
Courtesy of dermatlas.org
Critique: 221
In hand-foot-and-mouth disease, ulcers may appear on the tongue, soft palate, uvula, and
tonsillar pillars.
Reprinted with permission from Krowchuk DP, Mancini AJ, eds. Pediatric Dermatology. A Quick
Reference Guide. Elk Grove Village, Ill: American Academy of Pediatrics; 2007
Critique: 221
Oval or round vesicles with surrounding erythema occur on the palms and soles in hand-foot-and-
mouth disease.
Courtesy of D. Krowchuk
Critique: 221
Critique: 221
Vesicles and ulcers located on the soft palate, tongue, gingivae, lips, and chin are characteristic
of herpetic gingivostomatitis.
Reprinted with permission from Krowchuk DP, Mancini AJ, eds. Pediatric Dermatology. A Quick
Reference Guide. Elk Grove Village, Ill: American Academy of Pediatrics; 2007
Question: 222
You are following an 11-year-old girl who has Crohn disease involving the stomach, ileum, and
colon. Her maintenance medications are mesalamine and 6-mercaptopurine. Over the past
year, she has received four courses of corticosteroid treatment, but continues to have
intermittent abdominal pain and diarrhea. Upon review of her growth curve, you note that her
height has been the same over the past 12 months. You suspect that the combination of Crohn
disease and corticosteroid therapy has resulted in growth arrest. You discuss your concerns
with her gastroenterologist.
Of the following, the MOST appropriate medication to control this patient’s disease and reduce
her dependence on corticosteroids is
A. cyclophosphamide
B. infliximab
C. mycophenolate mofetil
D. tacrolimus
E. thalidomide
References:
Faubion WA Jr, Bousvaros A. Medical therapy for refractory pediatric Crohn's disease.Clin
Gastroenterol Hepatol. 2006;4:1199-1213. Abstract available at:
http://www.ncbi.nlm.nih.gov/entrez/query.fcgi?db=pubmed&cmd=Retrieve&dopt=AbstractPlus&li
st_uids=16872913
Rufo PA, Bousvaros A. Current therapy of inflammatory bowel disease in children. Paediatr
Drugs. 2006;8:279-302. Abstract available at:
http://www.ncbi.nlm.nih.gov/entrez/query.fcgi?db=pubmed&cmd=Retrieve&dopt=AbstractPlus&li
st_uids=17037946
Question: 223
A 4-hour-old newborn who weighs 1,890 g and was born at 39 weeks’ gestation has a serum
glucose concentration of 25 mg/dL (1.4 mmol/L), resulting in tremors and jitteriness. The child
appears to have intrauterine growth restriction and is small for gestational age (SGA), but is not
ill and exhibits no dysmorphisms. You admit the infant to the special care nursery and order an
intravenous dextrose 10% in water bolus and infusion and some additional laboratory tests. The
complete blood count reveals a hemoglobin of 23 g/dL (230 g/L), hematocrit of 68% (0.68),
platelet count of 150x103/mcL (150x109/L), and white blood cell count of 7x103/mcL (7x109/L)
with a normal differential count.
A. hyperbilirubinemia
B. hypercalcemia
C. hypertension
D. hypokalemia
E. hyponatremia
References:
Cashore WJ. Neonatal hyperbilirubinemia. In: McMillan JA, Feigin RD, DeAngelis C,
Jones MD, eds. Oski's Pediatrics: Principles & Practice. 4th ed. Philadelphia, Pa:
Lippincott Williams & Wilkins; 2006:235-245
Manco-Johnson M, Rodden DJ, Collins SM. Newborn hematology. In: Merenstein GB, Gardner
SL, eds. Handbook of Neonatal Intensive Care. 6th ed. St.Louis, Mo: Mosby Elsevier; 2006:521-
547
Mentzer WC, Glader BE. Erythrocyte disorders in infancy. In: Taeusch HW, Ballard RA,
Gleason CA, eds. Avery’s Diseases of the Newborn. 8th ed. Philadelphia, Pa: Elsevier
Saunders; 2005:1180-1214
Quinn CT, Buchanan GR. Hematopoiesis and hematologic diseases. In: McMillan
JA, Feigin RD, DeAngelis C, Jones MD, eds. Oski's Pediatrics: Principles &
Practice. 4th ed. Philadelphia, Pa: Lippincott Williams & Wilkins; 2006:440-450
Thureen PJ, Anderson MS, Hay WW Jr. The small-for-gestational age infant. NeoReviews.
2001;2:e139-e149. Available at: http://neoreviews.aappublications.org/cgi/content/full/2/6/e139
Question: 224
An 8-year-old boy is brought to your office after falling from a swing at school. He complains of
right leg pain. Physical examination findings are normal, except he walks with a limp. A
radiograph shows no fracture, but a bone cyst is noted.
E. unicameral bone cysts occur most commonly in the humerus and femur
References:
Arndt CAS. Neoplasms of bone. In: Behrman RE, Kliegman RM, Jenson HB, eds. Nelson
Textbook of Pediatrics. 17th ed. Philadelphia, Pa: Saunders; 2004:1717-1722
Carnesale PG. Benign tumors of bone. In: Canale ST, ed. Campbell’s Operative Orthopedics.
10th ed. Philadelphia, Pa: Mosby; 2003:793-812
Springfield DS, Gebhardt MC. Bone and soft tissues tumors. In: Morrissy RT, Weinstein SL, eds.
Lovell and Winter’s Pediatric Orthopaedics. 6th ed. Philadelphia, Pa: Lippincott Williams &
Wilkins; 2006:493-550
Critique: 224
Unicameral (simple) bone cyst: An anteroposterior radiograph of the tibia shows a metaphyseal
bony defect with minimal expansion of the bony contours and few septations.
Courtesy of D. Mulvihill
Critique: 224
Lateral radiograph of the humerus shows a lytic, expansile lesion with thin internal strands and a
thin continuous rim of bone. These features are characteristic of an aneurysmal bone cyst.
Courtesy of D. Mulvihill
Question: 225
An 11-year-old boy has been participating all summer on a swim team. He presents for his third
visit in 2 months because of ear pain and bloody, crusty drainage from the ear. He is afebrile,
and physical examination reveals normal tympanic membranes bilaterally, debris in the ear
canals, and pain during otoscopy. You prescribe fluoroquinolone ear drops.
References:
Hughes E, Lee JH. Otitis externa. Pediatr Rev. 2001:22:191-197. Available at:
http://pedsinreview.aappublications.org/cgi/content/full/22/6/191
Sander R. Otitis externa: a practical guide to treatment and prevention. Am Fam Physician.
2001;63:927-942. Available at: http://www.aafp.org/afp/20010301/927.html
Question: 226
You receive a telephone call from the mother of one of your patients, who tells you that she is 27
weeks pregnant and that her obstetrician has diagnosed a fetal arrhythmia. In discussion with
the obstetrician, you learn that the fetal heart rate is 240 beats/min and that there is a 1:1
relationship between the atrial and ventricular contraction.
B. fetal therapy will require umbilical vessel catheterization to deliver medication to the fetus
E. the development of fetal hydrops would suggest fetal congestive heart failure
References:
Copel JA, Friedman AH, Kleinman CS. Management of fetal cardiac arrhythmias. Obstet
Gynecol Clin North Am. 1997;24:201-211.
http://www.ncbi.nlm.nih.gov/entrez/query.fcgi?db=pubmed&cmd=Retrieve&dopt=AbstractPlus&li
st_uids=9086526
Kleinman CS, Nehgme RA. Cardiac arrhythmias in the human fetus. Pediatr Cardiol.
2004;25:234–251.
http://www.ncbi.nlm.nih.gov/entrez/query.fcgi?db=pubmed&cmd=Retrieve&dopt=AbstractPlus&li
st_uids=15360116
Wren C. Cardiac arrhythmias in the fetus and newborn. Semin Fetal Neonatal Med. 2006;11:182-
190.
http://www.ncbi.nlm.nih.gov/entrez/query.fcgi?db=pubmed&cmd=Retrieve&dopt=AbstractPlus&list
_uids=16530495
Question: 227
A mother brings in her 4-year-old boy because she is concerned about his increasing
clumsiness. He has been previously healthy and achieved developmental milestones on time.
His growth parameters are normal. On physical examination, his mental status is normal, as are
results of cranial nerve and sensory examinations and reflexes. However, he cannot rise from
the floor without using his hands, and his running looks clumsy.
Of the following, the MOST appropriate next test to assess the cause of this child’s symptoms is
A. electromyography
B. lumbar puncture
D. muscle biopsy
References:
Emery AE. The muscular dystrophies. Lancet. 2002;359:687-695. Abstract available at:
http://www.ncbi.nlm.nih.gov/entrez/query.fcgi?db=pubmed&cmd=Retrieve&dopt=AbstractPlus&li
st_uids=11879882
Sarnat HB. Muscular dystrophies. In: Behrman RE, Kliegman RM, Jenson HB, eds. Nelson
Textbook of Pediatrics. 17th ed. Philadelphia, Pa: Saunders; 2004:2060-2068
Critique: 227
Question: 228
An 18-year-old adolescent comes to your office for her precollege health supervision visit. She
tells you that she is especially concerned about dating safety issues and sexual assault
because a friend recently was assaulted.
References:
McIntosh G. Adolescents and violence. In: Osborn LM, DeWitt TG, First LR, Zenel JA, eds.
Pediatrics. Philadelphia, Pa: Elsevier Mosby; 2005:1512-1518
Neinstein LS, Warf C, Sherer S. Rape and sexual abuse. In: Neinstein LS, ed. Adolescent Health
Care: A Practical Guide. 4th ed. Philadelphia, Pa: Lippincott Williams & Wilkins; 2002:1477-1510
Question: 229
A 9-month-old exclusively breastfed baby presents with a seizure. A chest radiograph obtained
to rule out aspiration pneumonia reveals rachitic changes of the ribs.
References:
Calcium, phosphorus and magnesium. In: Kleinman RE, ed. Pediatric Nutrition Handbook. Elk
Grove Village, Ill: American Academy of Pediatrics; 2004: 285-297
Rauch F. Etiology and treatment of hypocalcemic rickets in children. UpToDate Online 14.3.
Available for subscription at:
http://www.utdol.com/utd/content/topic.do?topicKey=pediendo/11968&type=P&selectedTitle=14~
21
Question: 230
During the health supervision visit of an infant, you place her prone on the examination table.
She is able to track your penlight, following it 180 degrees by lifting her head and shoulders off
the table.
Of the following, these developmental milestones are MOST typical for an infant who is
A. newborn
B. 1 month of age
C. 2 months of age
D. 4 months of age
E. 6 months of age
References:
Blasco P. Motor delays. In: Parker S, Zukerman B, Augustyn M. Developmental and Behavioral
Pediatrics: A Handbook for Primary Care. 2nd ed. Philadelphia, Pa: Lippincott Williams & Wilkins;
2005:242-247
Needlman RD. The first year. In: Behrman RE, Kliegman RM, Jenson HB eds. Nelson Textbook
of Pediatrics. 17th ed. Philadelphia, Pa: Saunders; 2004:31-37
Question: 231
You are called to the nursery to examine a newborn who has a large head. Your review of the
maternal records reveals that the mother had negative findings on rapid plasma reagin, human
immunodeficiency virus, and hepatitis B surface antigen testing and was rubella immune. The
finding of interest in the mother’s history is that she has a “bunch of cats” living with her. The
male newborn has a birthweight of 2.2 kg (<5th percentile), a length of 47.5 cm (25th percentile),
and a head circumference of 39 cm (97th percentile). Physical examination reveals
hepatosplenomegaly and generalized adenopathy. The platelet count is 45x103/mcL (45x109/L).
You suspect congenital toxoplasmosis.
A. acute and convalescent serum immunoglobulin (Ig) G titers for Toxoplasma gondii
B. blood culture
E. urine culture
References:
Critique: 231
Retinitis due to toxoplasmosis: Superiorly there is an old lesion. The yellow-white area represents
inflammation and edema and is surrounded by an area of hyperpigmentation. A new lesion is
seen inferiorly, with inflammation and edema but no hyperpigmentation.
Critique: 231
Cytomegalovirus retinitis: White areas, often arranged along the veins, represent edema due to
inflammation. Areas of early scarring (hyperpigmentation) are present.
Question: 232
You are evaluating a 4-year-old girl who has had a temperature of 102ºF (38.9°C), abdominal
pain, and pain on urination. On physical examination, the child appears apprehensive and has a
temperature of 101.8ºF (38.8°C), mild abdominal pain in the suprapubic area, and an
erythematous urethral opening. Laboratory tests demonstrate a peripheral white blood cell count
of 10.0 x103/mcL (10.0 x109/L) with 55% polymorphonuclear leukocytes, 40% lymphocytes,
and 5% monocytes. Urinalysis is positive for blood and leukocyte esterase, and microscopic
examination shows 5 to 10 red blood cells per high-power field (hpf) and 10 to 30 white blood
cells/hpf. You send urine for culture and start therapy with oral cefuroxime axetil. Two days later,
her mother telephones to report that the patient continues to be febrile and to complain of pain
with urination. The urine culture is growing gram-positive cocci in chains.
A. Enterococcus faecalis
B. Escherichia coli
C. Pseudomonas aeruginosa
D. Streptococcus pneumoniae
E. Streptococcus pyogenes
References:
Dobson SRM, Baker CJ. Enterococcal sepsis in neonates: features by age of onset and
occurrence of focal infection. Pediatrics. 1990;85:165-171. Available at:
http://pediatrics.aappublications.org/cgi/content/full/85/2/165
Murray BE. The life and times of the Enterococcus. Clin Microbiol Rev. 1990;3:46-65.
Stevenson KB, Murray EW, Sarubbi FA. Enterococcal meningitis: report of four cases and
review. Clin Infect Dis. 1994;18:233-239. Abstract available at:
http://www.ncbi.nlm.nih.gov/entrez/query.fcgi?db=pubmed&cmd=Retrieve&dopt=AbstractPlus&li
st_uids=8161632
Question: 233
A 4-year-old female presents with fever, chills, and vomiting. She has had abdominal pain and
dysuria for 3 days. Her temperature is 104.2°F (40.1°C), and she has left-sided costovertebral
angle tenderness. Laboratory evaluation reveals a white blood cell count of 18.7 x 103/mcL
(18.7 x 109/L) with 85% neutrophils, 5% bands, 7% lymphocytes, and 3% monocytes. On
urinalysis, the urine specific gravity is 1.025 and pH is 6.5, and there is 2+ blood, 1+ protein, 3+
leukocyte esterase, and positive nitrite. Urine microscopy demonstrates 5 to 10 red blood
cells/high-power field, 50 to 100 white blood cells/high-power field, and 3+ bacteria. Findings on
renal/bladder ultrasonography are normal. After a 3-day hospitalization for administration of
intravenous antibiotics, discharge with a prescription for oral antibiotics is planned.
Of the following, the MOST appropriate study to complete this child’s evaluation is
B. cystoscopy
C. intravenous pyelography
E. voiding cystourethrography
References:
Lee RS, Diamond DA, Chow JS. Applying the ALARA concept to the evaluation of vesicoureteric
reflux. Pediatr Radiol. 2006;36(supplement 14):185-191. Abstract available at:
http://www.ncbi.nlm.nih.gov/entrez/query.fcgi?db=pubmed&cmd=Retrieve&dopt=AbstractPlus&li
st_uids=16862421
Question: 234
A 5-year-old child experiences acute hives, angioedema, and wheezing approximately 15
minutes after receiving his measles, mumps, and rubella (MMR) vaccination. He recovers
completely after prompt treatment with intramuscular epinephrine, intravenous antihistamines,
and corticosteroids. His parents ask what caused this reaction.
Of the following, the MOST likely cause for this child’s reaction is an allergy to
A. egg
B. gelatin
C. latex
D. streptomycin
E. thimerosal
References:
Cox JE, Cheng TL. In brief: egg-based vaccines. Pediatr Rev. 2006;27:118-119. Available at:
http://pedsinreview.aappublications.org/cgi/content/full/27/3/118
Madaan A, Maddox DE. Vaccine allergy: diagnosis and management. Immunol Allergy Clin North
Am. 2003;23: 555-588. Abstract available at:
http://www.ncbi.nlm.nih.gov/entrez/query.fcgi?db=pubmed&cmd=Retrieve&dopt=AbstractPlus&li
st_uids=14753381
Peter G. Immunization practices. In: Behrman RE, Kliegman RM, Jenson HB, eds. Nelson
Textbook of Pediatrics. 17th ed. Philadelphia, Pa: Saunders; 2004:1174-1183
Pool V, Braun MM, Kelso JM, et al; the VAERS Team. Prevalence of anti-gelatin IgE antibodies in
people with anaphylaxis after measles-mumps-rubella vaccine in the United States. Pediatrics.
2002;110:e71. Available at: http://pediatrics.aappublications.org/cgi/content/full/110/6/e71
Question: 235
A 4-year-old child who was struck by a car is unconscious and has a shallow respiratory rate of
6 breaths/min on arrival at the emergency department. You begin bag-valve-mask ventilation
and prepare to intubate the child. The respiratory therapist asks what size endotracheal tube
you would like.
A. 4.5-mm cuffed
B. 5.0-mm cuffed
C. 5.5-mm cuffed
D. 5.0-mm uncuffed
E. 5.5-mm uncuffed
References:
Newth CJ, Rachman B, Patel N, Hammer J. The use of cuffed versus uncuffed endotracheal
tubes in pediatric intensive care. J Pediatr. 2004;144;333-337. Abstract available at:
http://www.ncbi.nlm.nih.gov/entrez/query.fcgi?db=pubmed&cmd=Retrieve&dopt=AbstractPlus&li
st_uids=15001938
Question: 236
You are discussing contact dermatitis due to plants (Rhus dermatitis) with a resident whom you
are supervising.
Of the following, the statement that you are MOST likely to make is that
C. patients who have severe or extensive disease should be treated with a 2-day course of
prednisone
D. the rash of poison ivy may be spread by fluid from within vesicles
E. washing the skin immediately after contact with poison ivy may lessen the severity of the
eruption
References:
Krowchuk DP, Mancini AJ, eds. Contact dermatitis (irritant and allergic). In: Pediatric
Dermatology. A Quick Reference Guide. Elk Grove Village, Ill: American Academy of Pediatrics;
2007:31-38
Paller AS, Mancini AJ. Eczematous eruptions in childhood. In: Hurwitz Clinical Pediatric
Dermatology. 3rd ed. Philadelphia, Pa: Elsevier Saunders; 2006:49-84
Weston WL, Lane AT, Morelli JG. Dermatitis. In: Color Textbook of Pediatric Dermatology. 3rd
ed. St. Louis, Mo: Mosby; 2002:26-43
Critique: 236
Poison ivy typically grows in clusters of three leaflets, hence the saying "leaves of three, let them
be."
Courtesy of D. Krowchuk
Question: 237
A 3-year-old child presents with a history of intermittent painless rectal bleeding. Approximately
once or twice a week, she passes a formed stool that contains up to “a teaspoon” of blood.
Physical examination demonstrates no fissures or hemorrhoids. Hematocrit measurement and
results of coagulation studies are normal. The bleeding persists despite stool softeners.
A. colonoscopy
E. stool culture
References:
Critique: 237
A juvenile polyp (defined by arrows) may cause rectal bleeding and can be identified on
colonoscopy.
Courtesy of A. Bousvaros
Critique: 237
Courtesy of L. Connelly
Critique: 237
A juvenile polyp has been snared during endoscopy and is about to be cauterized.
Courtesy of A. Bousvaros
Question: 238
You admit to the intensive care nursery a male infant who was born at 32 weeks’ gestation and
has respiratory distress. His mother did not receive antenatal steroids and did not have
chorioamnionitis or premature rupture of membranes. Her group B streptococcal colonization
status is unknown. On physical examination, the infant has tachypnea, retractions, and grunting
respirations. There is no heart murmur. Under hood oxygen at an Fio2 of 0.50, his oxygen
saturation by pulse oximetry is 85%. You obtain a chest radiograph.
Of the following, the MOST likely findings on this infant’s chest radiograph are
A. air bronchograms, diffusely hazy lung fields, and low lung volume
C. fluid density in the horizontal fissure, hazy lung fields with central vascular prominence, and
normal lung volume
D. gas-filled loops of bowel in the left hemithorax and opacification of the right lung field
E. patchy areas of diffuse atelectasis, focal areas of air trapping, and increased lung volumes
References:
Aly H. Respiratory disorders in the newborn: identification and diagnosis. Pediatr Rev.
2004;25:201-208. Available at: http://pedsinreview.aappublications.org/cgi/content/full/25/6/201
Gross I. Respiratory distress syndrome. In: McMillan JA, Feigin RD, DeAngelis C,
Jones MD, eds. Oski's Pediatrics: Principles & Practice. 4th ed. Philadelphia, Pa:
Lippincott Williams & Wilkins; 2006:305-309
Welty S, Hansen TN, Corbet A. Respiratory distress in the preterm infant. In: Taeusch HW,
Ballard RA, Gleason CA, eds. Avery’s Diseases of the Newborn. 8th ed. Philadelphia, Pa:
Elsevier Saunders; 2005:687-703
Critique: 238
Underinflation, a "ground glass" appearance, and air bronchograms (arrows) are characteristic of
respiratory distress syndrome.
Courtesy of B. Carter
Critique: 238
Chest radiograph in an infant who has transposition of the great vessels: Prominent pulmonary
vessels (arrows) suggest pulmonary overcirculation.
Reprinted with permission from Aly H. Respiratory disorders in the newborn: identification and
diagnosis. Pediatr Rev. 2004;25:201-208
Critique: 238
Chest radiograph in transient tachypnea of the newborn shows increased pulmonary interstitial
markings and fluid in the interlobar fissures (arrows).
Reprinted with permission from Aly H. Respiratory disorders in the newborn: identification and
diagnosis. Pediatr Rev. 2004;25:201-208
Critique: 238
Plain radiograph of the chest and abdomen in a patient who has congenital diaphragmatic hernia.
There is bowel in the left chest, with displacement of the heart to the right.
Courtesy of B. Carter
Critique: 238
In meconium aspiration, there are areas of atelectasis (arrows) and hyperinflation (seen best at
the bases).
Reprinted with permission from Aly H. Respiratory disorders in the newborn: identification and
diagnosis. Pediatr Rev. 2004;25:201-208
Question: 239
You are talking to a group of pediatric residents about the use of specific radiologic modalities in
pediatric practice. You tell them that ultrasonography is used commonly because it is
noninvasive and does not require sedation.
A. Blount disease
B. Brodie abscess
D. Legg-Calvé-Perthes disease
References:
Scherl SA. Common lower extremity problems in children. Pediatr Rev. 2004;25:52-62. Available
at: http://pedsinreview.aappublications.org/cgi/content/full/25/2/52
Critique: 239
Normal hip ultrasonography findings in an infant. With the infant supine, the ultrasound transducer
is placed over the lateral hip and directed medially, creating a coronal view. Normally, the femoral
head is at least 50% covered by the acetabulum and cannot be moved out of the acetabulum with
stress.
Courtesy of D. Mulvihill
Critique: 239
Ultrasonography of the hip in developmental dysplasia of the hip. With the knee and hip flexed, the
femur is moved posteriorly (ie, Barlow maneuver). In the dislocatable hip, the femoral head
disappears from ultrasonographic view.
Courtesy of D. Mulvihill
Critique: 239
Developmental dysplasia of the hip in a 12-month-old who presented with a limp. The right
acetabulum (affected side) is shallow and steep. The right hip is dislocated superolaterally.
Courtesy of D. Krowchuk
Critique: 239
Legg-Calvé-Perthes disease: A lateral view of the hips demonstrates sclerosis around the left
femoral epiphysis and flattening of the femoral head.
Courtesy of D. Krowchuk
Critique: 239
Slipped capital femoral epiphysis consists of an upward anterior movement of the femoral neck
on the capital epiphysis (which becomes displaced posteriorly and inferiorly). In the normal hip
(right), a line along the superior margin of the femoral neck transects a portion of the ossified
epiphysis. This does not occur on the affected side (left).
Courtesy of D. Krowchuk
Question: 240
A 9-month-old child who had three episodes of otitis media during the winter presents for his
health supervision visit. On physical examination, you note bilaterally transparent tympanic
membranes with good landmarks but see bubbles behind the membranes and note no mobility
on pneumatic otoscopy. The child is otherwise healthy, and results of developmental screening
are appropriate for his age.
B. perform tympanocentesis
E. reassess in 1 month
References:
Daly KA, Hunter LL, Giebink GS. Chronic otitis media with effusion. Pediatr Rev. 1999;20:1985-
1994. Available at: http://pedsinreview.aappublications.org/cgi/content/full/20/3/85
Flynn CA, Griffin GH, Schultz JK. Decongestants and antihistamines for acute otitis media in
children. Cochrane Database Syst Rev. 2004;3:CD001727. Available at:
http://www.mrw.interscience.wiley.com/cochrane/clsysrev/articles/CD001727/frame.html
Leach AJ, Morris PS. Antibiotics for the prevention of acute and chronic suppurative otitis media
in children. Cochrane Database Syst Rev. 2006;4:CD004401. Available at:
http://www.mrw.interscience.wiley.com/cochrane/clsysrev/articles/CD004401/frame.html
Lous J, Burton MJ, Felding JU, Ovesen T, Rovers MM, Williamson I. Grommets (ventilation
tubes) for hearing loss associated with otitis media with effusion in children. Cochrane Database
Syst Rev. 2005;1:CD001801. Available at:
http://www.mrw.interscience.wiley.com/cochrane/clsysrev/articles/CD001801/frame.html
Paradise JL, Bluestone CD. Consultation with the specialist: tympanostomy tubes: a
contemporary guide to judicious use. Pediatr Rev . 2005;26:61-66. Available at:
http://pedsinreview.aappublications.org/cgi/content/full/26/2/61
Paradise JL, Dollaghan, CA, Campbell TF, et al. Otitis media and tympanostomy tube insertion
during the first three years of life: developmental outcomes at the age of four years. Pediatrics.
2003;112:265-277. Available at: http://pediatrics.aappublications.org/cgi/content/full/112/2/265
Thomas CL, Simpson S, Butler CC, van der Voort JH. Oral or topical nasal steroids for hearing
loss associated with otitis media with effusion in children. Cochrane Database Syst Rev.
2006;3:CD001935. Available at:
http://www.mrw.interscience.wiley.com/cochrane/clsysrev/articles/CD001935/frame.html
Question: 241
You are evaluating a 15-year-old girl in your office for her annual health supervision visit. She is
doing well in school and has no complaints about her health, although she would like to lose
weight. She has joined the cross-country running team at school. She is not receiving any
prescription medications but occasionally uses over-the-counter (OTC) cold remedies and
vitamins. On physical examination, she appears thin but is in no distress. Both her height and
weight are at the 25th percentile for her age, although her weight has decreased from the 50th
percentile 1 year ago. Her heart rate is 100 beats/min and blood pressure is 145/95 mm Hg.
Other findings on physical examination are normal.
References:
Chobanian AV, Bakris GL, Cushman WC, et al; the National High Blood Pressure Education
Program Coordinating Committee. Seventh report of the Joint National Committee on Prevention,
Detection, Evaluation, and Treatment of High Blood Pressure. Hypertension. 2003;42:1206-
1252. Available at: http://hyper.ahajournals.org/cgi/content/full/42/6/1206
National Heart, Lung, and Blood Institute, National Institutes of Health. Blood Pressure Levels for
Boys and Girls by Age and Height Percentile. Available at:
http://www.nhlbi.nih.gov/guidelines/hypertension/child_tbl.pdf
National High Blood Pressure Education Program Working Group on High Blood Pressure in
Children and Adolescents. The fourth report on the diagnosis, evaluation, and treatment of high
blood pressure in children and adolescents. Pediatrics. 2004;114:555–576. Available at:
http://pediatrics.aappublications.org/cgi/content/full/114/2/S2/555
Question: 242
A 16-year-old girl presents to the emergency department with the complaint of weakness for 3
weeks. She is having difficulty walking up stairs, particularly in the evening, and she has had
double vision intermittently. She has no pain. On physical examination, this slim adolescent
female appears sad and uninterested, with droopy eyelids. Cranial nerve examination shows
slight limitation of abduction of the right eye, with complaints of double vision on the right, facial
weakness, and a nasal-sounding voice. Motor examination documents 4/5 strength in the
hands, shoulders, and hips. Reflexes are 1+ in arms and legs. Sensory examination results are
normal.
C. lumbar puncture
References:
Hetherington KA, Losek JD. Myasthenia gravis: myasthenia vs. cholinergic crisis. Pediatric
Emerg Care. 2005;21:546-548. Abstract available at:
http://www.ncbi.nlm.nih.gov/entrez/query.fcgi?db=pubmed&cmd=Retrieve&dopt=AbstractPlus&li
st_uids=16096606
Keren R, Zaoutis TE, Bridges CB, et al. Neurological and neuromuscular disease as a risk factor
for respiratory failure in children hospitalized with influenza infection. JAMA. 2005;294:2188-
2194. Abstract available at:
http://www.ncbi.nlm.nih.gov/entrez/query.fcgi?db=pubmed&cmd=Retrieve&dopt=AbstractPlus&li
st_uids=16264160
Mehta S. Neuromuscular disease causing acute respiratory failure. Respir Care. 2006;51:1016-
1021. Abstract available at:
http://www.ncbi.nlm.nih.gov/entrez/query.fcgi?db=pubmed&cmd=Retrieve&dopt=AbstractPlus&li
st_uids=16934165
Morrison LA. Neuromuscular junction disorders. In: Maria BL, ed. Current Management in Child
Neurology. 3rd ed. Hamilton, Ontario, Canada: BC Decker; 2005:399-403
Sarnat HB. Disorders of neuromuscular transmission and of motor neurons. In: Behrman RE,
Kliegman RM, Jenson HB, eds. Nelson Textbook of Pediatrics. 17th ed. Philadelphia, Pa:
Saunders; 2004:2072-2076
Question: 243
An 18-year-old young man comes to your office for his precollege health supervision visit. In
screening for psychosocial risk factors, you find that he has ridden in a car with a drunk driver
and driven himself after having four to five drinks.
A. a blood alcohol level of 0.05% is not considered illegal for driving among those younger than
21 years of age
C. at any blood alcohol concentration, the risk of being involved in a motor vehicle crash is
higher for teen drivers than older drivers
D. teen drivers are more likely to drive after drinking than are older drivers
E. the minimum alcohol purchasing age of 21 years in all states has not been successful in
reducing alcohol-related crashes among teenagers
References:
Committee on Injury and Poison Prevention and Committee on Adolescence. The teenage
driver. Pediatrics. 1996;98:987-990. Available at:
http://pediatrics.aappublications.org/cgi/reprint/98/5/987
DuRant RH, Smith KS. Vital statistics and injuries. In: Neinstein, LS, ed. Adolescent Health Care:
A Practical Guide. 4th ed. Philadelphia, Pa: Lippincott Williams & Wilkins; 2002:126-169
Eaton DK, Kann L, Kinchen S, et al. Youth risk behavior surveillance-United States, 2005.
MMWR Morbid Mortal Wkly Surv Summ. 2006;55(SS05):1-108. Available at:
http://www.cdc.gov/mmwr/preview/mmwrhtml/ss5505a1.htm
Insurance Institute for Highway Safety, Highway Loss Data Institute. Q&A: Teenagers-
Underage Drinking. 2005. Available at: http://www.iihs.org/research/qanda/underage.html
Insurance Institute for Highway Safety, Highway Loss Data Institute. Q&A: Teenagers-General.
2005. Available at: http://www.iihs.org/research/qanda/teens.html
Question: 244
A 3,200-g term infant has an abnormal finding on newborn screening that was obtained at 36
hours of age. The thyroxine (T4) value was 3.5 mcg/dL (45.2 nmol/L) (normal, >7 mcg/dL [90.3
nmol/L]). The thyroid-stimulating hormone (TSH) value was reported by the screening laboratory
as within the normal range.
Of the following, the MOST likely reason for the low T4 value is
References:
Kempers MJ, Lanting CI, van Heijst AF, et al. Neonatal screening for congenital hypothyroidism
based on thyroxine, thyrotropin, and thyroxine-binding globulin measurement: potentials and
pitfalls. J Clin Endocrinol Metab. 2006;91:3370-3376. Abstract available at:
http://www.ncbi.nlm.nih.gov/entrez/query.fcgi?db=pubmed&cmd=Retrieve&dopt=AbstractPlus&li
st_uids=16787990
Ross DS. Euthyroid hyperthyroxinemia and hypothyroxinemia. UpToDate Online 14.3. Available
for subscription at:
http://www.utdol.com/utd/content/topic.do?topicKey=thyroid/9588&type=A&selectedTitle=1~3
Question: 245
You are examining a young boy during a health supervision visit. His mother reports that he
says “mama,” “dada,” ” bye,” “up,” and “ball.” Following the examination, he sits on the floor in
front of his mother while playing with a toy car. When he sees a jack-in-the-box on a shelf, he
points to it. After being instructed to do so by his mother, he brings the jack-in-the-box to her.
Of the following, these developmental milestones suggest that the child is CLOSEST to
A. 12 months of age
B. 15 months of age
C. 18 months of age
D. 21 months of age
E. 24 months of age
References:
Blasco P. Motor delays. In: Parker S, Zukerman B, Augustyn M. Developmental and Behavioral
Pediatrics: A Handbook for Primary Care. 2nd ed. Philadelphia, Pa: Lippincott Williams & Wilkins;
2005:242-247
Needleman RD. The first year. In: Behrman RE, Kliegman RM, Jenson HB, eds. Nelson
Textbook of Pediatrics. 17th ed. Philadelphia, Pa: Saunders; 2004:31-36
Question: 246
A 2-week-old girl presents to the clinic with a history of drainage from her right eye. Her mother
states that she noticed “the white part of her eye” turning red yesterday, but today the eye was
redder and swollen with some drainage. The infant was born at term via normal spontaneous
vaginal delivery without complications and is the mother’s second child. The mother denies any
illness or sexually transmitted disease during her pregnancy, but states that she did smoke one
quarter pack of cigarettes per day. Physical examination reveals an afebrile, healthy-appearing
female whose only abnormality involves her right eye. The upper and lower eyelids are slightly
swollen, and her conjunctiva is erythematous, with a nonpurulent-appearing discharge. You
suspect she has neonatal conjunctivitis due to Chlamydia trachomatis.
Of the following, the MOST sensitive method for establishing the diagnosis is
A. conjunctival culture
References:
American Academy of Pediatrics. Chlamydia trachomatis. In: Pickering LK, Baker CJ, Long SS,
McMillan JA eds. Red Book: 2006 Report of the Committee on Infectious Diseases. 27th ed. Elk
Grove Village, Ill: American Academy of Pediatrics; 2006:252-257
Sexually transmitted diseases treatment guidelines, 2006. MMWR Morbid Mortal Wkly Rep.
2006;55(RR11):1-94. Available at: http://www.cdc.gov/mmwr/preview/mmwrhtml/rr5511a1.htm
Question: 247
You are evaluating a previously healthy 3-year-old girl for white spots in her mouth and a
worsening rash in her vaginal area. Her mother states that except for decreased oral intake and
complaints of itchiness in her vaginal area, the child has had no fever or other systemic
symptoms. She has received three different 10-day courses of oral antibiotics in the last 2
months for a throat infection and otitis media. She completed her last antibiotic course
yesterday. Physical examination shows two small areas of whitish plaques (Item Q247) on her
tongue and right buccal mucosa that cannot be removed easily with a tongue blade. She has no
abdominal tenderness. Her vaginal area is erythematous, with several areas of excoriation; her
hymenal tissue and urethral opening appear normal. The remainder of her physical examination
findings are normal.
C. sexual abuse
D. undiagnosed immunodeficiency
E. viral infection
Question: 247
Courtesy of D. Krowchuk
References:
American Academy of Pediatrics. Candidiasis (moniliasis, thrush). In: Pickering LK, Baker CJ,
Long SS, McMillan JA, eds. Red Book: 2006 Report of the Committee on Infectious Diseases.
27th ed. Elk Grove Village, Ill: American Academy of Pediatrics; 2006:242-246
Fischer-Hoch SP, Hutwagner L. Opportunistic candidiases: an epidemic of the 1980s. Clin Infect
Dis. 1995;21:897-904. Abstract available at:
http://www.ncbi.nlm.nih.gov/entrez/query.fcgi?db=pubmed&cmd=Retrieve&dopt=AbstractPlus&li
st_uids=8645837
Hughes WT, Flynn PM. Candidiasis. In: Feign RD, Cherry JD, Demmler GJ, Kaplan SL, eds.
Textbook of Pediatric Infectious Diseases. 5th ed. Philadelphia, Pa: Saunders; 2004:2569-2579
Question: 248
A mother brings in her 4-year-old son because his eyelids are swollen. On physical examination,
the boy has normal growth parameters, normal blood pressure, bilateral periorbital edema (Item
Q248), and pitting pretibial edema. Laboratory findings include normal electrolyte concentrations,
blood urea nitrogen of 14 mg/dL (5 mmol/L), creatinine of 0.3 mg/dL (26.5 mcmol/L), albumin of
1.9 g/dL (19 g/L), and normal C3 and C4 complement values. Urinalysis reveals a specific
gravity of 1.030, pH of 6.5, 4+ protein, and 1+ blood, and microscopy demonstrates 5 to 10 red
blood cells/high-power field. Antinuclear antibody test results are negative, and serologic tests
are negative for hepatitis B surface antigen, negative for hepatitis C, and nonreactive for human
immunodeficiency virus. A purified protein derivative test is nonreactive after 48 hours.
Of the following, the MOST appropriate treatment for this patient is:
A. diphenhydramine
B. furosemide
C. low-sodium diet
D. prednisone
E. protein-rich diet
Question: 248
Courtesy of M. Rimsza
References:
Ehrich JH, Brodehl J. Long versus standard prednisone therapy for initial treatment of idiopathic
nephrotic syndrome in children. Arbeitsgemeinschaft fur Padiatrische Nephrologie. Eur J Pediatr.
1993;152:357-361. Abstract available at:
http://www.ncbi.nlm.nih.gov/entrez/query.fcgi?db=pubmed&cmd=Retrieve&dopt=AbstractPlus&li
st_uids=8482290
Hodson EM, Knight JF, Willis NS, Craig JC. Corticosteroid therapy in nephrotic syndrome: a
meta-analysis of randomised controlled trials. Arch Dis Child. 2000;83:45-51. Abstract available
at:
http://www.ncbi.nlm.nih.gov/entrez/query.fcgi?db=pubmed&cmd=Retrieve&dopt=AbstractPlus&li
st_uids=10868999
Valentini RP, Smoyer WE. Nephrotic syndrome. In: Kher KK, Schnaper HW, Makker SP, eds.
Clinical Pediatric Nephrology. 2nd ed. London, England: Informa Healthcare; 2007:155-194
Question: 249
You are evaluating a 14-year-old girl for seasonal allergic rhinitis. Despite a regimen of multiple
allergy medications, she continues to have significant sneezing, rhinorrhea, and nasal
congestion. You decide to consult an allergist for further evaluation, specifically aeroallergen skin
testing and consideration for allergy shots.
Of the following, the MOST likely medication to alter the results of aeroallergen skin testing is
a(n)
References:
Atkins D, Leung DYM. Diagnosis of allergic disease. In: Behrman RE, Kliegman RM, Jenson HB,
eds. Nelson Textbook of Pediatrics. 17th ed. Philadelphia, Pa: Saunders; 2004:747-751
Mahr TA, Sheth K. Update on allergic rhinitis. Pediatr Rev. 2005;26:284-289. Available at:
http://pedsinreview.aappublications.org/cgi/content/full/26/8/284
Question: 250
A mother brings her 2-year-old son to the office 30 minutes after spilling a cup of hot coffee onto
his arm and chest. Physical examination reveals a 2x3-cm ruptured blister on his chest that has
an erythematous, tender base and a 3x5-cm area of erythema on his right upper arm.
Of the following, a TRUE statement regarding the management of the burns is that
A. the burns should be cleaned with soap and water after debridement of the blistered area
C. the patient should be referred to a burn center because of the extent of his burns
D. the patient will require skin grafting for the burn on his chest
E. the burn on his upper arm should be dressed with bacitracin ointment and gauze
References:
Klein GL, Herndon DN. Burns. Pediatr Rev. 2004;25:411-417. Available at:
http://pedsinreview.aappublications.org/cgi/content/full/25/12/411
Nagel TR, Schunk JE. Using the hand to estimate the surface area of a burn in children. Pediatr
Emerg Care. 1997;13:254-255. Abstract available at:
http://www.ncbi.nlm.nih.gov/entrez/query.fcgi?db=pubmed&cmd=Retrieve&dopt=AbstractPlus&li
st_uids=9291511
Palmieri TL, Greenhalgh DG. Topical treatment of pediatric patients with burns: a practical guide.
Am J Clin Dermatol. 2002;3:529-534. Abstract available at:
http://www.ncbi.nlm.nih.gov/entrez/query.fcgi?db=pubmed&cmd=Retrieve&dopt=AbstractPlus&li
st_uids=12358554
Critique: 250
Partial-thickness burns, such as that shown here caused by hot water, involve the epidermis and
dermis.
Courtesy of M. Rimsza